You are on page 1of 131

AJC Mid-Year 07

PAPER

1.
2.
3.
4.
5.
6.
7.
L
9.
10.
11.
12.

To what extent should parents be held responsible for their children's actions?

Art has no place in today's modem wodd. Comment.


Should the State interfere in what a person wants to do with his body?
"The road to hell is';ften paved with good intentions". Discuss.

Examine the view that when fighting a war, we should not be concerned
about
principles.
Freedom of choice

is this always good?

Shorrld young people in your country be fearful about the future?

The need for ethical standard among scientists is greater today.


Comment.
Shopping is_a national pastjme for Singapore. How far should
this trend be
encouraged?
Small businesses cannot survive in toclay,s world ot big players.
Do you agree?
Break the rules. How much would you advocate this?
To what extent should the problems of a country be
the world,s concern?

AJC Mid-Year 07
INSERT

ctTrEs
Passage

l:

Peter Hallwrites ..-.

Freud famously said that after 30 years researching the feminine soul, he'd never
answered his real question: what does a woman want? We urbanists could likewise
confess that we've failed to answer our conundrum: what makes cities tick? Why do cities
have brief golden ages, but then languish? Why are Athens or Florence or Vienna no
longer qeative powerhouses? Why have Manchester and Berlin, once workshops of the 5
world, given way to Guangzhou and Shanghai? Why do some citjes seem to retain, or
regain, their power?
Some of us think the answer ljes in number crunching-as with economist Richard Florida,
author of ''fhe Flight of the Creative Class,,'who iound that concentrations of gay people
correlated with urban creativity. The problem js that even jf this approach works io;one
city 10
at one lime, it may not work for others. The other method is to use history, to ask how
precisely it was that great cities came to be great. Here, the danger js that you may
end up
with a series.of unique one-off explanations. The challenge ts to-RnO if tneri,s anyihing
thb
stories have in common_
And they do. Look at creative cities at their zenith: plato's Athens, Michelangelo,s
Florence,
Shakespeare's London, Mozart,s Vienna. All were economic lea;ers, cities";t the
heart of
vast trading empires, places in frenzied transition, magnets for talented people
seeking
fame and fortune. Outsiders made these places what thjy were: ntnens;i vJrstn
greenot
card holders, the noncitizen Metics; the Jews in .190d Vienna; fo.ergn inists in paris
tge. They were patrons because many had made-monet from rrade, 20
::":.19 as anrsts., tney occupied
a speciat marginat position: not at the hean of courfly or
::_ye
anstocratrc estabtishments. yet not entirely shut out either And thus they absorbed
and
reflected the huge tensions between conaervahve and radrcar rorces thZt
ihreateneo to
divide these societies.

tl"-TT

It was the same, but with sub e differences, in the great manufacturjng
cities.. Consider
lvlanchester in 1780, ctasgow in 1850. Detroit in ig.to-, Silicon Vafley
in f-S6d
bassage; egalitarian places open to tate'nt, Jt-improving ano
1L:"sen-eoucattng, :']:t""ilJi9
engaged in learning and innovation through networks that were at once
competitive and cooperative. There are astonishing pa..Gls between L;ncashire
in the
1780s and t790s.and Siticon Va ey in the 1960s anO tne tgZOs. tn Uottr,
one innovation 30
Droughr ronh another in great chains of creativity. places like these flourished
not because
of physical circumstance, but because their peopre demonstrated
innou"tive
energy.
""iejtionat

ylP:t

n;

;"i"

What are-the 2'tslcentury equivalenls? They are the great global megacity regions
mr ion people around. cities ritJLondJn. ruew i"ork ind -Hong as
li:"1"9^r"9
llli,o-"."r
19
Kong. here too ts huge innovative power,_charging through the cores of the great
centraj
cities but also diffusing out into neighboring places ihrough elaborat; networks of
information exchange. This information, generaGd and exchangid and reprocessed,
forms
the raw material of the new urban economic drivers. This nelpei to establish lhe
advanced
services: finance and business services; command and contror tunctions- uotr-li +o
governfient and private business; creative and cultural industries like the media,
higher
education and health care. And these, in turn, generate a vast array of consuimer
services----entertainment, personal, and hospitality:which simuttaneously cater
to the

disposable incomes of affluent residents and to business and leisure tourists.

Ihus, great cities thrive if they do a good job of information processing and knowledge 45
creation. lncreasingly, there is a winner-takes-all rule: top places and their regions grow at
the expense of smalle. minnows in the pond. Success breeds success, aided by builFin
self-reinforcing advantages like a major jnternational airport. One additional factor asserts
itself in a world where information is a homogeneous traded commodity: the English
language. The great English-speaking metropoles, London and New York, increasingly
dominate the global information economy, attracting firms, investment, capital and talent.
But cities and their people have surprised us before. The 21st century, experts agree, will
be the Asian century. China, for sure-lndia, most probably-will win back the foremost
positions they occupied in the pasl history of civilization. Thear past record and present
achievement both suggest that they are powerfully placed in the great race to marry artistic
creativity and technological innovation, despite China's language disadvantage. Their
return will come through the creative power of their great cjties*Shanghai, Beijing,
perhaps l\,4umbai-rapidly rising in the global hierarchy to challenge today's teaders.

50

55

Adapted from Peter Hall's'How Cities become Great,'. (Newsweek International)

Passage 2: George Yeo wntes


Economically, the world is breaking up from empires and big natjen- siates to small states,
provinces and city-regions. Small states, each with a population of less than i0 rnillion,
make up two- thirds of the members of the United Nations. lncreasingly, it is at the level ol
city-regions that competition for human talent and ,nvestments takes place. All over the
world, institutions that evolved in response to the needs of an earlier period of
industrialization are no longer adequate. Smaller, more responsive units of organization
are required. A pattern of competition and cooperation among city-regions will appear, not
unlike the pattern in Europe before the age of nation-statea, with international
organizations Iike the old Hanseatic League providjng loose coordination.
ln this age of city-regions, Singapore,s experience as a city-state becomes usefulto others.

lndeed, we are somewhat surprised by the jnterest in Singapore by big nations such as
China and lndia and by distant places like South Africa, Kazakhsian and the new
Palestinian state. Their interest reflects the increasing fragmentation of the world into cityregions, each of a size and scale comparable to that of Singapore and its 3 million people.
China, for example, is now divided administratively into aity- regions, each of about 2
milljon to 10 million people. These city-regions have considerable autonomy. Each must
sojve probiems ol urban planning, housing, tratsportation, road congestion, education and
policing while attracting investments and creating jobs.

10

15

Singapore, as an independent city-state, has advantages over city-regions that are parts of
nalion-states. The greatest advantage is our ability to control the movement of people into 20
Singapore. lnstead of indiscriminate urban drift, we select migrants based on talent,
income and other criteria. Without this, Singapore would be like many other fast_growing
cities in the Third World, with high crime rates, traffic congestion, slums, prostitution, drug
addiction and severe pollution.

ln this new world, a new balance between rights and duties, independence and 25
interdependence, competition and cooperation wjll have to be found. The ideas of
democracy and socialism will have to be reinterpreted East Asia wjll make a major
contribution to this reinterpretation, not because East Asians are wjser. Almost 1S0 yeirs
of war and revolution have brought untold suffeting to the region. yet precisely beiause
the destruction has been so complete, reconstruction has been made much easier. 30
Singapore, like most of the countries of East Asia, is in a relatively youthful phase of
2

development. lnstitutions are still flexible.

can-do spirit, sometimes bordering

on

foolhardiness. fills the air.

Westeln liberals oflen sneer at Asian forms of democracy which are still relatively young.
Westem democracies prioritize individual rights, unlike the Asian model of group solidarity
However, Western liberals often forget that a democracy without group solidarity can
become a game where wealth is redistributed from the rich to the poor in the form of aid
and from the disorganized citlzens to the organized bureaucracy. Wjthout strong moral
underpinnings supported by the entire community, resentment from the rich and poor alike
will inevitably result. Democracies which see only rights without obligations eventually
destroy themselves.
40
the precise reason for Singapore's version of socialism- ln many ways, Singapore is
socialist, especially in its enormous subsidy of housing, health and education. Socialism
works when jt strengthens group responsibility. lt is dysfunctional when it leads to
individual iresponsibility. lnstead of tl^re Westem social security systems similar to that of 45
the unbreakable communal iron rice bowls in Maoast China, Singapote deliberately works
our welfare policies through the family. The objective is to strengthen the family net, not
weaken it.
Thas is

Treatment of minorities is another aspect of an evolving Eagt Asian democracy that bears
watching. ln a winner-take-all, one-man-one-vote situation, minorities will revolt against
dominant majority. Other ways must be found to ensure fair minorjty representation.
Singapore created Group Representation Constituencies, which forced all major political
parties to field a multiracial slate of candidates in parliamentary elections. ln lndonesia,
pancasila democracy conscjously plays down Javanese dominance. ln the Association of
Southeast Asian Nations, considerable importance is given to consensus-building.

50

ln East Asia today, institutions are still plastic. lvlajor experiments in democracy and

55

socialism are being conducted. Some will succeed, others will fail. lf Western influence had
not affected every facet of life in Asia, this enormous transformation would not have been
possible. ln the same way, the rise of industrial Asia will eventually have far- reaching
effects on the rest of the world, inctuding the West.

Adapted from ceorge yeob "ln Asia and Eisewhere, Smaller Will Be the Better Way to Govern,l
( I nt ern ati o n al H e ratd Tri b u ne
)

Queslions on Possoge I
1. According to paragraph 1, what is the key question that urbanists have failed to answer? Use your
own words as far as possible.
trl

2.

ln your own words as far as possible,

a)
b)

3.

4.
5.
6.

identify two possible methods from paragraph 2 used to find out why cities thrive.
explain the problem with these

methods.

From paragraph 3, state in your own words as


cities have in common.

t3l

fal as possible three characteristics that creative


I3l

Explain in your own words as far as possible what the author means by the .special marginal
position" (line 2'l) that outsiders

occupy.

tlj

Why does the author use the word 'astonishing, (line 29) to describe the parallets between
Lancashire and the Silicon

t1l

Explain what the author means by 'a winner-takes-alt' (line 46).

I2l

Valley?

According to paragraph 7, what are the factors that led the author to believe that the 21st century will
be the Asian century? Use youl own words as far as
I2l

possible.

Queslions on Possoge 2
According to the writer, why should singapore be surprised by the sudden interest from big nations
(tine

S
9

11-12|?

Iil

summarize the factors which conhibute to singapore's success. using material from paragraphs 3-7,
write your summary in no more than 120 words. Use your own word; as far as possible.

singapore's success depends

t8l

on...

10 Give the meaning of each of the io|owing words as ihey are used inthe passage. you may write the
jn

answer a word or short phrase.


From Passage 1:
zenith (line 15)
baggage (line 27)
drivers (line 39).....

From Passage 2:
bordering (line 32)
plastic (line 55)

t5l

althols discuss factors contributing to a cityls success. How far do you agree with their views?
How far do you think Singapore is a great city? lllustrate your arguments by referring both to what
articles
experiences.
I8l

11. Both

you have read in the

and to your own

AJC Mid-Year 07 Paper 2 Answer Scheme


Questions on Passage

paragraph 1, what is the key question that urbanists have failed to


answer? Use your own words as far as possible. [1]
1. According to

Lifted
L3-7 Why do cities have brief golden ages
(1/2), but then languish('1l2)?
OR
Why do some cities seem to retain (1/2),
or regain (1/2), thei. power?

Paraohrased
They failed to identify the reasons tor a
city's rise (1/2) and fall (1/2).
OR
They failed to identify the reasons for a
city's ability to maintain (112J ot teestablish theirdominance/success ('112).

2. ln your own words as far as possible,


a) ldentify two possible methods from paragraph 2 used to find out why cities thrive

l2l
Lifted
..number crunching..
.. use of history....
Stalisiics ol populalion
Refer back io lhe past

b)

ParaDhrased
Analysis of statistic$ or data/ trend ('l)
Study/ examination of past events/ history (1)

>

1m
m

Explain the problem with these methods [1]

Lifted
...even if this approach works for one city
at one time. it may nol work for others ..

Paraohrased
Because it is difficult to find similadties
between cities
OR
The results cannot be applied to all citjes
OR
The methods cannot be used to draw
conclusions.

Finding pasi records


orn
Simiarities between cities would be imposslble lo find
0m
Allhough it is suiiabe for a cily al lhat period, !!j?) may noi be suitable for olhe6 (0)
(0)
Too many dislinci reasons/ d ffcjlies would have difi reasons

3. From paragraph 3, state in your own words a9 far as possible three characteristics
that creative dties have in common [3]
Lifted
(L16- '18)All were economic leaders, cities
at the heart of vast trading empires, places
in frenzied transition, magnets for talented
people seeking fame and fortune.
Excellent economic stalus (12)
Crucial stalus forlrading (O)

Crcalive people
Arts people (0)

11)

ParaDhrased {Anv 3\
Each is an economic power ('l), a centre

forlrading activities (1). These cities


experience franiic ('l12) development (1/2)
and draw skilled personnel (1) in search of
reoutation and wealth.

4. Explain in your own words what the author means by the 'special marginal position"
(line 21) that outsiders occupy ['1]
Lifted

L21-23 : not at the heart of


courtly or aristocratic
establishments, Yet not entirely
shut out either.

Paraphrased
Outsiders seem to occupy a unique peripheral role
among the nobility - socializing within their midst but
not being entirely accepted by them.
OR
Outsiders are not part of the aristocrats (1/2) but
were welcomed bV them anyway (1i2)

X noi aboul physical location

parallels
does the author use the word 'astonishing' (line 29) to describe the
Valley?
[1]
beMeln Lancashire and the Silicon

5 Whv
I ifted

lrnferredl

ParaDhrased

-CGxbt

ahiost two centuries apart (1/2), so they


should be vastly different. Yet they share many
srmilarities. (1/2)

6. Explain what the author means by'a winner-takes-all" (line 46)i2m)

(L46-47) lncreasinglY, there is a


winner-takes-all rulei too olaces
and their regions glg!{q!]he
expense of smaller minnows in
the pond.

Cities who have managed to get ahead in the race


(1/2) \,!ill expand or develop(1i2) ,
leaving no chance to losing crties to calch up/
hinder the gro$4h of others/ monopolise the

author to believe that the


7. Accordrng to paragraph 7, whai are lhe factoB that led the
21e century wtll be the Aslan century? (2m)
Lifted

(L54-55)Their PAg! record and


present achievement both
suggest that they are PgllglbllY
plA!9d in the great race to tr4ry
adistic creativitv and
technoloaical innovation

What they have done prevlously (1/zJ ano now


r112) demonstrates that they have the capablldy
irlzj to orawl comb,ne the merits / origrnal ideas rn
the arts and science (1/2)

Questions on Passage 2
should Singapore be surprised by the sudden interest
why
writer,
8. According to the
,.^nari^nc /li^a 11-12
'.i^
Paraphrased
Lifted
as
such
nations
(L11-12) ... bY !!g
Because even significantly larger nations
China and lndia and by distant places
(1/2) and those who arctaraway \112\ ate
like South Africa, Kazakhstan and the
observinq us.
new Palestinian state
thew ter

@rhesurprlseof

9. Summarize the facto.s which contribute lo Singapore's success


usinl materiattrom paragraphs 3-7, write your summary in no more than 120 words
I8l

Lifted
..independent city-state..

our ability to control movement of


people into Singapore. (120-21)
...

lnstead of indiscriminate urban


drift, we select migrants based on
talent, income and other criteria...
... relatively youthful Phase of
development (L31)

..flexible (132)

..can-do spirit (132)

....a democracy without grouP


solidarity... (136)

Paraphrase
The sove.reign nature of Singapore as opposed
to other city-states who are subjected to the
sovereiontv of the country.
strict immtgration rules

w-ave

And we grant entry only to eligible applicants/


based on criteria of ability/ merit
ln terms of progress, Singapore is still in its
budding period. ,

And hence, Singapore is willing to accept


changes
And remains adventurous/ gung-ho/ willing to ky
Singapore's success is also attributed to their
people working towards a common goal
bond (0)

strong moral underpinnings (L38)

while being guided by the same set (1/2) of


values(1/2) that are firm/ unwavering/
unchanging/ firm (1/2)

...Singapore's version of
socialism... (L41)
...works our welfare policies through
+h- {.mih, /l 1a\

Singapore has a peculiar brand of socialism


(1t2)
which strengthens group responsibility ('112)
through strengthening the family net (1/2)

10

...minorities will revolt (L50)


......ensure fair minoraty
reoresentation. (L51)

Total of 11 marks

Singapore ensures stability (1/2) through


ensuring that all minority groups participate in
the oolitical svstem. ('112)

10. Give the meaning of the following words as ihey are used in Passage 1 and Passage

You mav write vour answer in one word or a short phrase. (5m
1t2
0
peaU
high poinu furthesu
(P'1,
maru
line15)
highest
zenith
prime/ most
top
successful period of
time/ acme/ apex/
oinnacle
weight
(line27)
something that
burden /
baggage
encumbrances
holds you back/
obstacle
force
catalysu impetus/
driver (line39)
propeller/ propellant
(idea of force must
be there)
resembling
extreme proximity/
bordering (P2, line
elmost like/..close
32)
to/ edoino
plastic (line 55)
malleable/ subject
to change/ plianv
flexible

Both authors discuss factors contributing to a city's success How far do you agree
with their views? How far do you think Singapore is a great city? lllustrate your
arguments by referring both to what you have read in the articles and to your own
experiences.
'1

1)

Question requirements:
Make a stand for both questions
Cite both

fl

2.

Economic leadcrs

(Ll6)

Singapore, as an indpendent ciry-state, has advantages


over city-regions thal arc parts ofnalion _ states (Ll920)

Citics

ofvast rradin8 empires (1,16-17)

Our ability to controlthe movement ofpeople into


Singapore (L20-21)
lnstead of ind iscriminate urban drift, we select
migrants based on talert, income and ollrer criteria

at the heart

(L2t-22\

3.

Places in frenzied transition

rates, little traffic congestion, no


slums, prostitulion and drug addiction arc not serious

(Ll7)

lmplicd: Iow crime

problems, no pollulion (23-24)


Magnets for talented people seeking fame and fortune

In a relatively youthful phase ofdevelopment (1,3l -12

Ll7-18
Outsiders made thcse places what they were

(Ll

lnstitutions are still flexible (L32) ), institutions are

.
.

6.

7.

ioutsiders] occupied a special marginal position:


not at tho heart of courtly or arislocratic
establishments, yet not entirely shut out either'
They absorbed and reflected huge tnsions
belween conservative and radical forces that
threalencd to divide these societies

stillplastic (L55)

Manufacturing cities
Places without arisiocratic baggage (L27)

A can-do-spirit, somelimes bordering on foolhardiness,


fills the air (32-31)

Egalitarian places open to talent (L27)

Singapore's version ofsocialism


enormous subsidy ofhousing, health and

.
.
8.

Sell improving and self-educating (L28)

.
9.

engaged in leaming and innovation through


networks that were at once conpetitive and

education. (L42)
Strengthens goup responsibilify (L47)
Works wclfare policies through the family
strengthen the family net (47-48)

Treatment of minorities:

Ensute fair minority representation through

GRCs (52)

coopcrative
l lroir peoplc demonslratcd exceptional
innovative energy

Huge innovative power, charging through the corcs ol


the great central cities but also diffusing out into the
neighbouring places through elaborate networks of
exchangc (L36-38)

'nibrmation
]0 Advanced services: finance and business scrvice,
command and control lunctions both in government
and privatc business;creative and cultoral industries
like the media, higher education and health care. (1,39-

42\

tl

Vasl array of consumer srvices cntertaiDmenl,


personal and hospitality - which cater to the disposable
incomes ofaffluent residenls and to business and
leisure tourists. (L,l2-44)

t2 Creat cities thrive ifthey do a goodjob olinformation


processing and knowledge creation. (L45-46)

ll

'fhe Englisb lang.rage. The great English speaking


metropoles increasingly dominate the global
information economy, attracting fi rms, investment,
oapital and talnt ((L49-51)

14 Marry artistic creativily and technological innovation

(L55-56)

Passage 1:
RE: 'magnets for talented people" (Line 17)
EV/ EX: Agrees to this being an important factor for success. Student can discuss the
importance of foreign talent in cosmopolitan cities in the world today.
But any 'attractive' city may very well attract the'wrong people' and hence lead to the
problems raised by Yeo (L23-24 - high crime rates....). Singapore has done well in
preventing such problems from occurring through their stringent immigration policies
Foreiqn t;lent plays a major role in ensuring Singapore's success Through tilling up of
jobs that the educated Singaporeans do not want (eg. Nursirg or labourers) to plugging
ihe gap left behind by the aging population, Singapore has managed to maintain
ecoiomic development at a sufficient level and till today remains a "magnet for talented
people". Our attractiveness, thus, is testament of our success
Other points discussed:
l\reritocracy
Multiculturism
Hall (impt of creativity) - lnnovation / eg. Spring Singapore; creative community
Singapore's emphasis on lifelong learning in relation to Hall's view in para 4 ("engage in
learning")

cJc

Mid-Year 07 Paper

1.

How far do you agree that terrorists should be pitied, not hated?

2.

An effective government requires a free press.' Do you agree?

3.

"Human life and dignity are sacrosanct." Do you agree thatthis


principle is overemphasised today?

4.

Environmental conservation is more important than economic


developmeni today. Discuss.

5.

"

6.

'Globalisation privileges the rich more than the poor.' How far is this
true?

7.

To what extent should the decision to get marrie'd be an individual


choice?

L
L

Do people rely too much on medical science for the ills of life?

Women do not need equality today. Men do." What are your views?

Should extreme sports be banned?


stumbling block for young people is their sense of hopelessness.'
To what extent is this true with regard to Singaporean youth today?

10. "The

11. Are

museums still important in modern society?

12. Why write?

END OF PAPER

,/tr

CJC Mid-Year 07 Paper I (Version 2)


"Singapore's education system has failed to address the needs
people." How far do you agree with the statement?

l)

.
.
r
2)

of our young

Essays should evaluate the extent to which policies, curriculum, teaching


strategies etc havg catered to what young people require or want out of life or
find necessary or useful in life (these wants must bejustified).
Weak students who merely list the failings of the education system without
linking it to the needs oflhe yonng people will not pass.
For balance, essays should examine both the successes and failings of the
Singapore education system in relation to what the young people require.

"lndividuals must do more to reduce environmental danage." Discuss.


Keywords: MUST DO MORE. Answers should analyse the necessity of
individual efforts in conserr'ing the environment.
Answers should evaluate and assess what is cunently being done before
deciding if more shouid be done or what more should be done in the near

.
.

r)

future.
Students need to evalllale the extent to which the individual can effect any

change in the conservation of the environment. Students may consider


individuals linking up with olher $oups to increase their influence and
ability to cut down on environmental damage.
A balanced essay should have a comparative analysis ofthe efforts ofthe
individual in relation to the govemment and other environmenlal groups.
For example, through govenment lobbies and not voting for political
parties that do not support or cncourage envircnmental conservation.

How much influence do parents still have on the lives oftheir children?
Students need to compare and conhast the degree ofcontrol/ sway parents
have on thc lives of their children in ateas such as studies, work, leisure
and character in relation to parents ofthe previous generatiot
Better essays should provide justification lor their stand by citing reasons
such as changing societal values, disintegralion of families, changing
nalule of work, mass media influences, inlbrmation technology, alcohol
irnd drugs, educational changes etc.
A weak essay will simply list thc positive,r ncgative things parents do and
avoid the discussion ofparenlal issues in relation 10 changes ovcr time

.
.

4) "Advefiisements aro manipulative and misleading." Discuss.

Students should evaluate the degree

to which

advertisements exploit
consumers by playing on their minds and providing inacourate information
is necessary to identiry and evaluate the techniques of persuasive
advertjsing and give specific examples as theirjustification.

. lt

IJ

For balance, students should show how advertisements can be honest as well
as inlbrmative of new products and services available or relay impoflant
information and policies to the masses.
Weak essays will merely list advertisements that are deemed to be misleading
without justificalion or analysis or make a simplistic comparison between
persuasive and informative advertising.

5) Are youths loday complacent about the future?

.
.
.

Students should evaluate if the atl.itudes of youths today reflect


overconfidence, smugness and a lack of worry about the challenges of the

fuhlre.
They should compare and contrast youths oftoday with previous generations
of young people in lerms of their attitude / behaviour. Thc 'Future' should
encompass political, environmental, economic and social scenarios.
Specific examples oI youth complacency include poor voter turnout, politicai
apathy, lack of community/environmental awareness and social indifference.
Examples slrould not only come from Singapore.
A wearl essay will limit itselfto personal anccdotes.

6) Assess the impact ofmodeln methods olltransport on societies.


Students should examine thc ways modern methods of transport like Mass
Rapid Transit, cars, aeroplanes, buscs, even energy-saving modes oftransport

havc changcd societies and weigh both the positive and negative effects of

.
.

these changes.

Sludents should consider the economic, social, political, cultural and


environmcntal impact and also its effects on the lifestyles ofindividuals.
Weak students wiil be purely descriptive of the different modes of transport
and their advantages and disadvantages.

7) With globalization, talent is becoming more mobilc. ls this a positive trend?


Students must show an unde$tanding of global compctition for the best and
the brightest. They should evaluate if such a phenomenon is positive for the
individual and the nations that gain and lose the 'lalent'.
Examples of negative elfects would be loss of cultue, palriotism and family

r
.

values. Positive eflects could include the acquisition of new knowledge,


expertisc and better opportunities, leaming of new cultures and countrics
becoming m.rre cosmopolilan.
Good arswe$ would consider the effects ofa reverse bmill drain and assess i1.
Other arguments can be that of the rise of a common intellectual pool, a
common language, a common culture that may result fiom the mobility. They
should also give a global perspective and a range ofexamples.
Weak answers may just iist some consequences of the movement of talent
without much evaluation

8) Can television promote healthy lifestyles?


Students have to discuss the potential of television to actively or indirectly
encourage a healthy way ofliving.
Students should look at programmes that actively promote healthy living and
representations of healthy or unhealthy living (smoking, drinking, bad eating
habits) in the content of the programmes
Healthy lifestyles would largely refer to a way of life that leads to physical,
fitness, mental alertness the idea of a sound mind in a sound body through
active participation in areas such as spofis, travel, yoga and dance.
Weak ossays may just list the vaious types ofhealthy programmes to justif

.
.

the stand.

e) Account for the phcnomenon ofviolence in schools today.


Students must give an cxplanation/ reasons for the occurrence of violence
in many schools in the world today (E.g America gtur cultuc and easy
accessibility to guns. Japan, Singapore, Korea unhealthy pressure on
young people to stay ahead of the pack which manifests itsclf in lu d
ways).
Good essays will evaluate and explore the underlying causes behind rccent
tends of violence in schools causes by looking at changes in the l'amily'
society and school. lor example they should look at the underlying
reasons behind thc increase in bullying incidents in schools today
Weak Essays will merely list examples of violencc that maybe narow,
isolated and anecdotal.

10)

Will

newspapers become obsolele in the near future?

Students should evaluate

if the newspapers will remain viable and suNivc

in

coming Yeals.
Studenls should show an unde$tanding that thc newspaper as an industry may
not be economically viablc due to the ioss of advertising rcvenue as they are
curcntly tlueatened by newer forms ol mass media like the internet ncws
websites, sms news updates, l-tc and an emerging younger population thal
want their inlomation fast, accessible and in bite sizes.
Weak scripts may list the pros and cons of newspapers without linking i1 to
whether newspapers would be phased out in the coming years'

ll)"Singaporc has not done enough to foster

entrepreneurship

"

How far do you

agree with this statement?

Answers must evaluate the extent to which schemes and policies by


the govemment, govemment-linked bodies, national organizations
and corporations have effectively encouraged or curtailed the growth
of private enterprise (i.e. individuals setting up their own business
ventures and taking risks) in Singapore.

. It is necessary for students to give specific examples

.
.

of national
e g'
Singapore
initiatives to develop the entepreneu al spirit in
A*Star, SME funding, MOM policies, educational policies, role of
EDB. Answers must assess whether or not such initiatives have been
successful in developing entrepreneuship.
Students may evaluate the examples oflocal successful entrepreneurs such
as Oli,"ia Lum, Sim Wong Hoo. Adam Khoo' Ceorge Quek in rheir
justification.
Weak Essays will merely provide a list of local initiatives without
assessing its effectiveness in fostering entrepreneurship'
influence ofpop music on society today
A requirement would be the evaluation of the exlent and nature of the
effects ofpop music on modem society by considering its impact in any of
the following rclevart aleas: social, moral, psychological, political'

12) Assess the

.
.
o

economic and cultural


Studnts should justify the social, political effects of pop music by
citing specific eiampies of the impact of pop starc such as Michael
Jackson, Elton John, Bon Jovi, Stephanie Sun'
Better answers should have a wide range of examples and cover different
countries and cultues in relation to effects on the different segments of
society such as children and youth.
Weak answers will merely list the positive and negative effecfs of pop
music without evaluating the degree ofinfluence'

'

/(

l.

CJC Mid-Yeqr 07 Poper 2


Lewis Thomas writes...
Everyone must have had at least one personal experience with a computer error by this time
Bank balances are suddenly reported to have jumped from $379 into the millions, appeals for
charitable contributions are mailed over and over to people with crazy-sounding names at
your address, department storcs send the wrong bills, utility companies write that they're
tuming everything off, that sort of thing. lf you manage to get in touch with someone and
complain, you then get instantaneously typed, guilty letters from the same computer, saying,
'Our computer was in error, and an adjustment is being made in your account.'

These are supposed to be the sheerest, blindest accidents. l\,4istakes are not believed to be
part of the normal behaviour of a good machine. lf things go wrong, it must be a personal,
human error, the result of fingering, tampering, a button getting stuck, someone hitting the
wrong key. The computer, at its normal best, is infallible.

'10

I wonder wheiher this can be true. After all, the whole point of computers is that they

represent an extension of the human brain, vastly improved upon but nonetheless human,
superhuman maybe A good computer can think clearly and quickly enough to beat you at
chess, and some of them have even been programmed to write obscure verse They can do
anything we can do, and more besides.

15

It is not yet known whether a computer has ils own consciousness, and it would be hard to
find out about this. When you walk into a computer laboratory and stand listening, it is easy to
imagine that the faint, distant noises are the sound oi thinking But real thinking, and
dreaming, are other matters.

20

On the other hand, the evidences of something like an oncorscious, equivalent to ours, are all
around, in every mail. As extensions of the human brain, they have been constructed with the
same property of error, spontaneous, uncontrolled, and rich in possibilities.

lvlistakes are at the very base of human thought, embedded there, feeding the structure like
root nodules. lf we were not provided with the knack of being wrong, we could never get 25
anything useful done. We think our way along by choosing between right and wrong
alternatives, and the wrong choices have to be made as frequently as the right ones We get
along rn life thrs way. We are built to make mistakes, coded for error'

We learn, as we say, by 'trial and error'. Why do we always say that? Why noi 'trial and
rightness' or'trial and triumph'? The old phrase puts it that way because that is' in real life, 30
thg way it is done.

A good laboratory, like a good bank or a corporation or govemment' has to run like

a
add
up
to
the
numbers
book,
and
all
by
the
is
done
flawlessly,
everything
computer. Almost
the predicted sums. The days go by. And then, if it is a lucky day, and a lucky laboratory,
somebody makes a mistake; the wrong buffer, something in one of the blanks, a decimal
misplaced in reading counts, the warm room off by a degree and a half' a mouse out of his
box, or just a misreading of the day's protocol. Whatever, when the results come in,
something is obviously screwed up, and then the action can begin.

The misreading is not the important error; it opens the way. The next step is the crucial one lf

/7

35

at thall',then the new finding' 40


the investioator can bring himsell to say, 'But even so, look
i";"Jt io"r snatctring what is needed' for prosress to be made' is the move
;;il;i;,
based on the error.
10

or new varieties of music'


Whenever new kinds of thinking are about to be accomplished'
sldes debaling in the same mind
ir]!i" r'r"i u" an argument-beforehand With hto
other wrong sooner 45
ia an imiable understandrng thal one is.nghl and the
;i"r.a-nn;;;s,
not the two sides and
lifrt""|. tiJir'ing is settled, but there can be n; action at all if there are
towatd error The
tendency
the
;;.-;;;;""1. ir'e hope is in the facultv oi wrongness
side represents
wrong
of info';ation to l;nd lightlv on the
r""p

i"
in;;

i"o*,i,ii"

"",""i?ounrtin"
the highest of human endowments

11

12

our genetic 50
human grft p--"'hap" eu"n
"jl?'31:9-li
as a
mistakes
making
for
instru&ions Other creatures do not seem to hlve DNA sequences
guide
action
for
iJin" p"rt oic"irv ri"ing, certainly not for programmed error as a

It may be that this is a uniqueiy

therc are more choices than two


We are at our human finest, dancing with our minds, when
but.one, bound to be wrong'
iorn"t,r*"tn"* are ten, even twe;ty different ways tolidgo,usallonto
totally new ground This 55
of selection in such situations can
fa ibility. tf we had only a single center
,ri"""" i" i"i[J "-ptoiaiion anc is based on human
rnstead of
of responding onll, wl"en a correct decision was to be made
provlde
lor being
ir," i,,-nf" ot aiterent. credulous, easrly conned clusters of neuro'les thal
rp ir"", down dead ends. out rnto blue sky. a'ong wrong turnings.
irr"iiriii"t"' oi"J
60
"] "v".
tn" *ul, *e are todav stuck

;;i i;; ;il;"t"


;;;;r;;;;;;;t

rast

#,Hffi; ;; ililonl lw

13

14

are limited' most of them to


The lower animals do not have this splendid freedom They
never seen a
Cats, for all their good side never make,mrstakes ltrave
"il"oiui"-.f"ffiOiriiy
dogs are sometimes fallrble' occasron-ally,able to make
r"jli-t, "r""i"1ll "i blundering catget
their masiers Fish are
;;"ii;itt"kes, out iney ihis way bv trying to mrmic
perfect in 65
"n"i.i"g'
tney oo tnoiv.iuai cells in a tissue-are mindless machines'
n"rr""""i"
""",vtni"g
as absoluteiy inhuman as bees
their performance,
complex computers for the
We should have this in mind as we become dependent on more
let them go thelr way lf we
arfanoement of oul affalrs Cive the comouters iheir heads l say:
proceeds' the
;;:, ,ulning ;ui h""a" td one side and wincins while the.work
good 70
.nsqih,lrlies for the fulure of manklnd' ano computerkind are limitless Your average
;;r."r"iion" in ;nt,"nt which would take a liretime of slide ru'es for anv
"n
gain from the near infinity of precise' mechine-made
J- *n'rt
"""r0
would begin the solving of some
rn,slomoutat,on wnicn is now so easrly withrn our grasp. we
for
r*-iti"t-"e should we so about orsanizins ourselves
plain fact of life a single 75
fNins on a planetary scale, now that we have beco-me' as a
do;ng thrs are
"*i"f
ussume. as a working hypothesis. that allthe right ways of
""-i.,,.n"'" w.
much
alternatrves
of
wrong
"ln
in"" tor mivrng ahead is a sel
w-h"i
up
think
can
of
us
""-"""4,than the short list;t mistaken courses that any
il'^o. :nrt more interestinq
to
computer
when rt is printed out we need the
in iuli in inrin,t" r,"t
w;
";;;,
tn" "nd to go lf it is a brs enoush mistake' we could 80
"6*r "av
fhd ourselves on a new level' stunned, out in the clear' ready to move agaln

il;i5;;i;'a;
::;"";J;;.;;
-""
; "::t;;i
;i'":'i"";;"i ;;;i;ti iiot
;ilr[#L
;;;i;;

ffi'#ii;i;il;il,lli""oot,

Adapted from To

Et is

Human by Lewis Thomas

/&

From paragraphs 1 and 2, why would one be surpdsed at the examples of computer
error? Use your own words as far as
[2m]

2.

From paragraphs 3 and 4, how are computers and humans different in the way they
'think'? Use your own words as far as possible. [4m]

3.

From paragraph 7, Why do we always say'trial and error'and not'trial and rightness'?
Use your own words as far as possible. [2m]

4.

Explain the paradox (apparent contradiction) in the sentence, 'What is needed, for
progress to be made, is the move based on the error.' (lines 41-42) [2m]

5.

Explain the meaning ofthe following phrases in your own words as far as possible.

a)
b)

possible.

'We are built to make mistakes, coded for errof (line 28) ll ml
'give the computers their heads' (line
[1m]

68)

6.

Why is making mistakes beneficial to society? Summarise in no more than 130 words,
using material from paragraphs 6 to12. Use your own words as far as possible. [6m]

7.

From paragraphs 12 and 13, why does the writer draw attention to the 'absolute
infallibility' of 'lower animals' (line 6'1-62)? Use your own words as far as possible. [2m]

8.

From paragraph 14, what is needed for mankind to move ahead (line 77)? Use your own
words as far as possible.[3m]

w rc
vr u rv ruluwl r9 wvr us ds rEy drv ulcu nr urc PdJJdgE r vu
'ldy
your answer in one word or a short phrase.

e. ervc

re

'

Icd|r

19

[5m]

(a)
(b)
(c)
(d)
(e)

blindest (line 8)
embedded (line 24)
tendency (line 47)
endowments (line 49)
stipulated(line50)

10. The writer argues in favour of the benefits of error-making. To what extent should your

society be more tolerant of people making mistakes? Justify your answer with reference to
the ideas in the text and to your own ideas and experience. iTml

tq

CJG Mid-Year 07 Paper 2 Answer Scheme

'L From paragraphs I and 2, why, according to the writer, would one be surprised
at the examples of computer eiroi? UsG ygua own words as tai as possible. [21

Lifted
lvlistakes are not believed to be part of the
normal behaviour of a qood machine.
If things go wrong, it must be a personal,
human error.

Possible reDhrase
Computers are not expected/ lt is typical of
comDuters to make anv blunders / slips. I1l
lf errors were made, it would be assumed
that it is a result of human incompetence,/
miscalculation/ oversioht. Ill
Bonus mark
The computer errors described are of an
extreme nature. / Computers should be
efficient but enors are almost absurd,
bizarre in nature. Ill

2. From paragraphs 3 and 4, how are humans and computers different in the ways

they'think'? Use your own words as far as possible.

A good computer can think clearly and


quickly enough to beat you at chess,

t41

Possible reohlase
a) Computers can think more logically /
supercede humans while still maintaining
the essence oi human thought. ['l]
b) Computers can also work faster
process ideas faster. [1]

Computers have improved functions. 0m

It is not yet known whether a computer has


its own consciousness, and it would be
hard to find out...
(it is easy to imagine that the faint distant
noises are the sound ofthinking.)

But realthinking and dreaming are other


matters

NB: Answers for a) and b) must be


pniasec! in inc COmF,aiai;ve ioiin, ic [' s:
logically, fastg!, etc in order to get the full
mark. Answers without the comparative
(eg computers are logical and think fast)
0m.
Computers are programmed to respond in
a mechanical manner and lack awareness.

t1l
Human beings on the other hand have
genuine cognitive and imaginative
capacities. ['1]

why do we, according to the writer in paragraph 7, always say'trial and erro/
and not'trial and rightness'? Use your own words as tar as
t2l
3.

possible.

J6

Possible reohrase
Lifted
it
ls We use that expression as ii captures most
life,
the
way
in
real
that
is,
-.because
accurately what we experience when we
done.
try to accomplish or achieve something. [1]

Unacceptable lift: real life


Reality [1/2m]
Part of life Ilml
We don't often get it correcu succeed on
our first attempt. OR We become more
proficienU competenV skilled through
experimentation and unsuccessful
attempts. I1l

4. Explain the paradox (apparent contradiction) in the sentence' '\lvhat is needed;


121
for progress to be made, is the move based on the error'' (lines

4142)

Lifted
for progress to be made

the move based on the error

Possible rephrase
It would be expected that to
advance/improve, no mistakes would have
been made / or one would only move
forward. not back. l1l
lnstead what is implied here is that
advancements are made only after a fault
or blunder has been committed. [1]

5, Explain in your own words as far as possible what the

writer means by the

phrases:
a) "We are built to make mistakes, coded for

erlor" (line

28)

tl]

programmed to
Human beings are engineered/ created and predisposed/ biologically

commit blunders.
Not perfect / imperfect by nature [0m]
b) give the computers their heads (line

68)

ttl

-Computers should be given autonomy/ allowed independent functioning/ operations

I,

Vocabulary:

Blrndest (line 8)

t51

1m
-Purely by chance/
luck.

-Entirely
unintentional

Embedded (line 24)

Deeply entrenched

0m

0.5m

-Totally lacking in
perception /
judgement /
reason/foresight
- unexpected /
unforeseen /
unpredictable

accidental

hidden

rooted
2

Tendency (line 47)

lnclination
Propensity
Naturalleaning
SubconsciouslY
favouf

Endowments (line
49)

cifts

Stipulated (line 50)

Dictated/ sPecified/
laid down/ set down
as an essential part
Set down explicitly

Natural capacities
Natural abilities

in no more than

S'ummarise
6. Why can making mistakes be beneficial to society?
your own words as far as
Use
6
to
paragraphs
130 words, using materialfrom

l2

t6l

possible.
l\rlGtirkes are at the very base of human

ihought... (line 24)

Possible rePhrase
errors are the foundation of human
cognitive activity/ critical reflection [1]

were not proviOed with the knack


of being wrong, we could never get
anything useful done. (lines 25-26)

We could nol achieve anything or


engage in any productive activity. [1]

We tearn, as we saY, bY'trial and error'


. in real life, the way it is done. (lines
29-31)

We neconre wiserl more informed/ make


better judgements by making errors ['1]

Almost everything is done


flawlessly. something screwed up, and
then the action can begin (lines 33-38)

tvtGtaGiire

Tfre miweading is not the important


the new
error; it opens the
(lines
39-40)
finding...
OR
The capacity to leap actoss mountains
of rnformatron to land ....highest of
human endowments (lines 49-50)

istakes allow us to break new


conceptual ground/ have new
perspectives/ dlscovenes / lnnovation
and lll

f,at is neeoed, for Progress to be


made, ;s the move based on the error'
(lines 41-42)

io

rt we

way

ihere has to be an arqument

unacceDtable lift: learn


a

catatvsv impetus for

chanqe, lest we become too complacenv


mech-anical/ seemingly perfect [1]

mike advancements based on these

new perspectives/ discoveries. ['1]


Unacceptable lift: progress' error
one Dersoective against the

l1

beforehand ({ine 44)... there can be no


action at all if there are not the two
sides, and the argument.. (lines 45-46)
8.

9.

other, it leads paradoxically to a creative


tension. [1]

the richness of selection in such


Mistakes give us diverse and manifold
situations can lift us onto totally new
options ['l]
ground. (lines 55)
lf we had only a single center in our without which we become stagnanv
brains, capable of responding only intellectually static.[1]
when a correct decision was to be
made, instead of the jumble of different,
credulous, easily conned clusters of
neurones that provide for being flung off
into blind alleys, up trees, down dead
ends, out inio blue sky, along wrong
turnings, around bends, we could only
stay the way we are today, stuck fast.
(lines 57-60)

7. From paragraphs 12 and 13, why does the writer draw attention to the ,absolute
infallibility' of 'lower animals' in line 62? Use your own words as far as possible.
t21

Lifted
lndividual cells in a tissue are mindless
machines, perfect jn their performance, as
absolutely inhuman as bees
Sometimes there are ten, even tlventy
difFerent ways to go, all but one bound to
be wrong, and the richness of selection i;
such situations can lift us onto totally new
ground. This process is called exploration
and is based on human fallibiliiv

Possible rephrase
-Animals do not make errors because
-their actions are biologically/ genetica y
predetermined / diciated bv instinct. I1l
-Human beings on the other hand have the
capacity for reflective choice which could
resLrlt in making errors.[1]
Acceptable lift: lower animals

8. F.om paragraph 14, according to the writer, what is needed for mankind

move ahead (line 77)? Use your own words as far as

Lifted
A set of wrong aiternatives much longer
and more intercsting than the short list of
mistaken courses that any of us can think
up right now

possible.

to
t31

Possible reohrase

lvlankind would need to accept the


unlimited array of faulty [1] but stimulating/
engaging options {llthat the computer can
generate and pay less attention to the
restricted number of errors that humans
can anticipate.
lf it is a big enough mistake, we could find
lf the relevant computer error is
ourselves on a new level, stunned, out in
substantial/ massive enough, [1/2mj it
the clear, ready to move again.
would elevate us to the next paradigm/
plane/ intellectual horizon from which
mankind can progress. I1/2ml
Unacceptable lift: level
10. The writer argues in favour of the benefits of error-rnakingr To what extent
should your society be more tolerant of people making mistakes? Justify youl

answer with reference to the ideas in the text and to your own ideas and
exPerience. [4
More tolerant:
'pts in summary are relevant but must be contextualized.
*pt of discrim bet good and bad students diflerentiate bet mere blunders and
productive ertors.
*compadson with other more tolerant societies is in order.
Singapore has not tapped the potential of making productive errors - essentially
intolerant of genuine errors which we either condemn or draw into mainstream culture.
-Would encourage entrepreneurship and risk-taking.
-Creativity vs conformity (following a template)
-Gracious and accepting society - allowing people who may not succeed initially to
blossom.
-stress levels would be reduced.

Egs
Education - unforgiving and rigid.
Employers - intolerant of efforc - employees become risk-averse and timid. Conversely,
people who commit less errors become arrogant and complacent.
Politics - repressive and uninviting.
- give us a right understanding oi being human

both limitations and potential

Lers talqrq!!
-Security and order - there is little margin for error in these times (terroism, crime,
political choices etc) - consequencesl
-Excellence and efficiency are required in a city that wishes to go global.
-Mistakes could be simply due to bad judgment / negligence - for eg, risktaking vs
making wild decisions.

J+

lJc Mid-Yeor 07 Pqper I

'!.

To what extent does the Singapore education system meet the needs of society?

2.

Examine the validity of the statement that poverty is the parent of crime (Aristotle)

3.

"Technological progress has caused society to regress in other ways " Discuss'

4.

Consider the implications of dishonesty

5."Theworkingmotherphenomenoninmodernsocietyistherootcauseofanti-social
behaviour among young people." Do you agree?

6.

"Sport does not build character. lt reveals character'" How far do you agree with this
statement?

7. Would you consider Singapore successful in instilling in her people a sense of


belonging?

8.

Should animaltesting be banned?

g.

"The weak believe in luck. The strong believe in cause and effect " Discuss

'10.

Can the mass media be blamed for all the problems of the modern world?

1.

'12.

Does religion still have a place in our scientiflc world today?


"We continue to only harm the environment " ls this a fair assessment?

IJC Mid-Year 07 Paper

1. To

what extent does the Singapore education system meet the needs of society?

D-E Essav

.
.
.
.

Will detine the term "reeds of socie,y". For e9, Singapore needs to nurture good leaders,
forward thinkers and entrepreneurs and she needs to establish an open and inclusive
society and a civil society.
Will define "education system" as formal education system and what it comprises (eg
cufficulum, school structure, teachers, assessment etc).
lvlay be ote"sided discussion highlighting that Singapore education system does meet or
does not meet the needs of society.
Will have limited exarnpres to suppod one's view.

c OR BETTER Essav

.
.
.

Will discuss how the Singapore education system is ablp to meet the needs of society but
also give a baranced discussio, on how the Singapore education system fails to meet the
needs of society. For example, Singapore's education system seems to adopt a more
didactic approach to National Education which does not encourage the young to speak
their mind and actively participate in forging a civil society.
Will give varied and cunent examples to support the views. For eg, the Social-Emotional
Learning Framework guides the establishment of school programmes for the holistic
development of the child.
Will hiqhlight current trends. For e9, attempts to diversify the education landscape to
address and recognise the varying abilities of differcnt members of society and establish
"peaks of excellence".
Will evatuate whether all needs of society can be met by just an education system (in
relation to Singapore education system) or are there other institutions (eg government and
charity organisations) and processes that need to work hand in hand with the education
system. For example, the family, just as much as the education system, plays a signiflcant
part in the upbringing ofthe young to ensure that we have a clvil society

2. Examine the validity of the statement that poverty is the parent of c.ime (Aristotle).
D-E Essav

.
.
.
.

Ctatily whal "pouerty is the parent ot crime" means


Recognise that the question presents an efretne view, that is to say that povedy is the
root cause for crime and, hence, questions the accuracy of such a statemeni
Discusses whether poverty is the cause ol crime, without analysing whether poverty is the
root cause (parent) of crime.

Superticiat examples will be given, mainly assuming and glossing over the causal
relationship betvveen poverty and crime, without addressing the complex factors at wo*
For eg, a student might explain that a poor person will be forced to steal to feed himself,
without acknowledging that many developed societies have put in a welfare system that
provides a safety net for ihe impoverished and underprivileged.

C OR BETTER E59av

J6

Will understand and illustrate through examples lhal povedy can resull in different
types of crime, ranging from those which are in response to impoverished conditions such
as theft, prostitution and dealjng in drugs and those which are the indirect result of such
condilions such as family and gang violence resulting from anger and frustration at being
deprived or marginalised.
eyaruale whether poverty is the root cause (parent) of crime. There will be a
comparison with other reasons for crimes such as greed, revenge, desire for power, in the
name of religion and even for thrills, highlighting examples wfen crimes have taken place
due to other reasons apatT from poverty, such as the NKF Saga corruption (due to
greed).
Will have a good mix ol global examptes, comparing developed and underdeveloped
countries, to illustrate the arguments crafted. For eg, student might highlight that rn poorer
countries like Laos, Vietnam and Thailand, there is a high rate of child and female
prostitution due to poverty. At the same time, in developed countries like Japan, some
teenage girls prostitute themselves not because of pove.ty, but so as to support their
extravagant lfestyles
Witl pay attention to the keyword "validity" and "parent" and analyse whether lhe
stafemena is a realistic description of present society at large orjusl a generalisation.

Wll

3. "Technological progress has caused society

to regress in other ways." Discuss.

D-E Essav

.
.
.
.
.

Will clariry the term "regress" and "technological progress'.


Will recognise that "regress" presents a negative view.
Will rest/ict the discussion to only effects of technological progress on society and not
examrne whether the eflects have led to regression or not
W;ll provide a one.sided point of viewwhere studenl willexamine onty how technological
progress has caused society to regress.
Will give ,imiled and out-dated examples.

C OR BETTER Essav

Will recogrise that the question already assumes that there is progress in society due
to tec h nol ogical p ro g ress
Will explain and evaluale trow technological developments may lead to deterioration or
progress and examine the effects of "technological progress" on society from diferer(
perspectives (eg sgcial, political, economic, environmental, medical) For eg,
communications technology like the lnternet expands markets for businesses but, because
there is no central control of the lnternet and limited means of censoring its materials, it is
often exploited by paedophiles who post more than 20000 images of child pornography
online daily.
Will provide a Mde range of examples lo supporl one's views.
Will provide a balanced discussio, on how technological progress has led to progress
and regression.

4. Consider the implications of dishonesty.


D-E Essav

Will define "dishonesty".

el

.
.

Will describe the implications of dishonesiy, for eg, how it can destroy ones' relationship,
working life and personal life.
Will provide limited examples, that is, restricting the discussion to a personal perspective
or providing only examples from Singapore.

C OR BETTER Essav

.
.

.
.

Will disc{rss the implicalions of dishonesty on a micro (individual) and macrc (global)
perspective. On a micro level, the student would discuss dishonesty, for e.9., in tems of
slealing, in relationships, in workjng life and consider the implications ofthis dishonesty. On
a macro level, the student would discuss dishonesty, for eg, in foreign relations, global
politics, global business, international sports arena and the implications of it.
Will evaluate the ex,ent (scale and duration) ofthe repercussions of dishonesty.
Will ,llusfrate through current examples the implications of dishonesty, for eg, NKF T.T.
Durai's dishonesty has ruined his career, bankrupted him and caused him to be a pariah in
society and, for eg, the doping scandal involving French cyclist Floyd Landis who tested
positive for synthetic testosterone and recent admissions by Lance Armstrong's former
team-mates that they had taken the used the banned endurance-boosting drug EPO in
preparation for the 1999lour have undermined the credibility of the sport.
Will explain the impodance of honesty
Will be able to point out that sometimes the implications of dishonesty may be posftiye.

5. "The working mother phenomenon in modern society is the root cause of anti-social
behaviour among young people." Do you agree?
D-E Essav

.
.
.
.
.
.

Will exprai, the phrase "working mother phenomenon".


Will discuss types of anti-social behaviour ranging frorn the deviant to the criminal (For eg,
promiscujty, prostitution, school bullying, vandalism, being inebriated).
Will have a few outdated examples, ignoring the time frame of the question, i.e. "modern
society".
Will recognise that the question presents ar extrcme view since the question reads as
"working mother.....root cause of anti-social behaviour... "
Will discuss how the working mother phenomenon adversely affects the young, wilhout
anatysing whether it is the root cause of anli-social behaviour among the young.
Will p/ovide limited examples of anti-sociai behaviour among young people.

C OR BETTER Essav

.
.
.
.

Will pay atlention to the keyword "root cause" and analyse wfefrrer the sta,emert is an

ovef-genefalisation.
Will evaluale and have a good variety of rccent examples evaluating whether working
mother phenomenon is the root cause of antisoaial behaviour among young people.
Will offer a balanced discussio, by comparing olher causes of anti-social behaviour
among the young with the working mother phenomenon ta delemine the root cause or
whether it is a combination of several faclors. For eg, peer pressure, stress trom school
and society and exposure to the other sources of influence through the mass mediaWill recognise that there are alternaive caregivers who may be just as effective in
fulfilling the roles and responsibilities of the mother such that the blame cannot be pinned
on the absence of lhe mother.

J'

May attempt to highlight that though there are other causes for anti-social behaviour among
the young, the working molher phenomenon mighl have aggtavated the situation. Fot eg,
the mother's absence can result in a child looking for guidance and company elsewhere.

6. "Sport does not build character. lt reveals charactel." How far do you agree with this
statement?
D-E Essav

.
.
.
.
.

Will attempf to defrne the key phrase "charactel' in terms of qualities that one has such as
resilience, determination and diligence.
Will restricl the discussion and examples Io behaviour and, hence, taits displayed
during the course of engaging in the spor7.
Will explain the causal relationship between sport and the development and revelation of
character.
Will fimit discussion to sport not building character but rather revealing character. ln other
words, the student would not be discussing the other perspectives that sports builds as well
as reveals character or that sport builds character but does not reveal character.
Will have a limifed range of examples from the same spoti (For eg soccer).

C OR BETTER Essav

.
.
.
.
.

Will point out that "character" in the first sense refers to poiitive traits while "character"
in the second sense may refer to both positive and negative araits.
Will attempl to give a balanced discussion, illustrating how sport also builds character.
WiIt discrss how it is both a nature and nurture issue.
Wlll ustate argurrents using a good variety of rccent examples in the international
spofting afena.
l,4ay highlight a lotally different perspective that sport, in some cases, does nol
necessariiy build nor reveal character such as in entertainrnent sporis, as in ihe case of the
popular wrestling show, World Wrestling Entertainment, where some of the action is staged
and wrestlers have to take on the role of a hero or villain at difletent times.

7. Would you consider Singapore successful in instilling in her people a sense ot


belonging?
D-E Essav
Will laryely describe what Singapore has done to instil in her people a sense of belonging.
Will have miaimat drscussion on the keyrvord Sr.rccessful"
Will give limiled and mundane examples such as singing the national anthem and saying
the pledge in schools, or giossing over National Education in schools.
Will have a narrcw scope of discussion, focussing mainly on schools, paying little or no
a{tention to other groups

.
.
.
.

C or BETTER Essav

.
.

means to have a sense of belonging, especially to a small


cosmopolitan city like Singapore for instance, (a) Feel passionately about some aspect of
our country (b) Get together, do something for the community lNational Day Rally Speech

Will discuss what

it

20061

Will evaluate the extenf ofsuccess in instilling in her people a sense of belonging
1

A wide range ol recerl examples to subslantiate their arguments is necessary for them
to do weli in this quesfion. (For eg, the overseas Singaporean Unit established in 2006 to
reach out to Singaporeans livjng abroad, the National Youth Forum in 2004 and Youth
Workgroups set up by the Ministry of Community Development, Youth and Sports to
involve young people in issues relating to community involvement, national engagement,
opportunities for all and youth entrepreneulship)

Will go beyond discussing the successes/ llaws of the National Education programme in
schools and look at other ways in which the government instils a sense of belonging to
Singapore: recogrr:se that heattware is not just something we do l', sctools; it is also
about getting all Singaporeans to engage and padicipate in shaping the character and life
of our society.

8. Should animal

testing be banned?

D-E Essav

.
.
.

Will take an ernotiora, stand, denouncing animal testing as cruel and totally unnecessary,
condemn supporters of animaltesling - rack barance
Will have rimited discussior onwhy animaltesting should be banned - reasons may not
be always sound or welhargued.
Will only put forth reasons against animal testing bul wrl not be able to suggest
atternalives and just insist plainly on banning animal testing.

C or BETTER Essav

.
.
.
.
.
.

Will be objective in their argument, putting forth both the benefils and drawbacks ol
anrmaltesting.
Will recognise that the word "should" reflects lhat a moral dimension ought to be
addressed by evaluating the morality of lhe motive, the ac{ itself and the consequences
that arise.
Will d,btr'guish lhe different purposes of testirg (for eg, medicine, cosmetics) and
Will also be able to give a wide range ol examples of types of animal tesfing (for egi
LD50, Draize eye test, skin irritancy test) and evaluafe whether these forms of testing are
absolutely necessary.
Will discuss some araernatives to animal lesarrg (for eg, in-vitro te6ts, computer software,
dalabases of tests already done to avoid duplicalion)and cite some examples of some
companies which have stafted to adopt these alternative measures (for eg, Body Shop,
L'Oreal).
May divide the animals lnto groups - they may perceive that it is more inhumane to conduct
tests some animals like chimpanzees and dogs compared to rodents.

9. "The weak believe in luck. The strong believe in cause and eflect." Discuss.
D-E Essav

.
.
.

Will attempt to explore the truth of the statement though will not always be able to pin
down what they consider "weak" and "strong".
Ihe key phrase "cause and effect" may not be properly understood or explained.
Will attempt to bring up a couple of exceptions for balance.

jD

c or BETTER Essav

linked
.- w,ri-r
oe ,ol" to po int out what the key phrases mean and exptain how they are
;iiirg'l
and
self-belief
of
independence'
idea
ind "cause and etfeci" rcfet lo lhe
i.".
'o"ing;

of one's own destiny by imposing a rational approach to ?nalysing and


one's circumstances, while "weak" and "luck" rcfer to the idea of helplessness'
maniging";ntrof
uncerLiniy and allowing oneself to be subjected to the vagaries of life'
of
Will exptain why suci a claim is made in lhe statement by giving a wide range
;strong"
moves
strategic
or
wellplanned
who have put in efforl and made
exampies-the
will hardly afttibute their success to pure luck (For eg'
if
success
form
sot"
to attlin
a
winning an election, economic success, having meaningful relationships and leading
ofthose
iurririini tit")-uno oy aiscussing the lrane otmind(ie whether they are weak)
Deal"
or
No
game
shows.like."Deal
participate
in
;# ffit ;:D, Totoind Big Sw-eep and
there
whether
evaluating
present)
and
which aie based on luck with no cause and effect
ii i "",r""t rctationship between one's frame of mind and one's attitude towards
one's ability to control one's circumslances.
Wtiquestio, tne a.sumptions reflected in the slatement lor the countetargumentby
nigf'lijt'ii"g occasions where the slrorg seize the oPpolultilie: aflorded bv
succeed in
iJniaipiiou" incidents and determine causal factors that would help them
he
deduced a
penicillin
when
of
(For eg, Alexander Fleming's discovery
in"ii
petri dish
a
"nl""uort"
relationship between the rnould that accidentally contaminated
lausal
ionLining , St+nylo"iccus bacteria culture and the subsequent elimination of the bacteria )
ir," .rtirni , emselves demonstate a belief in luck (For eg, highly skilled and
oi

"n"n soccer p-layers who believe in wearing their lucky jersey number')
successful

10. Can the mass media be blamed for all the problems of the modern world?
D-E Essav
Witt attempt to

.
.
.
.
.

tist some of the problems of the modem world


presents
Will pay attention lo the keyword "atl" and recognise that the question

an

extreme view .
media
Will address the qu eslion tty comparing a variety of factots with the mass
question
i'e "modern
of
the
frame
[,4ay nave a few outdated'examples, ignoring the time
wo.ld"
superticial exampres will be given, mainlf glossing over the disadvantages the mass
media bring about without rnuch analysis

C or BETTER Essav

.
.
.

giue- Oro"a sp eclrum of Problems llte modem world faces


tyPes of mass
Will iave a good va riety of recenl examples that refer to lhe different

Witt

media.

Will offer
Oi""rii"O

r"Oi"

.
.

blamed.""n

problems
a balanced discussio, by comparing other causes of these
or
if
the
mass
t" evaluate whether it is a combination of sevenl factors
can or should be

ttrfv be blamed for the problems and to what extenl they

the
be able to provide balance by having examples of problems not caused by
Witt
problems
media and show how the mass media help alleviate these
mass"lro
case of whether or
Will be able to rec ognise that sometimes lhere may not be a cleal

(for eg' while we are now more


not the mass mediishould be blamed for the problems
face
of terrorist threats' the terrorists
aware about precautionary measures to take in the
perpetuate
fear')
have also indiiectly made use ofthe mass media to

3l

May recogrise lh at the mass media may be a convenient scapegoaffor societies to pin
all ihe problems on so that the source of the problems can be kept concealed or they can
absolve themselves from blame.

'11. Does

religion still have a place in our scientific world today?

D-E Essav

.
.

\ /ill be able to recognise that the science and religion may potentially be conflicting
because of their fundamental assumptions about otigins and reality and the values
that they subscribe to.
Will adopt lhe narrow view that dichotomises science and religion' that is' the scientific
persp""iive is based on empirical evidence and is' hence, objective, whereas religion is

is thus more subjective, without acknowledging

that
data
are
where
word
scientific
subjectivism/relativism is a typicat leature of the

Lased on faith and

.
.

always subject to manipulation and interpretation


Will confine argument lo mainly a discussion about terron3m
Wilt give rinited exatnpres (For eg, focus mainly on 911 bombings' London bombing on 7
July, draw a simplistic conclusion about the importance of religion from the fact that places
of worship still exist without being able to elaborate the example )

C or BETTER Essav
will recogni"u that the statement assumes thai science has come to dominate the wcid
and there is an imptied assumplion lhal rcligion does not have a place in our world today
because scierce ard religion arc mutually exclusive.
Will be able to point out that religion is able to meet certain needs in our lives that
science may not be abte to (for eg, psychological function; dealinq with death, suffering,
fear, anxiety; makes lhe world comprehensiblel assigning meaning to events and providing
a framework for interpreting events that seem randomly disconnected; provides meaning
and purpose in life; trans;ending the mundane to look beyond everyday events and
attempting to understand them in a metaphysical framework, ior eg, interpreting life cycle
events such as birth, adulthood, marriage, death; helps individuals adjust to changes in
iheir livas; prcvides c framework for the behavioural norms, that is, how one should act and
behave both as an individual and as part of society )
Will be able to.ecognisethat retigion sfirl is a sersdrve issue/ held in high rcgard in
our world today. Willdefinitely be ab,e to point out that religion is a sanctuary and a source
of identity for people living in a world that is undergoing rapid change and, in situations
when that sanctuary and identity are challenged, it leads to conflict that, in extrerne cases'
manifesis itself in acts of terrorism.
Will give a wide range of globat examples lo Provide balance (fot eg, ihe influence
religi6n has over legll systems (e.g. Sharia Law) in l\4uslim counlries, the increasing
nuriber of people embracing Buddhist philosophy as a means of coping with the slress of
modern living and to counler materialistic values, the growth of evangelical Christianity as
people seek-meaning in life and the atternpts to marry science and religion through the
iheory of lntelligent besign. All these have taken place despite the advancements in

.
.

science).

12. "We continue to only halm the environmenl'" ls this a fair assessment?
D-E Essav
Will be able to point out that "otly" is an exteme word.
Will clarry and explain in some detail lhe ham we have done io the environment as
well as what we are stit doing to ham the environment ("continue")

.
.

.
.

Will provide balance by pointing out what we have done to protect lhe environment
andlor lhe efforts to minimise the harm
Will provide outdated or a limited range of examples lo illustrate the argument crafted
(For eg, may give mundane examples such as the 3Rs Reduce, Recycle and Reuse,
saving eleckicity/ water at the individual level or discuss deforestation and other forms of
pollution without being able to give details.)

C or BETTER Essav

.
.

in great detail using recent


and
the consequences which we
to
the
envjronment
harm
we
have
done
exarnples of the
(Refer
in The Straits Times dated
to
examples
are already facing and will face in the future.
27 Juoe -29 June 2007.)
Will have a good mix ol recent global examples (For eg, global warming and its impact
on the local climate e.g extreme weather patterns such as flooding in Australia and heat
waves in lndia, destruction of habitats due to pollution or the logging and farming industries
which lead to dwindling animal populations and marine life.)
Will atso be able to list some ol lhe rccenl efforfs (For eg, Live Earth concert) to save the
Eafth and evaluate the success ofthese measures vis-d-vis the harm done.)
Will be able lo explain and illuslf,al9 their argument

:3

UC Mld-Yeor 07 Pqper 2

Secular Humanism is Hamtul

John Gray writes.....

Of all the myths spawned by the Enlightenmen{, the idea that we live in a secular age is

the most absurd. Throughout much of the wodd, religion is thriving with undiminished

vitality. Where believers are in the minority, as they are in Britain today, traditional faiths
have been replaced by iiberal humanism2, which is now established as the unthinking
creed of conventional people. Yet liberal humanism is itself very obviously a religion-a 5
shoddy derivative of Christian faith. lf this is not recognised, it is because religion has
been repressed from consciousness in the way that sexuality was repressed in Victorian
tames. Now as then, the result is not that the need disappears, but rather that it returns in
bizaffe and perverse forms. Secular societies may imagine tlFy are post-religious' but
10
actually they are ruled by repressed

religion.

Liberal humanism inherits several key Christian beliefs-above all, the belief that
humans are categorically different from all other animals. According to humanists'
humans are unique in that, using the power over nature given them by science' they can
create a world better than any that has existed before. ln this view, the earth as simply a
mass of resources for human use, and the other animals with which we share it have no 15
value in themselves. Those who hold to this view of things see themselves as toughminded scientific realists, but in fact they are in the grip of one of the worst legacies of
Christianity. The humanist view of the earth as an instrument of human purpose as a
securar rendnron of the biblical myth of Genesis.

The role of hollowed-out versions of Christian myth in humanist thought is particularly 20


clear in the case of lvlarxism. Marx's absurd idea of'lhe end of history", in which
communism triumphs and destructive conflict then vanishes from the world, is
transparently a secular mutation of Christian beliefs. The same is true of Francis
Fukuyama's equally preposterous belief in universal salvation through "global democratic
25
capitalism". ln both cases, what we have is myth masquerading as

science.

Ihe trouble with secular myths is that they are frequently more harmful than the

real
thing. ln traditional Chrislianity, the apocalyptic impulse was restrained by the insight that
human beings are ineradicably flawed. ln the secular religions that flowed from
Christianity, this insight was lost. The result has been a form of tyranny, new in history,
that commits vast crimes in the pursuit of heaven on

ebrth.

The Enlightenment refers to the historical intellectual movenenl

it

30

the 18d Century which advocatd rc!!94 or

rationality as the primary basis of authorit.


god.
'?Liberal or secular humanism is the philosophy tbat rejecrs th existence ofa

3+

The role of humanist thought in shaping the past century's worst regimes is easily
demonstrable, but it is passed over, or denied, by those who harp on about the crimes of
religion. Yet the mass murders of the 20th century were not perpetrated by some latterday version of the Spanish lnquisition3. They were done by atheist regimes in the service
of Enlightenment ideals of progress. Stalin and l\4ao were not believers in original sin. 35
Even Hitler, who despised Enlightenment values of equality and freedom, shared the
Enlightenment faith thal a new world could be created by human will. Each of these
tyrants imagined that the human condition could be transformed through the use of
science.

The irony of secular culiures is that ihey are ruled by myths. lt is a commonplace that
science has dlsplaced religion. What is less often noted is that science has become a
vehicle for needs that are indisputably re{igious. Like religion in the past, though less
efiectively, science offers meaning and hope. ln politics, improvement is fragmentary and
reversible. In science, the growth of knowledge is cumulative and now seemingly
unstoppable. Science gives a sensation of progress that politics cannot deliver. lt is an
iliusion, but that in no way diminishes its power. We may live in a post-Christian culture,
but the idea of providence has not disappeared. People still need to believe that a benign
pattern can be glimpsed in the chaos of human evenis.

40

45

The need for religion appears to be hard-wired in the human animal. Certainly the
behaviour of secular humanists supports this hypothesis. Atheists are usually just as 50
emotionaliy engaged as believers. Quite commonly, they are more intellectually rigid. No
doubt there are many reasons for this state of affairs, but I suspect it is the repression of
the religious irnpulse that explains the obsessive rigidity of secular thouqht.

Liberal humanisis repress religious experience-in themselves and others-in much the
way that sexuality was repressed in the straiFlaced societies of the past. ln secular
cultures, religion is buried in the unconscious, only to reappear-as sex did among the
Victorians-in grotesque and illicit forms. lf, as some claim, the Victorians covered piano
legs in a vain effort to exorcise sex from their lives, secular humanists behave similarly
when they condemn religion as irrational. lt seems not to have occurred to them to ask
where it comes from. History and anthropology show it to be a species-wide
phenomenon. There is no morc reason to think that we will cease to be religious animals

bU

than there is to think we will some day be asexual.

liberal humanist thinkers were adamant that religion would die out with the advance
That has not come aboul, and there is not the remotest prospect of it
happening in the foreseeable future. Yet the idea that religion can be eradicated from
human life remains an anxiously defended article of faith among secular humanists. As
secular ideology is dumped throughout the world, they are left disoriented and gaM/ping
l\,4any

of, science.

The Spanish Inquisitjon was a religious tribunal esiablishcd in


orthodoxy jn Spain. It was notorious for ils use oflorture.

lll?!

65

by the Spanish monarchyto maintain Calholic

3/

10 lt is this painful inability to reconcile

their expectation with the reality around them, I


believe, that accounts for the peculiar rancor and intolerance of many secular thinkers.
Unable to account for the irrepressible vitality of religioh, they can react only with
puratanical horror and stigmatise it as irrational. Yet the truth is that if religion is irrational,
so is the human animal. As is shown by the behaviour of humanists, this is never more so
than when it imagines itself to be ruled by reason.

1,1

Here we have the paradox of secularism. Secular societies believe they have left religion

behind, when all they have done is substitute one set of myths for another- ln effect,
liberal humanism has taken Christianity's unhappiest myth*the separation of humans
from the rest of the natural world-and stripped it of the transcendental content that gave
it meaning. ln so doing, it has left secular cultures such as Britain stuck between a
humanist view of mankind that actually comes from religjon and a more genuinely
scientific view in which it is just one animal species, no more capable of taking charge of
its ciesiiny ihan any other....
12

13

70

Humanism is not an alternative to religious belief, but rather a degenerate and unwitting
vercion of it. Among the many varieties of religious life that are thriving among us-Hindu
and Buddhist, Jewish and Muslim, along with many new and hybrid traditions-this pale
shadow of Christianity is surely an anomaly.
Weighed down with fears and anxieties that the rest of us have never known or have long
since left behind, it survives only as a remnant of a time when .eligion suppressed natural
human impulses. We rnay not be far from a time when atheism will be seen as a relic of
repression, like the frills that may once have been draped over piano legs.

)[

75

80

85

'1. Accordingtotheauthor,whyisitabsurdtoclaimthat'weliveinasecularage'(linel)?

2.

1l

'the unthinking creed of conventional people' (lines 4-5)

What does this reveal about the autho/s attitude towards liberal humanjsm? [2]

3.

What does the author intend you to understand by ending the second paragraph wfth three
dots (...)?[1]

4.

rom paragraph 3, describe the two mutated versions of Christianity and exPlain how they are
similar. Use your ow, words as taras possible[3]
5.

dentify the paradox in paragraph 4 and explain it.[2]


People still need to believe that a borigt pallem can be grimpsed in the chaos of human
events.' (lines 4748). Explain what the author is saying by bringing out the meaning of the
rtaLrcrsed words. [2]
xplain the authols criticism of the secular humanists rn paragraih 1A. Use your
as far as possrb/e. [2]

own worcls

8.
hat is the author implying by using the word 'animal' in the penultimate sentence of paragraPh
10?

l2l

9.

ive the meaning of the following words as they are used in the passage. You may
write your answer in one word or a short phrase.

1)
(b) bizarre (line 9)....
(a) spawned (line

................................

. .
(c) categorically (line 12) ........................
(d) legacies (line
(e) vehicle (line 42)

17)

Ill

... .. .... ................ nl


.......................................111

............................ ..... t1l

t1l

tO. Using material from paragraphs 7 to I of the passage, summalise the author's reasons fol
asserting that religion is an inherent need of human beings and his argumenl against the
repression of religion. Wriie your summary in no more than 120 words not counting the
opening wods which are printed below. Use yo ur awn wards as fat as possible.
According to the author, one reason for human beings' inherent need for religion is... [7]

li

11

ohn Gray is confident lhat rellgion is a natural and healthy expression of Man s deepest needs
How convincing are his arguments? ls your generation becoming more or less religious and do
you regard this as broadiy beneficial or harmful?l8l

,r,

lJC2 Mid.Year Paoer 2 2OO7


Suqqested Answe.s
Question 1:

According to the author, why is it absurd to claim that'we live in a secular age'(line 1)?
(1m +% m bonus)
From the oassaoe
Throughout much of the world. religion is

thrivinq with

nd im in

ishedJilqliq

Suooested answers
ln mosl couniries, religion is glg.lgllg

viqorously/ flourishinq/Drosoering (%
m)
Note: grow ONLY (0m)

and continues to exisgstill exists


purposef ullv/meaninqfullv
OR
has !b9Jig9!Ilg[9!g!!lb9:49!to survive.

(%n)

Bonus:
undiminished

unabated (% m)

Question

2
'the unthinking creed of conventional people' (lines 4-5)
What does this reveal about the author's attitude towards liberal humanism? (2m)

the unthinkinq creed of conventional


people

The unretlective

ideoloqv/doctrine/svstem ot beliefs of
conformists/individuals lacking in
oriqinalitv. ('lm)
(Note: without thought -0m)

Nole: candidates must get ALL 3 poinG


to obtain 1 mark. For I or 2 points,
award % mark only.
The writer's attitude towards liberal
humanism is one of

disdain/ contempt (1m)


disapproval (% m)
Note: mockerv/sarcasm (% m)

Question

3
does
the author intend you to understand by ending the second paragraph with
What
three dots (...)? (1m+ % m bonus)
From the Dassage

The humanist view of the earth as an


instrument of hunan purpose is a secular
rendition of the biblical myth of
Genesis....(line 19)

Suooested answers
Since Genesis is ihe fi6t chapter of the
E!!!g (% m bonus), the author wants us to
understand there are other instances in
!!qqj!b which illusirate man's
manipulation of the earth. (1m)
OR

Any sensible answers to the effect of 'more


to follow' (% m)
OR

The writer intended the reader to deduce


the implications/draw his own conclusion
for his comparison between liberal
hLrmanisrn and chistianitv. i'1m)

Question 4

From paragraph 3, describe the two mutated versions of Christianity and explain how
they are similar. (2m) IJse your own wotds as far as possrbre.
From the

Marx's absurd idea of "the end of history' ,


in which communism-t umPhs and
destructive conflict then ygqisbglfrom
the world, is transparently a secular
mutation of Christian beliefs. (line 20-23)

The success of
Marxism/collectivtsm/soclallsm wll bnng
about the di9epp93l3!99".1b!d (% m) of
devastating/damaginq/harmf ul

discord/struqqle/clash/tension/difference

(%n)
Note: for'destructive confliet',
award (% m) only if the two words are
simultaneously paraphrased

The same is true of Francis Fukuyama's


eouallv Dreoosterous belief in universal
galyelig! through "global democratic
capitalism". (line 23-25)

Fukuyama's theory that the equal


opportunity and freedom inherent in an
international market-based /LaissezE3!r9-es9!94y (% m) will 93yq!!9Ih9!9
wotld. (% m)
Notei lor'universal salvation', if only
'salvation' is present, award 0m.

4a

ln both cases, what we have is

Eylb
25)

masque.adinq as science. (line

I ln both situations they are fiction/untruths


I (% m) disquised/masked as obiective
facls. (% mJ

Question 5
ldentify the paradox in paragraph 4 and explain it. (2m + %m bonus)
From fhe Dassaoe
(a) % m bonus for explaining technique. A

paradox

an apparentlv self-contradictory

statement /idea which is


nonetheless true/logical on closer
examination
(b) % m for identifying the correct paradox
from the text

'. . secular myths .. are frequently more


harmfulthan the Jeal thins. (lines 26-27)

oELurdr rEr9rerr \lrE zol

(c) Being essentially stories, myths are


not taken seriously by those who consider
them mere fictional creations- However,
the inaccuracies and distortions
inherent in myths can be more damaging
than the truth. (2m)

9! "'d' d,c wulu,y


or unrelated to spiritual and religious
affairs and it therefore seems
contradictory to combine lhe word with
"religion" in describing liberal humanism.
However, liberal humanism - though a
secular school of thought - can arguably
be considered a 'religion' because its
adherents resemble religious followers in

unqLrestioning faith, blind

obedience, dogmatism, intellectual


belief in imposing man's will on
nature (para 2)
assertion of man's right to exploit
natu.e (para 2)
expectations of paradise and
redemption, albeit in this world.

Any 1 of the above


(2m)

The word "heaven' with its implications of


a torm of tyranny... that commits vast happiness and moral perfection seems
crimes in lhe pursuit of heaven on eadh antitheticalto "tyranny" which is associated
...

(line 30)

with misery, coercaon and cruelty.


However, it is indeed a historical
paradox that diclalors have committed
great atrocities in an attempt to impose
their utopian vision on the world/to
create their vision of an ideaupertect
world (2 m)
Note:

.
.

Answers should be assessed


holistibally
lf marks are awarded for part {c),
dpjgl!award marks for part {b)

Question 6
'People still need to believe that a h9 g!_831re!! can be gtt4pgggl in the 9tA9! of
human events.'(lines 47-48). (2n)
Explain what the author is saying by bringing out the meaning of the italicjzed words.
From the oassaoe
Suooested answers
'People still need to believe that
I Humans stillwish to believe that they can
L see/disceln/perceive (% m) a
qlimpsed
be
b9!ig!]j!9!!can
in the gl!e99 of human
I benevolenUkindlyffavourable/constructive/hope
(lines
I (% m) design/purpose/trend (% m) behind the
confusion/utter disorder (% m) of human

47-48)

events.'

incidents/circumstances.

Question 7
Explain the author's criticism of the secular humanists in paragraph 10. lJse your own
urords as lar as possible. (2m\
From the oassaoe
It is this painful inability to reconcile theiexpectalion with the reality around them, I
believe, that accounts for the peculiar
rancour and intolerance of many secular

thinkels

Suooested answers
They are just preiudiced against religion
because they cannot explain/accept the
fact that (understand why) religious fervour
cannot be held bacldremains unabated.
(1m)

4,

lJnable to account for the irrepressible


vitality of religion, they can react only with
puritanical horror and stigmatise it
irrational. (lines

68-71)

as

... this is never more so than when it


rmagrnes {self to be ruled by

] They attack the supposed absurdity

of

I religion and delude themselves most


] especially when they think that they are
I governed by/directed by/ grounded in
logic/good sense while (1m)
i
I

reason.(lines 72-73)
(include this so that we can explain/justify
to our students the paft on deluding
themselves)

Question

What is the author implying by using the word'animal'in the penultimate sentence of
paragraph '10? (2m)
Yet the truth is thai if religion is irrational,
so is the human animal...(line 72)

Ouestion

By using the word 'animal'to describe the


human, the author iS sayrng that humans
are similar io animals (% m): that thev
are ruled bv instinct (% m). He is
implyins that it is_bs!!gli!g!!!gt_!el!el
than reason ('lm) that explains mankind's
need for religion.

Word in contexl
myths spa@ed by
the Enlightenment
(line 1)

1 mark

+
Y2 m
spread
multiplied

LJJI
started
began
brought
about

Y, metk

Any one of the


% m answers
suggested

caused
spurred
propelled
ignited
triggered

Any one of the


7, m answers
suggested

extraordinary/
special

Nole:
generated (1m)

it returns in

biare

and perverse
lorms ...

/'m

very
extremely

(l;ne 9)

Lm
odd
peculiar
strange
unusual

Note:

1r

weird (1m)
humans are

categoically
different from ... (line
12\
One of the worst
legacies of
Christianity (line 17)

Science has become


a vehicle fot
needs ... (line 42)

certainlY

undoubtedly
indisputably
undeniably
irrefutably

surely
definitely
absolutely

clearly
obviously

A situation that exists as a


result of things that
happened at an earlier
time generation/belief/
movement
Consequences left behind
by a previous
generation/beliefl
Movement
heritaqes
medium
means
channel

transporter

carier
tool
platform
vector
mode
lransmitter

Question 10

Using materialfrom paragraphs 7 io I of the passage, summarise the author's reasons


that religion is an inherent need of human beings, and his argument against repressing
religion.

Wdte your summary in no rnore than 120 words not counting the opening words which
are printed below. Use your own words as tar as possible. [7]
According to the author, one reason for human beings' inherent need for religion is...

Pt
no.
1

From passage

paraphrase

The need for religion appears to be


IAlglgilgd in the human animal.
(line 49)

that it is inqrained in our nature. [1]

...the behaviout ol seculat humanists


supoorts this hvpothesis. (line 50)

This theory/thesis/notion/concepgidea
I%ml is dgEelell1blggEeelaDle l%ml
by the actlons of the secu/ar
h u m anists/at h e i st s...

4+

:th&re

who are inflexible in their


thinkinqrwhose flame ol l-ejerence
about the world is tixed

morelllelllplgelvJglt!

(line 5'1)

lll

tne glsglglyeiqiCjq
-etpla-as
secular thought. (line 53)

ot

nsi

h. this actually describes/makes


clear/accentuates lhe

IY,ml

unc;mpromisinq/unvieldinq/i4fl exible
dlY
[7,m] nature of the secular/wo
thoughLtu/orldview.

cuttures, rerglon is hgligd


in the unconscious, (lines 55-56)

r:lecutir

h{ietiesthat are non-rcligious, lhe

need for belief is submerqed/concealed


I%ml in the inner

ieiit's uuc66-scio,rslpsvc tre lzml

of religion has
,,ilu to reappear.. in qrotesque and FoweverJhis suppression
the ironic effect ot revivinq/.esurrectinq
''rriJit tor i'(ti-*s so-sz)

NB: 3 points here

t%ml it in various
unnatu ratta bnoroa!lb4E!9Cld!9!9l9C
r/,mPnd 1ig1P1 1%m1 manners

lflas some claim,

the Victorians
piano
in a y3!-q-gfigd to
legs
covered
9&!9!g9j9! from their lives, (line

Lut

behave similarlv
"eiulaihtimantsfs
leliqion as
condemn
when they

iecunliunaniststameisls act in the

!IIe!!g!4i.

(iines 58-59)

as tne

pqlsin-ggfsg!

[%m] from our

daily experience is fq!!!g, l%ml

they
[%rn] manner when

criticise/denounce
faith/belief/reliqion J%ml

Eigory and antnolology 9b9!

it to

be a sPecies-wide Phenomenon
(lines 60-6'1)

Moreover, hisforY and


anthropology/academic siudies have

provenlesiiblishedrdemonslrated [7'm]
th=t thJ nee-dior rilgton rs a thorouqhlv/
fullv/human excerience OR somelhinq

@rpsr!9!999

no rnore reason to thlnk that


we will cease to be religious
enimals than there is to think we wlll
6mE diy be asexual. (lines 61-62)

ihere s

Manv libera! humanist th,hkers were


ada;ant lhat reliqion would llgiq!
h the advance of science
lines 63-64

t'l'tl

itrerefore, the need for religlon ls as


inteqral to our beinq human as our
sexualitv. [1]
w-hilemany/,bera,humsnisf s/atheistswere resolved/unswervinqlunwavillnq
tzmt iilfr6ii betief tnat religion would
'k^-;-- -vlin.r l%ml with the rise of the

4t'

scientific wotldview.

-ha@493p991,

and th.ere

is not the remotest ProsPe9! ot

lt

their exoectation has not been


materialised/fulfilled [%m] and it may,
time to come [%m]

;;fi;;6-b;;tin

point)
Maximum 7 marks ('l ma* for each

Question

11

oJ Man s
religion is a natural and healthy expressron
-l*o'n'n-ii"v i" "onria"nt that
becomlng
generation
his arguments? (R1) ls your
0""""i "i"i..-n"* "onvincing areyou
(R3)
62fl-nful?
o'
req'aiJinis us o'ouoti o"nlti";"1
more or less reliqious (nzt ano oo

BAND DESCRIPTOR9

fri

1-3

writer
Those scriots with only a restatemenl ot the
itt"*pt to tel;te to the students gene@lion

maln ideas

lMut

"nvl"o."

passage to begin the discussion


ldentify at least one point from the

Tendstomerelvbeareferenceoftheviewsofthewriterbywayofrestahngthe
nuestion or summattzing or restating of the texl

question or
ivlisinierpreiaiion of ihe idea3 taised in lhs
ln addition to the above,
Tends to be superficial' with
Evaluation is attempted but not always convincing.
limited develoPment

of the issues, though not


Exolanallon shows adequate level of understanding
thorough in suppod and illustration
ln addiiion,

u".Jii;uin",no
"Jt"i"

conclusrons
evatuahon. by devetoping ideas to logical

"no "lear
irJljttion

J,,itri

;il;:I;"

personal rnsght

;;;;;;aluation

A verv good

anJ apt rllustratron

Greater

or poinls an-d treatment or content

support and apt


understanding of the issues' with thorough

illustration.

T{OTE:
'1

his arguments?' (R1),


On the Question: 'How convincing are
their own
with evidence from the passage and

iiualnis muit
experiences

suostantiate their stand

2.

On the Question: 'do you regard this as broadly beneficial or harmful?' (R3),

3.

lvlarkers should not be expected to infer on behalf of the students

RcsponGes should be backed by good ond relevant exampleg

5.

cood scripts should discuss if there is a need for religion beyq!!llhc-99!l!9!L.q]l

Students need to provide a balanced discussion of why being more or less religious
has both benefrcial and harmful consequences.

Sinqaoore

4+

JJC Mid-Year 07 Paper

1.

"The world is addicted to violence." Discuss the truth of this statement'

2.

Consider the effects of your country's move to establish a more cosmopolitan


environment.

3.

History is a tool for politicians to achieve their ends. Do you agree?

4.

"The purpose of education is to make civilised men of us all" ls this view still
relevant today?

5.

The benefits of environmental conservation do not justify its cost Discuss

6.

"Good governance is the only reason why Singapore has succeeded " Discuss

7.

"Technological advancement has worsened the problem of poverty " Do you


agree?

B. "competition is inevitable

in our increasingly connected world " How far do you

agree with this view?

9.

"Access to new media has empowered individuais, but weakened


establishments." ls this a fair statement?

ro. Freedom and responsibility are inseparable. Discuss.


'!1. Ac'tion packed movies have undenieble appeal but

qu-'stioneb!e'.'slue Discuss'

12. Can discrimination ever have desirable consequences?

4t

P5
Q

Pb

--

gn -

P9

E.E

:E

&=

E
q*E

:;_F
3 E 0: :
3o
! eE -P3]
irD:

ee

q= ;

3e

9;E

Eh

3h sElE $r E9E
o;6:;64:
t=zn..oa
rE.q

r:-;Sg

_E

-- E g o
E 5 RE

.0

6:
=s3
:9

6.e

UF2

d_j

e
3 3 E*
c

d
!-n. 6 d=

dPq
ok6

; Jrt o
s

e4e
z<1

9='rJ

p 6^

od<

-Ef

E+IE

-ro
o6;
9o;

-;8. E E
",9
FE-9::.9

fr

b-EP
F

gEq>

3",9;
F

n-9

izu

P9s

g=
';F

i6

!;lE

c= - {t;i
a5 6 E E; >
:d";"

E
E

E, P-q -s

6.: b

; o;g
- o>
-9

EEt

3p

o-

>i

3 9-3

K-

=el=o4=O{
>; qb-3 -;
oDGi= I F

o5

5r:E
EI6
;eaa

CE

F{

g!:

o,

-s:6Y
'= 6-,"

'

)!o4S

.q

ijp

,-

i9

^
e

q
F

-:o ?.
.".9

dP

qt

oo9.

n -6>E

ee

P l?;
i
;i qe!
9o F F b

:Ef

9bbb

P!

=!
; ols

tn

$;
-:
ed:
d* 6 6,! i

.9

Fdoii

a-U

>F

d-9

E9i

qca6
=H
bol q 0 >6 -

PaPi,q
:6
F " 96
5 o irl qro-

.eb e 6
E

FegP

.9

.e9o^9o;r

6# i; e;
^i

4el

P o-=^=

>uq9ii

g: e3;'4 I6 -: E;t
n6es$s-;Efi:!!fi5

i!)

3f;e
-sE

--Fo69-1:;b"go.-

*.E
o9)

s!-9;R9;e-sfr=icE
Ei:fr=
j ei*,;t .gF;:5E
fi'- i: v; s
s!ssse $9tei;s;$

ob
o,9 q
E E q,5
-69
Eo'E

'

g39Eae P
ePb
q.,>EEIEipFe

ii-^:Spr
Eb"9'9=

e93,
E=F

:g*EeiE
.!oys.69,3*

l:eE;
.,E "

gn:;

ij9.90

i;

{ pat P 9
E:e
*'P
b'i "a ;' .
\-:- I Sdte E.q
-s5:nA3 ;-e 9 g=>
eb-'.eF

q
] \i F!4 e F j:

,- E

'9

4*

EIc6l:b i5po s=-iiHrs


s f P:::s 5
i 3 Ha i
0;:!'o-te>; aft:;X:"
Eir

oii

.ii
drr.f

! O"=
Egb
s E; <Es

H.9

.E

96

: a't

;!i

E dE

6r ^ "b;;-e;Fs'
q t * LlE

i6bEi,,c;3

E!ur>o-"

6:
P-t

q-

o::!
g5:

9
tc Hig''9i,r.*Etigt,:i
: j o - o - I : i X :' o r 9 '
o,, - "^6c.
6i6
-djo
--0
E="s, ; i I : I g ;':ir 9 i **=' I
xs 3 Fri i rFi E!:E g g: ! e

I
qZ p

RE> >

2,.p 96:,sP.E
SE;3.
d !

H.#;9i b:J *g e

4;o
;3=epu6

ii!

?e *

o,
Po

., 9 >

sl r;

at

+**ij;+{iF!!EIpp=
} 3 F;E 5 -:
sg

;
I

o?ic
s9
;
6E;
P"-;P6P
9@
oF-qPJ?o
=oad
=
E;=E:s'6
i= P:,HE>
-j*5
,:;
E*-;r:g; fr:3:E
i ei b:"

PHHEg:ueHEEg F3!

e,ese;e;-:EE ;i3

o
9ra

r45 ii i;
E
E

c9

.90

;9

.9

+; ' 5;:
g{;c

+E"sggs

ig

to

'6

i ;:l:
F fE l;fr F:;:;gi:e"

; j
: i:

:^;st:i;!:4gEl
n -! ;aP9.3E
:tE

::+!ie:r";UbJHl.s
i n'ii;c} :t i: f, n[;l ; : g{
es j E i ii; ";i i-p ri : o o'
i Pii$ l!;;;Fs abt Fi: i;;
s

*'r lE ?Ffi ii !:: g, :i,"8


*E
EF;:;-i!:
iH; 5.'=
E;:
e; [.] i e e--!* a I;:.6:1
pgfi ;i r:l![; ;:p;; e
*ii5!;
d
g s{ l;'.:g; g 5;$E:51!: b::'Y
$ !ii :r:tsis $O: ille g gEl 5i
Pl i, ;tsi'BE i Jg tFSi lFiI:?
6

b5 o *

I -;

;-

._ ?

E;T

F*

r=i

S*

",
F

s;

I E d ;e -o
=3 63 q* 3E

".F
6ab
a

"

o,

ss1
q= d o

:;EE

ibi.
c Rb 6
I

;;R

d.s

qsE:91
31aF
vq
.::o;6:
-:
F s a 0E I
i *Ee " 99! 6*

il;

gi.PA""6

e-*r,"eg
e; *.; :i5 El
.;E
rd:;1"
:;:el:;lE.I
;q;i
ru"iq
ErrEr FFA'co^*id!,ol
";;s;oEsr_i .;;!;3
-oo=o
I i : t : * ir t e : ',! i-: : I ; ; : g : ". r g 3 I i e I
" ';6{-R:r !EiE",t
gp;gu:gfrrgtiiiiil,r;"Ll
o ipLp,i#!
4:; a? p g { r; S5r !f i?.; e 4: a p I95i3 ej
ii
E-e

;;o@

F
E

r:;.

Ii::t

3,
;2

aHF
-i'c"; EgE

iP

Ea

!, e; P E;5 5
3 3 a 5 E AE
.E

r bf

C] !

5,

i:-;o;
qq3[g* pg'iE
ud",o>o
;;
tu
+=,:iFP5;B:i=E
-:9q9^"

1^

3.3s;:.
..9i#BFp";

c <v, grU:ii,
5 :oqai

!;d

9s's.^
r3..o !'b
rt:i
: ;Ibq
i-6
iE:a[:d

tb
P3

6>nr> ao
esqF;Frqf-!o
BU
r... ?,[;E:f?$!i
io,o-.o.=,

o9-t=E-==

.gEi!!;g!F
E;;r 69t

5::

: aoYoi

c>;:

5/

.e.

6F!g:

3p 3n

:hg" :

I $6

'-q
E;
H: g r6
.,a399
g=
9Ea_ ; "i!l o-'@* rA EPl"l
Eo'
o.9e-t; :=5"9;ga
E'-

g:

;9E;

!!

!;

9r o
Eo

sl

.t
'a
sESE
E*:

6'

E{'38
qq

se;:SE

;;cI

-Pe. P;rC
;Ee *;; ;p s *; * s;
e3E cE
o5
F

o"o-c

!9=

9^6::
o,

ffiEi+aiiFiEgi;; }E

96
@:

!b
orfi "
".!
as
9.b+P;ij,'
HUi- F'l ,"Y:Pr-g

,":"
.li^

ar:l-oYoojj
oi uo

;;6",
,,6E;

EiiEJ

"4.i
Fl6=

i=iilE
!tbi!ii?E ;i;E
3iEi" g d,:,!e3sE5: 5,39+

.9

- ,,E

s,.icPh@o.

'o.>-

;iE:nrc;H;:A:i5rfiiFi
iff60cseFt3;i3g$;:ipF

"Ee

:'b

?
6:
!s90

hi

-F;

9a
sot s!h ::
:-t
;pI
eE

;i,EPP-oEft

a e EE:
9!>eotuE

l=
! 9 o q-

tr
^t
Pq 3i. e 9!iqPsIi=
,Pib'i
i*
tiEflt
!3iii:'-E3g6LEI
u:
iE3e,a"E;EP EE;3=3:i
b.'

t;ils;Rds

.!

3i 3 ter ; :::: "'[:

.9

oj

c.13ii

fl

eirc

.E -ts i

ri!
:EgIgiBI;IEEE
g'Hn;3::3;:
-E:;3s
iP;g;i sEggt#E!

9cob E P 3dt

Et

.9 ,. r
6tr o 6-

dI:

.,ooiroc2oo

63

.:

6P
EF

,E.,i:e
!q-e*t'.PE
6 9!:9? - F-:

41

E;ialiup

=o
6E

6.

.0,,Es5gE-E

t-

D-hnr@
!9d
:F9,
6

q
$
6gs
E

H! EEe;
E Eil;F;H

;!
r

qEiE

; :E,PT

;i; fesEl;,: s;i

EE -o e
-E I'

os.pEP

.9

a:

: Y

uFgI!
,3 g9 3:
.e

BF qB

ll

O P q

n"

J]>:-.

q .. ts o - - t ao+ t:9o
x o o
I:;6
E
EP

sadSPI

g:5

X:F;o.9
,^!;! xt o)

E
0l

Pe
o!

!e#:9!

96

+3

!E

F
F

t?
l

'6i

E"

5' -'coo::

46.;_3 $!

g
o

.9

>

i:b
1E

:i eE ".9
o 0 ":E

!;
cq
4
rc:
Y c o!
qeYP"4s
@;.
o-- o
, : E o< d o

So
I

o --

.9

EIEi!

!!6

H;

q6o
;P q.-gE
.e E e
Fg

3h5

-9q! o;
"

'6

>;6

d9
F

FE }

o6-E;^.

i !e 3 s
0 c; $;
q

6 A C;:

59

5 E;

ab;

C1

t3

-p
E:
l>co
4

b;3

ii

614

o9

5:5:

=g.'!o $ B
r;c o

s:;
d

.
P i.
AE
- o= 1.y
'=
; oo
;i

tsi9P

'99-

i; ;:;#:;E=:5FEgC+t
r :; f,;; as: ie,c

-o;
9pb
O:E

r')

"P
?:a

isi;e

(t;6
=

T:6=

iaE;i:siit

E{s!=sct5-EaP:5*:

35,

$.e

ar

! gi

m.

fi

s3d:E

!:

="iE
:, '-=i". s)!!

tH
=9q P s
1E{85.;--o:
!::
F-9:ii:
:66:,e609
r:'Fr
6
q
0
: ;".{3';!bE9
-? E a >E :: o oq c a
Pg
*E,'=".iS
!
gEe;E aoioeii;
9 V;E F 3 PP F "BE
.ii
p8
g

!6

9r

o h ! E h 5;

'

et;i;59Pi
r:!peEsE

..

i;
E;Ag;EEIFe
i* t r*;*=EePt
F

; ; e 9 3 X'5 ;; rp

g#$!*= HEE;Egr:lf

3E,

'6> =6t9i:E
> q: F 6 99E

i:i I E.'-i
""
P-

5e

.P

!e

o *E

c!:

.9

=,

3!'

E()

o
c
h(1

9;
-:6
Y:ooo
: ! 9.9
6:5oqF

o.91

oa s* 9 E
-go .>s':'--E
b

F69 ido 9,:PoBP

': -;

;3;;9
EE_ii E6
?tLt, i:
36P;.'
!:

PEf :f r
9: ;;=;3d
=*
FP:pi=s:

Y.9.!.E; ]I;',s
o

s4

]
E
F
r
; ;ti,5
-,i.i- o:
:Fi*i:[iriiPfi":[:in_
:cr

b er
oI

i l. "

F< E e 3q
d:, .: 9,6

E -A

:I

6E

gsF

;91
:o6
=

b , t.. d E ! d
F

f sghEt :iE!,s; i fi 3*;s g;

S: b
?9 E'
E

-o

i;

.gc

b:

59
93
6o

t;
lc

o-

6:r::
t-q-aiih

5o .a
=
E.
!fr
9o
-99
l9

F;E

iiN 3
E di-i
; I E FI

9-q
q

I
Eg!io
H

9:;-9
fi--5i 9- t

e9:

sE

egie Pi a i' ;!;;

a!

!:69>
I -o* e F
9,-i E de

o,

gq

Pe.

EgEs*3 F
.3PP:;=

6.9

9T E3#

I:

E:E
o 6 ?: F

o " - o @ ni ^ i,
;9
e3be

F 5-e'g

I .. !EEgBrE

o,

E -6

s g=

s:E; !!-3

*S

:E cS e

o::
cA5!E

Eb 3EE
l6

E;5 P!

:9

=
.9!r"ElE

sF

JF

]F

eR
0 9-

.9

q;'o.
q o!
6!:q9;n,:"1

;+:

3,rE!i

;!
P cE
50E3q

+E!3E I

3P.
q-!: i -:lo

'6

sTs
6i6

!*

3
2e

'6=

js

.)l

zl
=

'

E: P
= e =9:
z

Ql

i;=_

Ug

Fi!

99
:.;

FPQPPQ

q-9:,

.9

h g z'6aai,

ur]

". :;

L!,,,

oYl

EA!=3
9=q

-P.e

Hi'
*i
!r*Fi:TTit
EE

Prrnr
& tl

rE

:- Er o.:.o t o r ":E

.--<

9=

t6 t
lE

F"^. 3q

5i
n:j99^ie-q;3;

9a

! E;H;q; FEi !.9.


ER eEd.:*Eq3EE33
igE"a:E{fEeFs

::
=

e 6 3
=,=* p!

-,r

a aEE:e
;e;eaE!

e!
if
E1 t=

F:!F=gF

t;e:r

E*;:Ei;

Es; 65: Ei;FfrE


c

I$

=^

!4-

--"!
;.1

;i,d

<-a,:

-6:-o
,t;P
x6q'os5:"9g!,i;
5i::[,E6r-E;o{

H::,:JE3;Pi'Lre34:8P
0:6o
ijo
o(o!oaq-(of
o
d,gisPS$ii:H:;:3F;i5

rPP9

qL

r ooLo
c9o::Fr

l-

c,5r
:. q

50,06:cj.
9:qLdi-e

!S
!

t-9+
6;l.q

"E:
i;Hr lf i ispg:r. eb!

-:;_
5o;
d_L:9uo
Sipi ea? Eio-.<P3 ;6
*s!:
e;9 e:.3sq3,
; -rLsH=*r'i,IneiiHS
8"1H6*Xa
E6t!iEri;Bg
r<6<6r
oT36t
6FUl 6:

Et
-p =
o-P

.,-e
:6 Ft

^l

E<
o

'o

oq

t EE; P"EE

EF;3
::i
E :ri:i*rhi
e : 3 ij " e Tzr^-

tsit s*;:*i

b "oE.

aE Eg;
f=-i i ! ttrEed
i:38:;-".
iEFtig
r liE::i,uHfr
ai*Hh:eH
qlri.e -uQ;*;rs,

Eb,
6 >E
a2E

0?4

4 :3 E- !3s""i;. t
X$s ;;Halil s.f;
E
ebi;i; .;e 5 .Es;6 gq,

;EPE
Eg
o E;

, L o!rF u
,4ttP.
, j,"
l" {9!ee ^-

-c 4d_c

E.ii
t.s* La: 9P Po,i

3 c ii':"

og,

<tt

Et';::H-[E!
H

-3p:iadiil
;ip\aEEp:i!
5 b! e,h 3[.t. ,3 S i 3:3
6< or':4' ti i

EoJ

'

is
'6= E{-995E

;s gEpBE3
OE

g
FeI
f
f
E I E b:b
Q e c 9:,
p; E6 e 3

-o

e dq;

'io
B

'i;

o. b._;,;J
.5r
-:;;,o.i"
gt ur:sri
; o

!
I

;{6'F',:
i'1
?;:, - f {iol:i
ciFiiq-'i;q

.,

BI.e B" :;_.9


ss",T
3;;';e
acgS:6 eF9,
I
69

n....os:
.s3*Bf
EL:JEq

-t

E[;;
5-?.i3
- E9o.
o
E -:;

t,

.!

.9

if!

!i E o B
"
':::9*
&3n r:
r'ld:ocofz::.o

:6

qr o
:e
i
to566
oY
B g.r B E
r6:9 oJo06@:(! f -e 9 =geE',5sgi"- lx g, 5oE'-h::" --.
eg-b9
P;g
y
EeltEHs':li;e=:eE
si
t
s
2
q - o: 4
sz !! F h B;ec b E! 3Es,q q:6n
Fqg*s
=&R: ii PF;,lsPF*5E;;F39g ii9!6Y9cc
EB 6I
dl

13

-9

e.P"

+n,
e o qar
=
X'6!!
: o

oi!\o

!=:E
;E3?:
Eco,q !:-or;,;4
:: E- 9:r:5

(s

* e;

r P39 b
iib b; E:
:dii!6:

;3gPe*gl:"i6# 9od:os

;6s;:ieiiaE: 5 9 6c 6'
Ee

9*

q-9

'6=fsp
a

ii*;

HE3;Es3g=3t

o,

?
g;Cd
r

.9 .,

i2
F

HEa

FEF

= -=:
;Fig ",9 ->
.e E He:9
ai

9
9
e -s

fl;.'i;
EEP:Et
n

'

hE

rE -

r" E

e;

ci1e:!9

6+atse.
;; Enii

3E
.9

3
3

'
,9

E. l? e 6;=;,4
..;o;:,;tb5;

i!qi[".;:g
i E 9 P'- - e P!
C

E6qsEE*t;
;I:!6j::ou:!o

e .2

orE

;E
9{9
doi
-oc

-9

i;?E-

.ooY!
.:ort

o,

:bsll

.9

;.e o ii;

.,:

Passage

Ruth Hubbard says...

This century has wilnessed holocausts of ethnic, racial, and religious exlerminalion
in many parts of our planet, perpetuated by peoples of widely djfferent cultural and
political affiliations and beliefs. lt is beyond comprehension that educaled persons
can come forward to argue, as though in complele innocence and ignorance of our
recent history, that nothing could be more interesting and worthwhile than to sort out
lhe "racial" or "elhnic components of our species so as to ascedain our root idenlity.
And where to look for that identity if not in our genes?
Every decade or so, a new prophet promises lo decode the currently advertised
Rosetta Stonel that will enable each of us to not only know who we tauly are, but to
use that knowledge for our individual and collective benefit And, of course, no
period has inspired more hope and determination than the present, now that large
sums of money have gone into spelling out-"sequencinq"-the molecular
composition of the "genes" which are part of that long string of'bases" curled up
within the nucleus of each of the cells in our bodies. To make good on this, effort has
been rendered all the more urgent by the fact that some of the scjentists pioneering
the lask have suggested that the very volume and complexity of the new information
make it virtually impossible to interpret- So, what to do?
Why not use the intormation to tackle the meaning of'Jace," the question that has
bedevilled Europeans and both European and African Americans for a very long
time? And belter yet, why not use it to tackle the relationship of race to health, a
question that haunts us in more recent times? Let us disregard the differences rn
income, family slructure, education, rales of employmenl and incarceration, and
such Let s gel down to the basics-our essential nature: our DNA (or "genes"). lt we
could establish "racial" gene cluslers and if we could use these to predict disease,
and so prevent il, wouldn't we all be better off?

'4

t0

15

20

25

Severalthings are wrong wilh this plan. For one thing, lhe manifestations of inheflteb
conditions can vary considerably and unpredictably from one person to anolher and,
indeed, in the same person at different times. This is so because many factors, both
within and oulside ourselves, affecl lhe ways we develop and function This is true of
our biological characteristics as well as our psychological and social ones. Even 30
conditions such as cystic fibrosis or sickle cell anemia, which foilow predictable
patterns of inheritance, can exhibit a wide range of symptoms that differ in thetr
severily in different people or, indeed, in the same person at different times
When it comes to the more common and prevalent health conditions or diseases,
such as the various cancers or the vascular conditions that can lead to hearl atlacks
or strokes, genes do nol predict the facl or tjme of their occurrence or their severity
with any degree of accuracy. For these sc{ts of conditions, a person's life
circumstances, beginning al birth are betler predictors than their genes are This rs
not to say thal "genes" aren t involved DNA is involved in everylhing that goes on rn
our bodLes by virtue of the fact thal DNA specifies the composilion of proteins and
differenl proteins are continuously being synthesized and participate in all our
biologica{ functions. In fact, that's why DNA is important But the relationshrps
belween our DNA and our proleins are neither simple nor one on one and they

35

40

rThe Rosella
Slone ls an ancient stone wilh iexis engraved to honour lhe Egyplian pharaoh ti is commonty
used as an idiomalic lerm lo suggesl a discovery ihal provdes knowledge ior lhe solvrng of a puzzle

36

change over lime. That is what makes genelic predictions problemalic and

rrnreliable

45

Furthermore, different degrees of disabiliiy are associated with different mutations


and the degree of disability is different for different people as also for the same
person al different times. Stranger yet, a mutation that elicits severe symploms in
one person may elic'l no symptoms at all in others. No one yet understands why this
is so except to assume that the relevant genes and proteins engage in multiple 50
interactions wlth other things going on within the organism as well as outside it.

The complexilies that emerge upon closer examination of such relatively well
understood patterns of gene function make a mockery of the prelense thal we can
use the worldwide distribution of the genes of human populations to shape our
understanding of our hislory or lo guide social

policy.

55

Adapled from Ruth Hubbard, "Race and genes'

Passage

Moore & wilson says...

Let's begin our slory with a pop quiz on genetic science. We have a group of follr
men. lf you were able to look jusl at their genetic code, their DNA, which of these
men have the most genetic differences? I have no question, given what we know
about human genetics, that the tall person and the short person aIe gignificantly
more different than lhe black man and the white man of a similar height
The human qenome project is one in which scientists around the wodd are kying to
map the location and function of every gene in the human body. The viewpoint of
most geneticists today is that."race" has no real meaning in science. Trying to mix
genetics with race is inappropiale, it cannol be done. Race is'something we do to
each other: it has nothing to do with what our DNA does to us

10

lndeedp it's based genetically, but it's a very small part of us*How small? While each

cellin the human body has 100,000 genes, only about six genes control skin color _six out of 100,0001 What's more, everyone reading this article shares the same six
genes, including lhe genes for dark skinl What that means may shock some people;
that each of us has the potentialto produce skin as black as an African native. That
is why geneticists say race, like beauty, is only skin deep. lf yoLl expect that there
exlsts pure races, lhat is totally absurd Think of what people would look like without

15

skin. When you do that, you see we're all alike, we're prirnates.

So now that this information is out. peopie might accepl rt people mrght

not But you

can'i era6e what you've learned throughout your whole life. Like what about racial
differences in sports? lsn't that genetic? Sorne people say black athleies have a
genetic trait called 'fast-iwitch muscles " which allow thern to run, iump, and shool
bel1er than whites. But science tells us that lots of athletes have fast-lwilch muscles,
while nol all black people are good at sports. Why are Latinos good at soccer? Do
we ever talk about their 'soccer" genes? ls there a "martial arts" gene for Asians? A

20

"golf'gene for whites? What about Tiger Woods? Did he gei his abilities from his
black falher or his Asian mother? And that's exactly the problem with trying to deilne
people by race; there are no clear scientific categories that truly separate humans by
the color of their skin. Bul that doesn't mean some people don't try

E
'

So. f race rsn't based on scLence, what is it? The best evidence is lhat race is not in 30
our q"nes, rl's rn our heads, something we made r.lp a long lime ago ln the days
when humans could see lhat the earth was flat, they could also see that people
looked different. And though we now accepl that our eyes deceived us aboLlt the
globe, the belief in racial differences has been tough to change

a
-

Essentially, race rs like the dots on the screen. ln science, race means nothing The 35
dols mean nolhrng until we create the meaning, as we've done over human history'
Race isn't science. Race isn'l biological So it is helpful to go back and use science
to uncover the fact that there's no scientific basis for all the racial differeaces that we
make such a big deal over. But we can't deny the fact that, even though a social
construci, it has become a parl of our lives, our everyday lives And il stands in the 40
way of our being able to achieve all that we need to individually, collectively' or as a
nation

r'

The solutron to race relatlons may be to form relationships with one anolher. lt is
absolulely recognized thal this partlcular friendship is diffictllt to create People have
a whole pile of history that makes it difficult to create these kinds of relationships. Do 45
we throw up our hands and say it's dfficull, it won't happen? I think not
Adapted frcm Pam Moorc & Pete Wilson, "What is Race?"

6o

Questions on Passage

Fram Paragraph 1:
Explain how it is ironic for "educaled persons lo wanl "lo sort oul the tacial' or'ethnic
components of our species so as 1o ascertain our rool identily " (lines 3 - 6)
121

From Paragraph 2:

What does lhe author mean when she says "a new prophel promises 10 decode ihe
currenlly adverlised Rosetta Stone..."? (lines I - 9)
t2l
From Paragraphs 4 6:
The wriier suggesis reasons why the knowledge of genetics cannot be relied on io predicl
diseases.
Summaflse the reasons provided from paragraphs 4 - 6 of Passage 1 in no more than
120 words, not counling ihe opening words given below Useyourown words as far as

The ftrsl reason why genes shauld not be used ta predicl health isthal

[8]

Frcm Paragraph 7:

ldenlify the writeis attilude and explain how il is onveyed jn the phrase "make a
mockery of the prelense..." (line 53)
t2l

Questions on Passage 2

From Paragraphs l &2:


ihe writers' inlention in using ihe Pop Quiz.
Explain
.

I2l

'

From Paragraph 5:
'Why are Latinos good at soccer? ...A 'golf' gene for whiies?" (lines 24 - 27) Explain the
writers'inlention in lhe use of rhelolicalquestions

I2l

Hom Paragl apn o


ldentify
and explain the figuralive language in this paragraph
.

t3l

"

From Paragraph 7:
Why does the author feel thal il is djfficuli to create relationships wiih one anoth_or of
differing race? Use yo,r oW,? words as far as poss/b/e.

t1l

bt

Question on Passages 1 and 2


Give the meaning ofthe following words as lhey are used in Passage 1 and Passage 2
You may write your answer in one word or a shorl phrase.

exlerminalion (line
bedevrlled (line

1)

...............

19) ......

Faam Passage 2;

map (line 7)

absutd (l'ne

I7)

essentially (line

10

...........

35)

.........-................-.-..........t51

Ruih Hubbard argues thal it is unreliable to depend on genes to predict our illnesses.
Moore and Wilson discuss how our race is not delermined by genes.
ln your opinion, is there a value foa the study of genes? Using relevanl argumenls from
lhe passages as well as your own experiences, justiry your
I8l

view.

(L

JJc Mad-Year07 P2 answer scheme


Questions from Passage

r.

Explain how I is ronic for "educaled persons" to wanl "to sorl out ihe tacial or 'ethnic' components of
our species so as to ascea6in our root idenlity " (lines 3 6)
l2l

Lifted
educated persons can come iorward lo argue

as though rn

"o.pr"t"

rnno""t

agnorance of our recent hrstory

""

it

.,s expeciea-

qars!!I!!9q4!&{9d

i,lit

eoucated

pe"o* -,ll

be

*lt

unj informed/most knowledgeable about histoical


events and lraqedies arisinq fronr genetic/racial
determinism/ discrimination to wanl to
prcvenu.void tha qdnF IragFo,er lo n happening
again/makinq the same mistakes. (1)

"

Onty

thal nothing could be more interesting and Howeve. these educated people are actually the
wonhwhile rtJn ro son oul lhe ldcdl' or ones most obsessed/ fixatpd the sortinq of
''eihnic components ol our species so as lo difierences within the human species/carrying out
asceriain our root idenlity
the segregation which may lead to hislory repeaung

Whal does lhe author mean when she says a new prophel promises lo decode the currently
adverl'sed Roselta Slone.. "? (lines 8 - 9)

I2l*1
Paraphrased/lnfer.t
Jusl as a new prophei provides guiclance io make
sense of a difficult probl8m, (1)

(scientlsts pioneering the task)


who promises to clecode
sequencing")

.so new generations ot scientisrs hav emerged

(spelingaut

lhe curently advertised Rosetta Stone

lo decipher/make sense otlshare their


understanding otsolve the
puzzle
oflinterpreUtranslate the gene poou generic

(the vety valune and canplexty ot the new siudies (1) '

jnfornatian rnake it vitlualy impossible lo interyret)

'ln

bonus for answets lhat points

hyped nature of the pronise

to the aver

FromParagraphs4-6

The wnter suggesls that there are reasons why knowledge of genetics cannol be relled on to predict

Summarise the reasons provided trom paragraphs 4 - 6 ol Passage'l in no more than 120 words.
nol counUng the opening words given below. Use your awn words asfar as possible.
t8l
The

tu,

teason why genes should nol be used to predict heallh is that.

. ihe manitestatons of inheited conditions


can vary consiclerably and unpredictably

Different desrees

oi

nisabrl'ty

associaied wdh differenl murations

inheiled

diseases

signs/symptoms thai differ/fluctuate


loVnoticeably/signincantly and randomly/
erratically/ capriciously
Secondly. varying levels of affiction/ ailmeni j
are linkecu related lo various chanqes/ l

63

irom one person to another in

BD!h raLlats ate lurther oppendant on

the

Vaiations that

same pe6on al dit{erenttimes.

\hF

behryeen

individuals/people and between instances/


occasions within the same individual.

.degree of disability is difterent for


different people as also for the same

..due to lhe numerous/ large number of

lhis is so because many factors, both


within and outside ourselves atfect lhe

internal and external influences/


determinants/contributors lhai have an

ways we clevelop and function

eftecl on how we lrve our lrves/rnilLence ho*


OR

relevanl genes and proterns engage ln


multiple interactions wilh othe. things
going on withjn the organism as well as
Even cond tions .which follow predictable These varialbns occur evn for known
patterns of inheritance, can exhibit a inherited diseases/ illnesses lhat follow a set
wide range of symptoms that difler in of highly ascertainable/ foreseeable
severity n dirierent people or indeed. in
the same person aldrfferenl times
When ii comes to the more common and
prevaleni he;nh conditions or diseases.

The siudy of genes ls nol abte

foresee/foreteluanticipate

genes do nol predict lhe fact or time of

the

to

onseu

inception of widespread/prevailing itlnesses


OR

.cannot foretell when or why an ilness may

rity with

person

any degree of

ihe level of riskdanger/how

serious the condition may be wilh


'9!9!4!!!racJ4l9!!!$!.
ln tact,
a mo accurate indicator ofour heatth

s life circumstances are

better prediclore than their oenes are

the condition of life itself

But the relationships between our DNA ...knowing that the way our genes are
aQd our proteins are neither s'mple nor associared wilh/linked to our biotogicat
indirecv not

.e!d l|qyj!?ltgg 9I9IIUI9


",'un,opn

4
-he

srraishtrgrward/mqlli-E!9!9q-co,1pl9l
and tlrgtabte/do noistaykemaiionsrmt

-'An) S our al tO pan s dbare

lde,rilv Ine w teic dtl lLde a-d e{Darn how,t rs convFvFd

pretense
.o"1o e\

le"

(line 53)

a mockery of the
12)

mdhe arnoct<oy.

. ihat we can use the worl.lwi.lF


lhe genes of human populations
unclerstanding of our history

rn rne pl-rase "make

ot te The wiler

Paraphrased/lnfered

made use of words


connotations/degradation like

.to suggest ihe possible repercussiom/negative


consequences re. rh'n9 from gprerr derelmrr.sm
t%)

6ry

f ',""rr.

"r^,"*". ,

Questions from Passage


Paragraphs

5.

and 2:

Explain lhe wrilers rntenlion in using lhe Pop Quiz

The tall person and lhe short person


signiticantly more different

are

in iheir geneUcs

t2l

The pop quiz suggesls thai there are more


considerable/ noteworthy genetic differences

oetween

pFop

f earures/appearances

...1han the black and lhe white man of a similar


heighl

rulh differenLes

in

(%)

than there are genetic diflerences between


groups (%)

people trom different racial

' Answets lhat atlenpl b ad.Jress the concept of


having nore/less genetic .Jiflerences wlhoul
bringing n lhe conplele conparison between
appearance an.J nce wil be given %n; answers
with no attempt lo nake any forms of conpatison
Race is somelhing we do to each other; il has
nothinq to do with what our DNA does to us

The walers' intenlion is to debunU discreclit the


atlachmenl of genes to lhe concepl of race
OR

The wrilerc want to rejnforce their pornt that race


hacl nothang to clo with DNA/genes
OR
The wrilers want to engage/ altract the attention

Why are Lalinos good al soccer? A golf qene for whites?' (lines 24
inlention in the use of rhetoricalquesiions.
Liftecl
Why arc Latinos sood ai soccertD;we ever iark

about lheir'soccei genes? lS lhere a "ma'1ial


arls" gene lor Asians? A 'golr gene ior wiites?
What about Tlger lryoods? Did he get hrs abilities

27) Explain the wriiers


I2l+1

The wnlers are making a poinu reinforcingl

emphasizins/ pe6uadinq readers (1)


"1n bonus lat answerc lhal explah lhe function of

rhetorical queslion

as

slylistic lool

ta

pose a

from his black father or his Asian moiher?

question wnh an obvious answet

And thats exactly lhe problem with lrying.to


denne people by racei ihere are no ctear
sctentafac categories that truly separale humans
by the cololr ofiheir skln

. .lhat abilities/skills/talents have nothing to do

ldenlify afd explain the ngurative language in;his pa.agraph

l3l+1

Paraphrasedllnterrecl
ldenlifv simile

'Race !s like th dols on the screen (1)

'1m borl6 far

answets idenffytng
similes an.l analoqv here.

the use of

ts'

Jusl //(e dots on the screen makes no sense

The dots mean nothing uniil we create the


meaning, as weve done over human history

uness we interpreuinfer what is happenrng.

Just like dols have no meaning unless lhey are

n science race means nothing too

joined to torm a piclure. (1)

so race remains a meaningless concep,


baseless hypothesis uniil we inject meaning
into it
OR
. renders itselfto possible socialconskucts
OR
rd' e has no -eanrng as rt,i a (on(Fpl we l-aJF

Why does I he author feel that il is difi cull to create relationsh ips with one andihe r of diffed ng race ?
Use you a$/n words as far as possib/e
I1l

Lifted

Paraphrased/lnferred

ll is be;use peopie have a lot of/

People have a whole pile of history

(holocausts of ethnic, ncial and re gious


externination perpetualed by peoples of widely
differenl cu ural and polttical affilalions and beliefs

many (%)

lhat makes 1l difncult lo create these kinds of ...negative/ unpleasant predispositions/

preconceived notions/ prior negative


experinces towardspeople of anothe. race

1%)

' The idea of past experiences being many and


negatNe musl be ponrayed fot a lull nark
FromPassageslA2.

6.

Vocabulary:

I5l
0m

,o/ocarsts ot elhnic, racial


and religlous extermination

Eradication

(N)

Eliminalion

'To get .l af

sanething

the quesllon ihai

has

Complete/Total destruclion
Getting rid of complelely

Baffled

Geti'ng rid of
Ending of
Killing
Slaughler

Spoilt

bedevilled Europeans for a

very ong lime? (u

question

lhat haurrs r's

Cursed
in

'To be extrenely puzzled


Bugged

liyins to map the /ocatjon


and functton oi every gene

Denne

(v)

Charl

'fo arcnge and detine

in

ldeniiiy

Explain

Show

Aff?nge
Drait

ldentify/show the position of


Draw

Match
Link

d. No such lhing as race? Has


sc

ence suddenly gore ,rad? CGzy


thai is tolally absurd (Adj) Obviously/Ulierly senseless/

Laughable

Slrange

illogical/ untrue/ incongruoLrs


Contrary to allreasons
Laughably/

'Obvlous/y serse/ess

Silly
Siupid

llad

So... what is ;f? Essentially


(Adv) race is like the dols on

necessary

'A
n d ispe

sa

Basically
Chiefly

and

Overall
Largely
l$ostly

lMainly

ble crit e r ia n
Crucially

Obviously
Especi?1ry

7.

Application Queslion:

Ruth Hut'barcl argues that it is unreliable to depend on genes to predict our illnesses. Moore and
Wilson cliscuss how our race is not dte.mined by senes.

tn your opinion, is there a value for the study of genes? Using relevant arguments from the
passases as well as your own experiences, justify your view.
I8l
Sludenls need lo argue either in favour for the study ofgenes or against it by evalualing or discussing lhe
benefits lhat could be derived irom il, or the deleteious effecls that could possibly stem from it Ihey need
lo reier lo sonre ot the argumenls made by both aulhors and make clear their oM view and describe
some of the experiences or evidence that they could draw on to suppodlhem
The ldeas from the passages that studenis could discuss include:
There are those who leelthai we should delve inlalhe sllrdy oi genes in order
1

This plan has cerlain drawbacks Firslly. the symptoms of disease can deviale

from person io pe.son due lo varying faclors or cond(ions lt cannot be


escerl.ined d!e io oenes alone
5

Geneiic siudies are prob ematic and unreliable because genes do nol p.edicl
the facl or time oi thei. occurrence or severity of dlseases with any degree of

cenes do pray aj?4 in

9!jrq!q!!9!e!!r999!99!r n addition. there cannot be a constanl pattern of disabiliiy as.ribed lo any one
rson or time as lhe condiUons tend io fluctuale beyond any measure ol

0l

These complexit'es and complications thus show thal we cannoi


study ofgenes lo comprehend his]ojj- or to shape social pollc

The sludy ofqenes re.ders race differences

1o

be irrelevanl or minuscule

Race is a socialconsiruci miherthan a biologicaldifference


2

Genetic sludy actually shows us thal we are more alike biologically than we
3

think.
ll is difficull to eradicale social noms on how racl difierences are.ooted in
our oenetic make uD

Posgible Value of Gene

Benefits)

lVedicalBenefits
Gene study woLrld reap ce'lain bnefils in the medical sector for instance, in terms
of genelic tesiing ior disease prcvention Genes ideniiJied as those responsible for
cerlain diseases such as hearl drsease or breast cancer can be located within the
DNA sirands ot individuals cluring genetic lesling Thls helps lo wam people of the
irnpending health risks and allows lhem to take concrele measures lo avoid or
delay developing the diseases

Economic Prooress

The study of genetics prornises lo create new markeis and jobs especElly
perlaining to the research andjevelopnrent irduslry. ln Singapore, the biomedical
sector has been emphasised so as to make Singapore a biomedical hub This in
turn would generate g.ealer rcvenue for Singapore as medicaltourism would be

Slimulates Discussion on Ethics


Given lhe conhversjal nalure ol qene sludy, lhis would stirnulaie and create a
piattorm for invigoting debales or discussions on the pros and cons of uUlising
gene siudy to determine,or ireat cedain nredical conditions. ln Singapore lhe
Bidelhics Advisory Commitlee was sel up lo ponder over such elhical issues This
also acts as a sprlngboard tor fudher discussion amongst lhe citizenry

Eradicates Racism
Gene study would clarit misconceptions aboui racaal difierences as shows us
lhat we are biologically akin to one another. This is especially important
'l
in a

country like Singapore. amidst the modern socieiy

we live in where

raciaJ

diffrences have led to 9olatile situalions in the world.

Prevenlion of PsvcholoqicalDisorders
Genelic tesling has also promised 1o help identfy cetuin genes which purpodedly
cause psychological behaviours such as addiction lt would hlp to idenlify ii
children have cerlain genelic predispositions to abuse drugs and alcohol

6t

,l

Fosters Sense of Belonaino


People are cu.renily using DNA tesling lo irace their ancestry lhus enabling them
to develop a sens of identit and fosler deeper lies wilh their racial groups. This
also calses socal perceptrons of cial difterences lo abate as results often
indicate lhai each person has an ancestry coniaining ot various races rather lhan

Harmful Effects of Gene Study


Economic Conslrarnts

With genetic iesting becoming more prevalenl, health insurers may


genelic tesling lo bar cena n individuals who have been idenlited to
geneswh (h may caLse dspdse from pur'h"5i g insu anre
ln addltion, geneiic lesting is expensive and may nol be accurate
merely based on prediciions

Breeds Preludice

Although racism may be eradicated due to gene study, il may erstwhile resull in a
new form ot prejudicei genetic discrimination in which people are discriminaled
against due to the genes they have. fhis wouid be again, a new form of a
danqerous social construct which may prove io be haza.dous io social stab ity.

ln addilion gene siucly may link susceptibildes lo certain dlseases or undesnable


genelic dispositions wilh certain races, which may result in even fudher

PsVcholoqical Paranoia

Gene Sludy may resull rn psvcholoqical pamnoia as people may develop fala islic
or defeatist mindsels after rcceiving negalive results in genetic lesting. This may
lead io an ovecdependency on genetic iests.

Ethical Conside.ations
There may be abuse of geneiiciesling which would lead eihicalconcems such as if
parenls were to choose designer babEs based on genelic folmulalions o.
compositions Ihis may also iuelracialdilierences as parenlswould also be able lo
choose the skin colour of lheir offspring or genetically determine oiher
cha.acterislics closeJy associaled to race

6f

Ml Mid-Yeor 07 Poper I

"Punishment always the right solution to stop crime." To what extent do you agree?

Can the education system ever meet the needs of the young?

"Science is for the mind, literature is for lhe soul." How valid is this?

How necessary is adversity in nation building?

"The basic things are

We have more information but less knowledge. How true is this of the Singaporean society?

Discriminalion starts trom the privileged ;n our

Old isgold How true is this?

The worst kind of crime is crimes against the environment Comment.

10

There is heavier responsibility on the media to censor than ever before. Discuss.

'11

Maierialism is the new reliqion. Comment.

12

Do the arts today represent reality?

lie

best th'ngs in life." How far do you agree?

"*i"ty.

Do you agree?

)a

Ml Mid-Year 07 Paper 2

ln todays world, mulli-elhnic slates are the norm The tradilional nalion stale, where a
dlslinct national group corresponds to a lerritorial unit, has become an endangered
species Globalisation and the increasing movement of people across borders lhreafen lo
krll ofi the nation-slate once and for all However, some ml,'ths resist reality, and majorily
or dominant clritures in countries around ihe world still seek to impose their identities on
olher groups with whom lhey share a lerr ory
Altempts to impose uniculturalism in muiti ethnic environments often come at the expense
oJ minorily righls. To avoid marginalisation, minorilies oflen inlensitr/ their efforts to
preserve and protecl lheir idenlilies The hardening of opposing forces assimilalion on
the one hand and preseruation of minority dentity on the olher can cause increased
intolerance and, in the worsl case, armed eihnic conflict. ln such cases and in order lo
prevent escalalion, lhe proteclion and promolion of minorty rights is essential

1A

Even though the evenis of the twentieth cenlury have lalrght us to think ofthe term ethnic
contlict as one word, lhe two concepls do not have to go hand in hand Thal is, elhnic

conflict is not inevitable in mulli elhnic slates Jusi like a mosaic arl piece; where a
plethora of individual tiles ot contrasting textures and colours are inskumental 1o ihe

15

overall composition.
Good governance plays a vital role in involving minorities in socielies and protecting their
rights and interests. Through recognilion, dialogue, and participation, alllhe cilizens ol a
diverse society can form a greater understanding of one anothefs concerns. The media 20
and educalion have imporlanl roles lo play in this regard, as do political representalives
and communiiy leaders.

Although no counlry has a perfect record on minority righls, a couniry like Finland for
example, has worked hard 10 iinplemeni legislaUon in order lo promote good ethnrc
relalions among its populaiion The Swedish'speaking Finns are lhe largesl minorily in 25
Finland al 5 71 per cent of the population. The status of the Swedish speaking Finns is
exceptional compared to thai of other naiional minoriiies, due lo the fact thai Swedish is
in addiiion lo Finnish, an offcial language of Finland. ln recent years, the government hds
redoubled its efforls lo sellle the queslion of land ownership by the Sami, lhe indigenous
people of Finland. Finnish, Swedish or lhe Sami language is laught as the mother tongue 30
of lhe student, and under lhe new legislation, children who reside in Finl?nd permanenlly,
lhus including immigrant children, have bolh the duly and the right to go to comprehensive
-, hool
Other posttive actions taken by states include: legislalive measures that inlroduce higher
maximum penalties for Fcially motivated crimes; lhe use of ethnic monitoring to ascertain 35
the number of persons of particular ethnic and nalional origin in various kinds ol
employmenl and lhe setling of targels to increase the employment ol persons ot minorily
origins in fields where they were under-represented; the eslablishmenl of new advisory
bodies on maiters relevanl to combating racism and lntoleralce, including the launching
and implemenlalion public awareness campaigns intended to prevent racial discriminalion 40
and increase tolerance, and the establishment of human rights institutions and
ombudspersons lor ethnic and racial eqlrality.
State aulhorilies need 1o ensure that minorities enjoy the lundamental righl io equality,
bolh in wriflen legislalion and in society at large. The roles of local government, civic
organisaiions and NGOS are imporlant in this respect Police, prosecutors and iudges 45
need to be
of what conslitutes racial discriminalion and racially motivated
cr rnes and in some cases, changing the composition of police forces to better reflecl lhe
multFethnic communities lhey serve may be appropriale lt is also incurnbent upon

7/

3.
minonlies to rnlegrale themselves inlo lherr communilies. Other recommendations inclllde

mon(orrnq hale sp-ei h. promo|ng ampowermenl rhrough eclurdlion. and ensur,ng

50

adequate housing and access 10 heallh care.


During the Warsaw meeting, experts emphasised thal regional cooperation remained key

to

combaling discrimination

of

minorities. Special attenlion was paid

to the

rapid

proliferalion of hate speech, hale crime, and hate sites on lhe lnlemet. Experts agreed
that many counlries in the region ofien did not acknowledge thal racial discriminalion
existed and seemed to consider il only a problem in lhe United States and South Africa.
The World Conference, experts said, should help ensure lhal internaiional and regional
minorily rights instrumenls were publicised and understood by disseminating information
;n relevanl languages and conducting public educalion campaigns

55

ll is when staies lack the foundation for protecling minorjly rights or governments aclively 60
encourage intolerance for minority groups thal conflict-ridden environmenls ensue As
tensions involving nalional minority issues iniensify, disenchantment with one's
governmenl can evolve inlo conflict siiuations ln the pasl ten years alone, ethnic conflicts
have plagued a handful of countries such as Rwanda and Burundi, the former Republic of
Yugoslavia and more recenlly, lndonesia, East Timor and Fti. lt is tragedies like these lhat 65
compel the inlernational community to encourage a dialogue between minorities and
qove.nmenls within all societies.

t0

1',t

12"

Almosi three years afler former President Suharlo's dictalorship collapsed, lndonesia's
problems with its minority communiiies are growing despite lhe benefits brought by
democracy The new governmenl faces separaiisi activities in the provinces of Aceh and
lrian Jaya, East Timor has yei to recover from its vote for independence, and ethnic
violence has recently erupted in ihe lndonesian section of Borneo. Human rights groups

70

eslimate that between 3,000 and 4,000 people died in separalists and ethnic violence lasi
year in lndonesia and thal more than one million people are now homeless because of
these conflicis Although there have been new effods 10 devolve power 1o lndonesia's
reqions, ihe Governmeni has yel 1o implement pro'minority policies, which means there
may be more trouble ahead

75

The proteclion of minorily ights and the prevention of elhnic conflict was also diseussed
at the Oclober 2000 Regional Seminar of Experts for Africa, held in Addis Ababa,
Elhiopia. The Seminar agreed thal greater atlention must be paid to the economic 80
problems that give rise lo outbursts of ethnic conflicts in Africa. lt turther recognised lhat
lhe realisalion of economic, social and cultural rights and the right to development is of
crucial relevance to the preven$on of ethnic conflicts in Africa or any other region. lt
therefore encouraged "full parlicipalion in political life ior all, non-discriminalory treatment
85
of all regions and ethnic groups within a country, and respect for the rights of minorities."
Ultimately, implementing policies ior minorfties should be done 10 iosler long-term stability
and not just to appease the international community. Through dialogue, all parlies can
share their concerns and work toward iinding a common ground As the High
Commissioner on National lvlinorilies for The Organizalion for Securily and Co-operation
in Europe {OSCE) has said, "Accommodating minorily interesls should not be interpreted

90

as political correclness or pandering to special inleresls groups Nor should it be


diminished lhrough tokenism or shori term concessions lnstead there should be a
genuine commitment to protecl the idenlily of national minorilies and create condilions for

the promolion of that identity." The rnain objective is to prevenl conflicts in multi ethnrc
siales before lhey happen. As the Secrelary-General lo the World Conference has
repeatedly said, "At both lhe human and lhe financial level, a cullure of prevention is more
communiiy can
beneficial than a cullure oi reaclion". All slates as well as the
'nternalional
an inclusive approach to
work together 10 increase dialogue among parlies and creale

)r

Ml lvlid-Year 07 Paper 2
'1 Whal are the faciors thal put'the traditional nalion-siate' (line 1) at risk? Use your own
words as far as possible. [1]

2.

Explain why the aulhor believes'the protection and promolion of minority rrghts is
essential' (line i2) Use your own words as far as possible l2l

3.

ldentify the metaphor used to describe multi ethnic societies What is the writefs
intenijon in using it? Use your own words as far as possible. [3]

4.

From paragraph 4, what ingredienls are necessary in'a diverse society' in forming a
grealer understanding of one another's concerns' (line 19 20)? Use your own words as

far as possible.

[11

5.

How has democracy an lndonesia failed to resolve the'problems with ils minority
communities (line 69)? Use your own words as far as possible. [3]

6.

Whal does lhe word 'outbursts' (line 81) suggesl aboLrt the ethnic conflicts in Africa? I1l

7.

Explain why "a cullure of prevention is more beneficialthan a cultuae of reaclion" (line
96-97). Use your own words as far as possible. [2]

B.

Give the meaning of the following words as they are used in the passage. You may
wflte the answer jn a word or a shorl phrase.
ascertain (line 35)
fundamenlal (line 43)
ensue (line 61)
devolvA (line 75)
pandering (line 91) [5]

Summarise the measures taken by the states to protect minoriiy rights. Using material
from paragraphs 6 8. write your summary in no more than 120 words, not qounting the
opening words which are given below. Use your words as far as possible.
Thg stdte\ protect nlinorit!" tights h),...

l8l

10 ln

hrs conclLrsion, the wnter stales thal ,l rs


for staies to make a comrnitment to
equal treatment of all persons regardless of 'mporlant
their racial or ethnic origin .
Using arguments from the passage, what are some of the challenges governments today
face in observing such a commitmenl? How can lhese chalienges be overcome?

/3

Ml Mid-Year 07 Paper 2 Answer Scheme


From Paragraph 'l
Question 1
What are the factors that put'the tradjtional nation-state, (line 1) at risk? Use your own
words as far as possible.
I1l
From the Text
- Globalisation and

Possible ParaDhrase

The opening up of geographical/national


boundaries and markets worldwide (1/2)
and

-the increasing movement of people the growing numbers/ migration of


across borders threaten to kill off the people/individuals travelling to other
nation state once and for all.
countries ('1l2) have caused the t.aditional
nation state to be at risk of becoming extinct

From Paragraph 2
Question 2
Explain why the author believes.the protectron and promotlon of mtnonty rights is
essential' (line 12). Use your own words as far as possible.

I2l

From the Text

Possible Paraphrase

-to avoid marginalisation,

minorities
often intensify their efforts to preserve
and protect their identity The hardening
of opposing forces assjmilation on the
one band and preservation of minority
ideniily on the other
can cause
increased intolerance and, in the worst
case, armed ethnic conflict. ln such

in order to prevent
escalation. lhe prolectton and promol,on
cases and

of minofity nqhts becomes essenttal


to avoid marginalisatioi

They are esseniial because:


.

+il ensures minorities are treated fair,y or


favourably (1/2) OR
not torcibly set" aside/ be isolated from
society 11/2)

Note: "discriminated,

againsl

''sligmatised" not acceplable

"increased intolerance,,

+it

ensures greater

behaviour

,'ostracised

accommodating
accept

and willjngness to

differences (1/2)
]

-"armed ethnic conflict,'

+it deters the use of weapons during racial

c]2007 Mllnnra nsritulc


PU2 GP MY E:AMiN"ON

1+

disputes/clashes (1/2)

{o prevent escalation...

+it puls a stop to growing tension (1/2)

Note: Answers should be phrased


sitive manner
From Paragraph 3
Question 3
ldentify the meiaphor used to describe multi ethnic societies. What is the writer's
intention in using it? Use your own words as far as possible.
I3l
From the Text
Take for example a mosaic art prece

Possible Paraphrase
-The metaphor is the mosaic art piece {1/2)
-just like lhe mosaic art piece is made up of
many different components (1/2)

-where a plethora of individual tiles of so a.e the different races integral to the
contrasting textures and colours are nat'on (1/2) because they come together to
instrumental to the overall composition.
form a cohesive society (1/2)
OR
/can co-exist harmoniously
each indivjdual or racia, group is integral
{l/2) to the existing social makeup/fabric
(1/2) of the nation
OR

A vibrant sociely is made up of


racial groups and ind,viduals
cohesively integrated (1/2)
Flhnrc conflict t5
elhnic states

not

aevttdble tn

;altt-

different

(l/2) who are

writeis intention is to OeUunt/retute {11


the general perception that there wi,l always
be conflicts in multi-racial/cultural states
-The

From Paragraph 4
Question 4
From Paragraph 4, what ingredients are necessary in ,a diverse society, in forming ,a
greater understanding of one anotheis concerns (line 1 9 20)? Use your own words as
far as possible.
ljl

02007 Miennia isrillr


Pll2 cP [lY ExamnatDo

7d

Note: Any of 2 of the 3


ints.
From the Text
(From Line 19

Possible Para hrase


The ingredients arel

20):

Through recognition, dialogue,

and

participation, all the citizens of a diverse


society can form a greater understanding
of one anotheis concerns.

-acknowledgement of rights

(1/2)

OR

open discussion/communication (1/2) OR

-involvement
processes (1/2)

politica l/social/civil

From Paragraph 10
Question 5
How has democracy in lndonesia faiJed to resolve the 'problems with its minority
communities' (line 69)? Use your own words as far as possible.
I3l
From the Text

Possible ParaDhrase

...the new government faces separatist the recently estabrished democrattc


activities in the provinces of Aceh and state/state authority contjnues to
lrian Jaya...
encounter problems/troubles with groups
wanting autonomy/self-rule

..has yet to recover from its voie for -continues

(.1

to be

plagued by
strife/struggles/confJicts despite a majority
agreement for self rule (1)

independence..

ethnic violence has rec@nt'y erupted in -fresh racial fights and .iots have broken
the lndonesian section of Borneo
out in Borneo (1)
Note: 1/2 mark awarded lo response wlth
mention on 'additional ethnic confticts'
From Paragraph 1,1
Question 6
What d-ogs the word oLrtbursts (line 8 l ) suggest about the ethnic confljcts in
Afflca?
Nole: Bonus mark to be given (it applicable).
From the Text

Pa

I11

hrase

-suggests in unplanned/ sudden/unexpected


to reactive/spontaneous (1/2) AND
many /numeaous /widespread/prevaJent
'112) multi racial
clashes
to

From Paragraph 12
Question 7
02007 NirllenDi. nslituro
Ptr2 GP MY Fr3nin:iion

76

Exp,ain why a culture of prevention is more beneficial than a culture of reaction" (line
96 97) Use your own words as far as possible.
t2l
From lhe Text
Line 96

Possible Paraphrase

-A prevenlive culture is more humane

971

respecting the rights of individuals. (1)

.. at both the human and the financial


level, a culture of prevention is more
beneficial than a culture of reaction

..

as well as making more

sense

in terms of

economtc

cost-savings when
bloodshed and damage are minimised (1)
Question

Give the meaning of the following words as they are used in the passage. You may write
your answer in one word or a short phrase
t5t

Word/phrase
ascertain (line 35)

fundamental (line 43)

ensue (line 61)


devolve ([ne 75)

pandering (line 91)

l mark
-determine
-establish
basic
elemental
-foundational
-follow
-arise
-happens
imtr9qE!9ly_949l
-decentralise
-delegate
kansfer

th matk
-coniirm
-make sure

acceding to
others' wishes for
self gain or
benefits

FromParagraph6-8
Question

Summarise the measures taken by the states to protect minority rights


O2007 Millennia rnsllule
PU2 GP MY Fxaminal on

0 mark
ensure

-underlyinq

hand to
pass to
-entrust
-qive to
-accommodating
-specially
cstering
-acceding
-qivinq in

Using material from paragraphs 6 - B, write your summary in no more lhan 120 words,
not counting the opening words which a're given below. Use your words as far as

possible.

IBI

The stales prctect mino ty rights by ...


From the Text

Possible Paraohrase

l.legislative measures that introduce -by passing harsher laws/punishment (1/2)


higher maximum penalties for racially on offences relating to or against ethnicity
motivated crimes (line 34 35)

2.-the use

of

(1t2\

ethnic monitoring to -to delermine in order to keep track the

ascertain the number of persons of ratio/makeup of different races in numerous


pari.cJlar ethnic and naIonal origin in jobs (1)
various kinds of employment (line 35-37)

3.-the setting of targets to increase the to establish bigger quotas (1/2) in different
employment of persons of minority professions where certain racial groups are
origins in fields where they were under lagging/iacking in numbers ('112)
represented (line 37-38)
advisory {he setting up of organisations (1/2) to
bodies on matters relevant to combating deal/resolve with issues relating to ethnic
racism and jntolerance (line 38 39)
discrimination and prejudice/bigotry (1/2)

4.{he establishment of new

-the establishment of human rights -and oflicial organisations/committees to


institutions and ombudspersons for handle issues relating to civil liberties (1/2)
ethnic and racial equaltly (line 41-42)
and to appoint advocates to ensure fair
5

treatment for all (1/2)


6. the roles of locaf government, civic
organ'zations and NGOs a.e important in
this respect: (line 44-45)

{he functions of state authorities, non-profit

+more aware ofwhat constitutes racial


discrimination (line 46)

+gaining heightened knowledge of the


factors leading to ethnjc stigmatisation/

bodies are signjficant in:

segregation
+changing the composition of police
forces to better reflect the multFethnic
communities (line 47)

+altering the makeup of the police forces to


mirror/improve the representation of the
multi-racial groups in society

+monitoing hate speech (line 50)

+supervisinq incidences

of

racist remarks

(1t2)
+promoling empowerment through
education (line 50)
O2007 Millennla

+encouraging people io fight for their rights


through formal learning (1/2)

risirule

18

is

sure there
sufficient
accommodation and availability of medical
services (1/2)

+ensurinq adequate housinq and


to health care (line 50 51)

+makinq

+regional cooperation (line 52)

'

working logelher

wilh

ne,gl-'bour'rg

countries (1/2)
+ensure that international and regional
minority rights instruments were
publicised (line 57,58)

+rnaking sure that global measures for the


preservation of rights of people who do not
belong to the majority were promoted widely
(1t2)

+understood by disseminating
information in relevant languages (line
s8-59)

+presented in different languages so that


they can be comprehended easily (1/2)

conducting public education campdrqns

(line 59)

+carrying out planned activities (1/2) to instil


awareness of minority rights amongst the
masses (1/2)

Note: Any

-6 pornts ltsted

under 6) wrll be

qqc-gplgd

Question 10
Application Question
ln his conclusion, the writer states that 'it is important for states to make a commitment to
equai treatment of all persons regardlpss of their racial or ethnic origjn'.
Using argurnents from the passage, what are some of the cha,lenges governments today
face in observing such a commjtment? How can these qhallenges be overcome?
ln your answer, give your own views and the experiences which have helpecl you to form

them

From Paragraph I

igl
Possible Response
Challenges governments face:

'Globalisation and lhe increasing Globalisation has made il very difficult to instil
movemenl of people across borders Joyalty in people. I\Iovement across borders means
lhreaien to kill off the nation state once lhat people who make up ihe human capital
and for all.'

essenlial for a country's growth, may not stay. Th;s

is a problem for many developing nations and


those lhat face constraints such as limhed
resources like tn Singapore.

How to overcome these challenges:


11 is importanl 1o make a country a warm and
inviting place io live in ln this way. not only do we
relain our precious human capital, we also attract

t
PL12

e.nq
CP MY

-q3! lSlglglf

tatenls who

blng in

much

Eraoinatoi

)9

needed experlise.
The Singapore governmenl achieves lhis by:

Offering atlractive remuneration packages


which come with benetits for spouses and

From Paragraph 2:

children as well
Oflering benefils such as workfare bonus 10
look afler the welfare of its people
Acting as an example in promoting a more

gracious and compassionate sociely thal


does nol discriminate againsl any race,
religion, language or gender E g The
Singapore governmeni has even recenlly
allowed for ihe hiring of homosexuals in the
civil service
Challenges governmenls tqce:

"The hardenang of opposing forces - ln couniries like Australla and UK, all potential
assimilaiion on the one hand and immjqranls musl lake a lesl lo a\cerlain lheI
preservalion of minorily identity on the proficiency in English. ln some countries, poteniial
oiher

can cause increased intolerance ,mmigranls are even lesled on lhetr htstory.

and in the worsi case, armed conflict.

This is to ensure lhal foreigners are nol excluded


from mains{ream society and are successfully
assimilaied

However. torcigners slill

do end up

somewhat

isolaled lrom ma,nsteam socrely tn some pla(es


such as London and Paris, migrant communities or

foreigners are seen by the roadsides or train


sialions, selling pjrated wares, or at the park,

speaking lo each other in their nalive tongues even


lhough they are proficient in English or French

Howlo overcome these challenges:

"..the protection and promolion

minority rights

become essenlial.'

The governments have put in pjace a system of


affirmalive action to assimilate or reduce lhe

marg;na|salion

ol migrant

communrttes or

foreigaers.
E

g. ln Canada, miqrants are provided with fiee

English classes. Hiring of foreigners is made easy


as long as there is proper documenlalion.

ln Singapore, each major race is protected in the


.onslilulion Speeches tn parl'dment can bp g,ven
in any of the four malbr tanguages according to the
speakers' preference. Every beljever is allowed to
actise his/her relioion
PU2 GP MY EXAMINAIiON

jD

g. Some roads are closed,for

Thaipusam and

Chinoav Drocessions

paragraph
.Although

no counlry has a perfecl record


on minorily rights, a counlry like Finland
has worked hard .. to promote good
ethnic relalions amonq its population.

Challenges governments face:

As the aulhor has rightly pointed out that

the

corrmlnenl ro equal kealmenl of all pe,sons is an


arduous one as reflecied in the phrase 'no
country". Such a view is a reasonable one and it is
indeed one ol lhe grealesl challenges governments
ln the example about Finland provided, it is evident

thal governmenls need to do more than merely


a1lempl to inlegrale different races and ethniciiies
logelher. There is a lol more that has to be done
simultaneously such as the proteclion of lhe rights
of each ethnic group in order 1o achieve equilable
treatmenl for all.

E.g. The Ame.ican slory tells us that, despite


efforts lo provide lor equal treatment for all, the
cohscientious application and observation of such
a commilment can be difficul{ to follow.
How to overcome these challenges:

From Paragraph 6
"including ihe launching and
implemenlalion of public awareness
campaigns jnlended to prevent racial
discriminalion and increased

As seen trom the passage that it is necessary


for governments to make a qenirine effort to
implemenl legislalion that provides recognition
to the ethnic groups involved and nol just
acknowledge the presence of majorily groups,
just like what the Finns did

lmportant markers of elhnic groups include


language, access lo educalion and

employmenl, privacy are some examples of


faclors lo consider when allowing for equal
trealmeot.
Challenges governments lace:

. The

author, again righlly highlighied thal


legislative measures are just part of the
'package' ihat would pave the way for grealer
respF(l lo be shown to all elhnrc Aroups rn a

socieiy.

The challenges governmenls loday face are


the manifold programmes that have to work
hard ir hand so ac 1o d, hteve d maximLrm
level ol elhnic tolerance and equal lrealmenl
tor ali, which includes both hard' measures
such as passing of laws and soft measures
slrch as education programmes ihat promole

9t9919r!!99rqe!q!94!Jslgleryc
G]2007

P!2

Nf

i r-cnnia rnsr,rule

CP MY Eian nal

oi

f/

Such
polilical eflorl and

measures require much

How to overcome these challenges:


Governments need lo ensure or be more aware of

the elfectiveness of ihe measures that they have


proposed. This can be achieved by making sure
lhat lhere is reasonable represenlation of lhe
differenl elhnic groups and conducl regular checks
and reviews of existing laws This would aid in
making sure lhal measures are relevant and

From Paragraph 7
Siates aulhorities need io ensure that
minorities erljoy the fundamenlal right lo
equaliiy, both ,n wrillen leg'slation and in
society at large.

9[ec!v9

as far as possible.

Challenges governments
Hale crimes are hard lo eliminate, especially if

the police force is made up of the majorily.


Law enforcement could be difficull because
racism mighl exisl wilhin the police force ilselt.

New governmenl may face difliculties


promoting empowermenl through education
and also ensuring adequate healih care. New
governments may iace financial slrains and
may nol have enough processes in place to
carry oul their plans.
How to overcome these challenges:
Educatinq the public lhrouoi) public awareness
programmes and campaiqns
From Paragraph
Paragraph

EnsLrring disseminalion of information of

minorily rights

Chailenges gov;fiments facei

This maybe difflcult as developing countries and


coufllries currenlly in slrife may nol have the
faciliiies to disseminate lhis informalion. Facilities
such as inlernet and also dio maybe under the
conirol of ruling parlies lhal may not want such
information

How to overcome these challenges:


Use the media effecl ir{ely

O2o07 Minnn a luslitule


P(.r2 GP [iY Examrnation

&1

Challenges Faced by Governments Today


From Paragraph 10
".. lndonesia s problems with its minority Despile lhe fact thal democracy is praclised in
communities are growing despite the some nations, the issues of thic violence amongst
the minorilies are still teelhing problems wailing to
benefits brought by democracy...
be resolved. These riols and ouibursts of violence"
'' between 3,000 and 4,000 people died resulled rn innolenl lo\s ol lves and ddmagp ir
in separatisls and elhnic violence last property
year in lndonesia....more lhan one million
people are now homeless...
The Governmenls have lo dealwiih interesl groups
and aclivisls who have a differenl polilical agenda
"..lhe Government has yet 10 implement that go against the stale authorilies
pro-minority policies, which means ihere
may be more trouble ahead...'
These challenges can be overcome by:
Elecling a strong Governmenl thal is keen lo
implement pro-minority policies and other
legislative measures lo prevent further racial
conflicts.

Example:

ln the mid-1980s, children were caughl in lhe


elhno-national and religious civil war in Beirui, the
capitalot Lebanon, whose population is aboul two
ihirds Muslim and one fourth Chrislian; wilh each
faction fotther divided into sects In 2005. in view of
global p.essure, lensions receded in Lebanon and

the Syrian occupalion forces were forced lo


wilhdraw. The new governmenl proclaimed ils
commitment io help mainlain peace and provide
s{rong representation for all elhic and religious
factions.

Challenges Faced by Government Today

As mentioned in the passage, there will be more


unforeseen consequences it minority riqhts and fair
treatmenl are not addressed

From Paragraph 11
"...greater atlenlion musi be paid to the
economic problems that give rise to
outbursts of ethnic conflicls in Africa. "

lnfesled wilh elhnic disharmony and tensions, il will


be hard tor ihe Governmenls to sleer ils economy

A sluqqish economy and hioh rales of


''...economic, social and cultural rights unemployment among lhe minority would resirll in
and the righl lo development is o{ crucial sudden, uncontrolled and fatal racial riots where
rdevance 1o lhe prevention of elhnic lhe authorilies cannol avert
conflicls in Africa or any other reqion . "
Other underlying faclors such as marginalisation
''...encouraged full participation in and abuse of righls againsl the minoriiies will
political life for all, non-discriminalory further. hasien lhe siale of racial discord in the
lreatmenl of all regions and elhic groups counlry
within a counlry, and respecl for the righls
of minoriiies
These challenges can be overcome by:
Prolecting ihe economic, social and cullural
interesls of lhese minoriiies by working closely wilh
the non
r!t19 rt!91!I99rEe1!Sl!l!q9!
PU2 GP MY Eraminalioi

10

dJ

The aclive involvement of the M3Os can help lo


prcmole their shared interesls and ideals in order
lo influence the policies of state governmenls.
Example:
NGOS such as Amnesty lnternational, Save the
Children and Global Youlh Connect work with the

govemmeni to regulate certain issues sUCh as


human rights; and lhis is evidenl in autocralic
governments where NGOS work hand in hand with
the authorities 10 reduce the repression of minority
racial in ihe communilies.

ll is questionable if any one counlry can


".. encouraged'full participation in political lite for
all, non discriminatory lrealment of all regions and
elhic groups within a counlry, and respect for the
rights of minorilies..." it is indeed a challenqe for
the governmenis today lo exercise equalily and
meritocracy in its policies for all ils people
regardless of race and .eligion.
Example:
However, in my opinion, Singapore has done it with
pariicipaiory state policy where every
Singaporean is given a chance to voice out his/her

concerns. There

are many channels

o{

communication for differenl ethnic groups to


contribute their opinions.

For inslance, minisler visils, meel the resident'


gatherings and dialogue sessions are great means
lo get 'up close and personal' wilh the government
officials
Singapore's nalional pledge encapsulates ihe
essence of respect and sovereignty of rights given

10 every Singaporean so that future generations


are given the liberty lo actualise their dreams.
The close monitoring and heavy involvemenl of the
Government in the economy, social sectors, media
and education "ensure lhe inlerests of
Singaporeans

Chatlenges Faced by covernment Today


From Paragraph 12, we know it is hard for the
governments oi loday io find a common ground
and commitment to ensure the minority interesis
are protected as there might be a clash of
international agenda wilh lhe global communily

t
02007 Mr reniia nsllule
t,u2 cP MY Eramination

It coud qe an easy way out io focus on short term


11

t4

gains than to solve the key issues of concern to


prolecl the culiural uniqueness of these minorities,
allowino them to flourish rn lhe fulure.

These political blind spots will eventually result in


elhnic violence on a massive scale.

From Paragraph 12
implemeniing policies should be done
10 foster long ierm stabilily and not just to
appease the internalional community... "

''

Example:

On Seplember 11, 2001 the Al Qaeda terrorisl


network aliacked ihe United Siates to venl anger o{
its exlremist lslamic members against the West

'' all parties can share lheir concerns


and work loward finding a common

This lerrorist allack and global calaslrophe is a


siern warning thal ethnicity, racial conflicts and
ground...'
aggressron are isslres Ihal are nol drsaop-arinq
and becoming less imporlant. ln fact, the
"accommodaling minority interests should governmenls in the world such as the G8 nalions
not be interpreled as polilical correctness' should promole more dialoque and policies to
or pandering to special inleresi groups. resolve lhese issues.
Nor should it be diminished throuqh
These challenges can be overcome by:
lokenism
genuine
be
a
Promote an inclusive culture a culture of
concessions...there should
commitmenl io protect the identily oi prevention lhan culture of reaclion where ce.tain
national minorities and creale conditions measures dre laken rnlo prio'consideratio'
for the promotion of thal idenlity "
New policies and mandate should be established lo
".. a culture of prevention is more bring a revived notion of ethnicity sensilivily in lhe
beneficial lhan a culture ol reaction..all form of racial campaigns and public education
siales as well as the international where schools, media and the family funclion as
communily can work logether to increase positive p,atformslo dissemidaie information.
dialogue among parlies and creaie an
Nevertheless, not all countries can apply the same
inclusive approach 10 national identity .'
old slralegles for racial harmony; certain policies
have to be reviewed and localised for ils
effecliveness to lake place.

Closure
Wqshall always learn frdm history and reflecl upon
our roles as an individual member of the society we
live in

ll is disturbing lo know how elhnic conflicls


,mpact hve5 (eq Cenocide in Rwandd :n tg94
drove more than 1.7M refugees from their
homeland and lhe roving lslamic radicals
slaughtered 2M in Sudan).

Though these chailenges ale not easily overcome,


lhere is a need lo minimfse lhese elhnic sirile io
bring back world peace and order in the future.

12
PLr2 CP lL,lY Examnalon

t3'

Descriptors

Grade

79

O
Q
fl
E

4-6

I
O
O
D

Systemalic returenc 10 the requirements of lhe


question with evidence oia balanced lrealment.
Evaluation is very sensible, well-supported and
developed io its logical conclusion.
Explanation includes elaboralion and refercnce to
personal insights and interpretaiion.
High degrce ofcoherence & organisation.
Covers basic requirements ofthe question but not
necessatly a balanced lreatment.
Evaluation is not always convincing and tends to
be superficial wiih limlled development of ideas.
Explanation is not as through as lhat in an A

Coherence & organisation are not as sharp or


systemalic as thal in an

O
O
B
c

1-3

c)2007 [4illennia rnstilute


PU2 GP rVlY Examinalion

O
O

script.

Fails to address the given requirements of the

Explanation demonstrates limiled relevance and


developmenl ot ideas.

Evaluaton is non-xistent. Ihe answer is d mere


summary or restatement ot lhe text ratherthan
an evaluation ot it.
lnconsislency in the argument is evident.
Coherence is in question.
Higher incidence of misinlerpretation of the te)d.
There is evidence of inaccuracies.

13

8b

NJC l\titl-Year 07 Paper

"Discrimination in any form will always persis{, regardless of measures taken to counter
Discuss the slatement with reference to one form of discrimination

More needs to be done in Singapore to promote inlerest in ihe Arts Comment

Discuss the impoiance of marriage in today's sociely.

How important is il for young people to be aware ol current affairs?

Teachers will never be replaced bylechnology Doyou agree?

The lnternet has proven to be more a curse lhan a t,lesslng. Do you agree?

The developmenl of a counlry depends on its leaders, not ils citizens. ls this a fair comment?

How far do you agree thal lhe mass media needs lo be conlrolled?

ii"

"ln our counlry, lhe role of the domeslic helper has extended beyond mere housekeeping."
Analyse the view Presented.

10

All competition is detrimentallo genuine education. Discuss.

1l
'!2

Good sporlsmanship is no longer a realily Discuss.


Which is more wodh saving: a foresi or a starving child?

O Nar o..1 Junror Colleg 2007

8J

NJC Mid-Year 07 Paper 2


Passaqe A
When we talk about evil in lhe comfortable herd, lhe ethical core of the whole issue
is whal psychologisls would call passivity in groups o. the bystander etfecl.

The byslander effecl is watching some evil lake place, bul since we are watching
with others who are watching, and no one seems !o be doing an],'thing aboul lhe evil,

we go on walching and doing nothing about il lnstead of consulting our own feelings
aboul what we do, we iake our cue from lhe olher byslanders. They are not doing
anyihing. Therefore, we also do nothing lf something needed lo be done, somebody
would have done il
No wonder we lake so naturally to waitinq around tor the word. Our childhoods were
spent in groups. Theae we stood, smaller lhan the o1he6, to see what was coming.

We have 10 be bystanders while oLlr parents figure out how 10 load the car'
Sometimes the leaders ol the group were involved in evil, not elsewhere bul againsl
us, but lhe family leaders were what car ed lhe day. Whaiever simple orders of
papal bulls, whatever bealings or blessings, come down from our parents are seen
by us children not as just or unjusl but as reality, the way it is, the way it's done, the
way il always willbe.
Should one judge one's parents? I feel disloyal because akeady by the age of ien I
have lhese two moral values: staying sober and keeping promises. My parents have
failed in carrying ou1 these lwo vir{ues But I myself am really living at Slage llloyalty lo my particular tribal group, to my parenis. So I leave ofl any furlher mention
of the picnic. I will delav commilling myself lo judgment. I will hang around some
more and see how lhings develop. I slop planning whal to pul on rny wriling pad I
had ihoughl lwould write something beautiful, but something else goi in the way of
il so I stop
PeoplF who have bpen possve gFnerally havenl gol a qood sens; or tme lhey
don't distinguish behteen what is happening to them now, at this very minuie, what
has always been happening lo them, and probably will happen again in the fulure
Since they are out of the loop of planning lheir own lives, their minds don't categorize
time segmenls in the way minds must il they want to feel and be aciive. "l will do this
now, that later": lhat is the conversation of an active personalily, not someone who
doesn't expect lo lake control. As children, most of us spend years hovedng, happily
or not. at the circumference of people in authority over us.

10

15

20

25

30

Pulling oneself inlo wakefulness when others look dazed does not come natllrally
For one thing, passiviiy is in the shorl ietm a thousand limes more practrcal lhan
slepping forward to takeaction. One attracls attention from lhe enemy ifone moves:
if one freezes, lhe enemles in your line of sighl conlinue killing whalever or
whomever they have begun to kill and do not suddenly swing their atienlion to you A
lesson oflhe most ancieni part of our brain is to slay slill and nothing will get you
One needs learning from another part of the brain td overcome natural passivily. One
needs a molivalion more passionale than self-interested Practicaliiy. When a woman
is being knifed to dealh the praclicalthing ior those watching is todo nolhing. One of
the most difficull lessons children learn is how to make their minds shout at lhe
pasc've psyLmlogral mu<cles inlide rhem

Thal is how children have to live. lf lhey don't develop oul of the bystanding habit
ihey become adult byslanders

40

45

A.tapledlrcn Evil n the Conlatlable Her.i bv CaratBlv

B&

Passaqe B
l\,4oral drifl is a particular and fascinaling kild ot bystander effect, a plain fear of being
disliked if we stand apart tlom group flow Let us say lhal eight people are discussing
slavery One pelson is sirongly againsl it FoLlr are strongly in lavour. Three suppose
it is OK. or maybe not: they are backya.disls; ihey have no feel for the subjecl
because il hasn t come up in their own lives, and they have poor imaginations, being
self-cenlered like N,4acbeth. The three who suppose that slavery is OK make no
menlal image of enslaving someone, or of being enslaved themselves.

Thal's the ambience of the room. The one anlislavery person makes a sirong
objection 10 slavery. People defend slavery. Others shrug. The anlaslavery person
speaks again The others keep giving one another more and more eye contact and
less and less eye conlacl lo lhis inilaling speaker Nothing makes a strongly
opinionaled speaker yield so well as being denied eye contaci. Afler len minutes or
an hour or a week or two weeks, the sometime sirong dissenter says, "Well, I

10

suppose there are some lnstances wherc slavery mighl be all righl, of coutse"
15

lvloral drift is not just bovine behaviour at a public meeiing bul a lhreat to iustice
N4oral drifl has laken place when a strong person of universal morals lloals over to a
position oI cultural relativism or lo a less pronounced objection 1o the evil in question
lncidenlally, much of the "mellowing out" that forly and f'fly-year'olds smilingly
observe in one another is simply moral drifl The conscienlious d;ssenter has gotien
t'red and lefl the huslings

20

Moral drift is a dynamic of people lalking to olher people on issues requiring


conviction. For years lthought of gtoups as being venues in which one got all fired
up aboul some project ln fact, groups tend toward more gentleness lhan firing They
damp things down io a mossy agreeableness. That is why very great poems, like
very great short stories, are nol wriiien in groups.

25

As with any human proclivity, some people are rnore vulnerable 10 herding in groups
than others. A marvellous and readable expert in this field is Dr Alice Miller, who has
given us scholarly and poignant podraits of Adolf Hitler and his father of how
psychological abuse of children gives them a poor self image thal they attempl lo
repair or replace with love of a charismaiic leader. Anothet wise counsellor about
herd menlality, especially as il operales in our commercial world today, is Tome
Kitwood. He slates lhat because modern people spend such a significant share of

their lives working in large organizations, they scarcely can recover {rom the
psychological effects in lhe time left ihem afler retirement. His is a grim view
Accepling a low-level moral code during the day is bound 10 infillrale one's off lime
judgmenl as well

The gloomiest social insight is the facl that although the ways in which we spoil our
personalities by herding slarP us in the face, we ignore lhem. These obsewations are
corroboraled by the social work theory ihat some undesiaable behaviour, such as
acting aulhorilarian arorlnd one's workplace and one's home, comes nol of present
bad feelings bui oI yeaGin, year oul habit Such behaviour will not respond, iheretore
lo ordinary psychotherapies: one needs 10 exert 'character" lhat is, willpower lo
drop the bullying habil. Such characler or willpower, however, is a quality typically
urldeveloped or attophied by life in the "low levetjustice struclure" oflhe herd

How, precisely, does the herd make moral m'stakes? l-Jow can we spot the group
dynamic as il siaris up? For il we can spol a bad dynamic, we can stop it by showing
A.iapledfron Evlnt

Ih,"

30

35

40

45

Canlodable Herd by CanlBly

87

NJC Mid-Year 07 Paper 2


Questions on Passage A

1.

Using materials from paragraphs 5 to 7, summarise in about 100 words


(excluding the words provided),:
the characteristics of passivity
how and why people fall into passivity
how passivity can be overcome
be given lot good
Use your own words as far as possible. Crcdit

wi

organisation.
One ol lhe "haracleflsliLs
tel

ol

passrvrly

rs

Questions on Passage B
paragraph
2
From
lnfer how the example on slavery illustrates lhat "moral drift is a 'lhreat to
2
justice .
t3l

from paragraph

3a

What does 'smilingly" imply about the attitude of the forty- and fifty-year olds?
t1l

3b.

Explain such an attitude.


t1l

Wilh reference to lines 16- 17, how does "float' illustraie the image of "moral
dtill"2
12)

From paragraph 4
Explain the f'guralive language in the sentence: They damp things down to a
5
mossy aqreeableness (line 23-24\
"t21

From paragraph 5
Why are some people more vulnerable 1o herding? Use your own words as far
as possible.
t2l

From Paragraph 7
7. . Based on lhe writer's closing remarks, whal can you infer about his attitude to lhe
issLre of the herd mentality? Use your own words as far as possible

t:l

Questions on Passage A & Passage B


Give the meaning of the following words and phrases as they are used in
Passage 1 and Passage 2 You may write youa answer in one word or a shori
phrase

fo

core (A, line 1)


naturally (A, line 9)

proc[vrly (8. line 26)


qloomrest {8. line 37)

atrophied(8,tine144).............................
9.

...

..

..t51

Passage A and Passage B explore the 'bystander effect' from the psychological
and sociological perspectives respectively. With reference to both passages,
discuss the extent to which the 'bystander effect' is observed in today's world?
Suggest how yolrr society can possibly dealwith this concern.
Support your answer with examples from your personal experience.
tel

q/

NJC Mid-Year 07 Paper 2 Answer

Scheme

Using malerials from paragraphs 5 to 7, summarise in about 100 words


(excluding the words Provided),:
ihe characteristics of Passivily
- how and why people fall into passivity
- how passivrty can be overcome
Use your own words as lar as possible.
Credit wrll be given for good organisatron. [9 m + 1 bonus]

One of the characieristics of "passivlty" is

g!9Eq9rE!i.ci 9I

P9 "l

(a) haven't got a good sense

They lack temporalreasonrng skills / Their


lemporal orientation is lacking/ poor

ottime/

their minds don t categorise time segments in


the way rninds must if Ihey want 10 feel and be
active. (1)

A lift of the word tine is acceptable, providecl


the cancept is elaboratecl on a lillle more e g.
''does nal have a strang canceploftime, orthe

sequence of events
''poortinle managemen!

,.

(b) lhey don't distinguish between what is


happening to them now, at thisvery minute,

They are Lrnable to differentiate lhe


occurrences taking place al that instance

(c)what has always been happening lo them,


\1t2i

From habitual occurrences

(d) and probably will happen again in

the

lutute l1l2
(e) lthey do not] plan their own lives / don'l

Occurrences that might take place sometime


They are aimless/They lack/ are devoid of a
umose and aooalin life

" and how it can be oveacome:


ltow & 1ry!yj99p!9 tun into "
They haie been, a long time, being overly
years
the
hoverang
at
{f)spend
circumference of people in authority over us relianv too dependenl on those in power/
leaderc/ those who are rn positions of
(1)
(s) Pulling oneself into waketrlness when
dazed does not come naturally
oth

l)
(h) Passivity is in the short term a thousand

leadershlp in lheir society.


It is not one s instinct to be aler, observanU
sharp/ want to be dissocrate themselves from

9]!9I!

:!!tql]).i,4rp

5 nc.u,al, tarate/ e{lramely easy/,e./1. i;(

times more praclicalthan stepping forward to

useful/ pragnaticfat lhe byslander etfeci to

1i)A lesson ofthe most ancient pari ol our


brain is to stay slill and nothing willget you

We have been physiologrally wired/


proqrammed io be unresponsive / remain
1on' hdrarV -,l nvolved/ drsengeged esoeLidllr'
when danger is in sight (infe_rr9!)
To be able to get beyond the stage/state of
being nonchalanU indifferent requjres

rafearor ()
(1)

One needs learning from anolher parl of


the brain to overcome natural passivity (1)

l)

QI

(k)One needs a motivation more passDnate


lhan self interesied practicalrty (1)

One also has lo be rnspired/ stimulaied/


spufied/ stirred by a more noble desire than
merely ihat of fulfrlling one s personal
inclinations and desires

Passage B
Fro.n paragraph 2
-mo
lnier howihe example on slavery ilLslrales lndr

Fiom the
objection lo slavery
OR

People deiend slavery Olhers shrug /The


spks again.(1)

antis avery person

ne olner\ k.ep

rs

l"real to juqlr(

[^ml

Someone may feel exiremely passionately


against an apparent social iniusiice

T) one antislavery person makes a slrong

,21 I

dldtttl

qrv,nq l'ss and lFss eye

contact io lhis irritaling speake(1)

3) Nothing makes a strongly opinionated


speakeryield sowellas being denied eye

While ihe rest ot the group is supportive oithe


action / ihat apparent injusuce /Even as that
lone dissenter allempts io argue his case
againsl the iniustice
conceot: one aoainst ma
Fe e' pelences be ng gnored by l_e Iesrll is
because ol this experience ot belng turned a
deaf ear to/ neglected
Conceot: Beino iqnoredl Not qiven attention
that he willeventually surrender and shed/ lose
his moral convictions.

contaci (1/2)

Concept: S!trendeling

4) Well, I suppose ihere are some instances


where slavery mghl be all right of

He will a so eventually be brainwashed lnto


being able lojusiify why that apparenl social
injustice might al trmes be all right

course"l(1/2)

concept: Justify tnq

From paragraph 3
abolri !!931!4q99!!!ltq forty and,f i11v-vear olds? 1m
Suggested answer
disenchanted, resigned
disengaged,
Cynical,
that
forty-and
out
of
the
mellowing
Much
fifty year-olds smilingly observe rn one another
Note: Given lhal a smile can convey a mnge af,
rs srmply moraldrifi.
aftitudes/ emobons, please allow far as many
sersib/e arswe/s as lhere arc. The 1 nark

Exolain such an attilude Jlml

Suggesled ar!9!9I
Perhaps lhey rcognrse/are a\rvare that lhey
are shirking therr responsibilily (1/2)

Fe4!qP.a!!

lo speak oul against what they know is


righvshould be done (1/2)...
'

ne mi/\ wtt be g^ pr onty il n" c o'onal'en ' p.soniot, sLppois tne o'ir
Wiih reference io llnes 16 17,howdoes tloat ilustrate lhe image

l-Fromthqle$qsg f

of

'/ "

'd

moral dtII"? 12m)

,suqsesle! answer

f3

Just as somethrnq liqhunot anchored to the seabed floats to the sudace


of the waler
(1 m)
So, a sirong person of universal morals is expected to be anchored/
groundecJ/ rooied in firm moral values otherwise hd will drift /As when
moral drifl sets in. he loses his moralanchor. and hence loses his moral

From paragraph
Explain the iigurative language in the sentence "They damp lhings down to a rnossy

agreeableness'[2m]
Suoaested answer
They damp things down to a mossy

agreeableness

mots| soft condtlians


(1/2), in an uncontrallable OR shape/ess
Moss grows in damp/

manner (12)
Being in a graup is likened to a damp
condition as il promoles an exlremely
conducive envjronment for hreedng passive
padicipants, by smalhering ones personality

and opinions. (1/2)

Nate: The emphasis is not an damp . . dowF'


as a phrasalverb Whal is being emphasised is
the assaciatian belween "damp and "moss",
gjven lhal noss grcws in damp areas

The bystander effect takes place and becomes


widespread when a person s conviclion is nol
E\/en suppo4 and lhq petson gt-dudlly latet
his canvicttcn, aclopling lhe mindset

ofthe

majority, regardless of whether it is morally


acceptable

ar

ath e r,u

ise OR

ln addition, just like nosswhich has no


definite lorm or shape, people wha agree are
nol abaolutely ceftain aboul what they are
agreeing with.

o/2)
OR any other sensjble answe"r.

From paragraph 5

Why are some people more vulnerable to herding? Use your own words as far as

po\sible.

[2m]

Frol! the passagepsyclro ogicalabuse oi children grves ihem a


poor seli image that they atiempi to repair or
replace with love of a charismaiic leader.

Suggested answer _,
The first reason might be because oftraumailc
childhood expeiences where they were
mentalLy toriured/ shaken which resulled rn
them losingi confidence in ihemselves. that they
now try to re-assert themselves by hoping to
win the affeclrons/ approval of someone they

(Because modern people spend such a


signifcant share oflheir lives working in large

organizaiions,) they scarcely can recover


from the psychological effects in the time
left them after retirement.

The second reason might be"that people


nowadays, as a resull of staying loo long in a
particular job. find il difflcult, when lhey stop
work, to pul away the mentalstless/pressure
exerted on them dLrrlng their career

From Paragraph 7

7.

Based on the wriiels closing remarks, what can you infer aboul his attitude to the issue
of the herd mentahty? Use your own wods as fal as posslble. [1m]

From the passl

Suqqesled answer
(any sensible answerwill be accepled)
To encouraqe the readers io avoid being
passive, and to lake action when necessary
To convey a moral message (didactic)
- Stop ihe bad practices by exposlng ihe bad
-

- Ends on a hopeful noie that the reader will be


inspired to stand up for whal is righl

PassageA&PassageB

Give the meaning of the following words and phrases as they are used in Passage 1 and
Passaqe 2. You may wrile your answer in one word or a shorl phrase [5m]

%m,

0m

Core
Naturally

lnstinctrvely

Proclivity

Tendency, rnclnation

Gloomiest

pessimistic

(Passage B, 144

Passage A and Passage B explore the'bystander effect from the psychologicaland


socioloqical perspectives respeciively With relerence to bolh passages, drscuss the
exieni to whrch the bystander effect is observed in ioday s world? Suggest how your
society can poss bly deal wrth thrs concern Support your answer with examples from
your persona experience 19 ml

1
2.
3

Candidate should discuss one idea from Passage A.


Candidate should discuss one idea from Passage B.
Explain how their society can address ihe bystander effeci, using personal experience as

supporl

a.

Good answers would discuss new ways of addressing this concern

Nole: Given the phrasing of the question, it would be undesirable for the student to propose a
soluiion as part of their d iscussion for R 1 & R2.

7/

PJC Mid-Year 07

PAPER I

l.

'Today's technology is d;viding us as much

2-

Are harsh punishments the besl answer to rising crirne?

3.

To what extent are the values of teenagers in Singapore loday shaped by tbe

as

uniting us-' Comment

mass media?

4.

'ln .ealify, the United Nations

5.

'Work is more fun lhan fun.' (Noel Coward). Do you agree?

"

6.

'Educalion is the only effective instrumenl


society.' Is this true?

7.

'The envircnment must be sacriliced for economic developmnl ' Do you agree?

8.

'Enjoyable, but ultimately of littte practical use Consider the value


performing arts in Singapore today in the light ofthis commenl.

9.

has limited

value.' Discuss.

in bringing about changes

'

of

in

the

How concemed should we be about world poverty?

lO.'There

is nothing good in war except its ending.' Do you agree?

ll.

Has competition resulted in a less compassionale and caring society? Discuss


this with reference 10 your coun1ry.

12.

'Obsessivc behaviour is very much a part ofmodm

life.' Discuss

?l

PJC Mid Year 07 Paper 2

PASSAGIi A
Anne Nloir and David lessel lvrilc...
'l'here is now solid and consislenl cvidence liom scientisls all over the \a()rld that
thc bra;ns of girls and bo)s in the womb are struclurall) dialcrent and thal thesc
dillcrences come about as thc resuk olchemical subslances known as hormones Mnlc

lll

hormoncs chielly lestosleronc {rgan;se the dcvcloping brain into a male paftern
\\'hich leads l(J malc behaviour. Abscncc of male hormones pcimhs the brain lo pcrsisl
ils ltmal pallcm, rcsuliing in lemalc bchaviour. The wide diffcrences in the amounts
of tesioslerone fiooding the growing brain account lbr lhe .ercal lariet) of sexuxl
dilferences humans display. l'his variel' is turlher;ncreased by thc fact thal hormones
conlinue lo be prod ced by thc body througboul lifc, and exen their influcnce on our
behaviour. nolably al pubeny. In the extremes. young men \!ill be pushed lo*ards l0
liolcnce, and loung \lomcn to$'ards irrational and disruplive swings of mood and
behaviolrr- More commonlv- nrcn become more confidcni. sho\v grealer conccnlratioD
and channel their aggrcssion into molivalion and ambition. whilc women are slimulalcd
to dcsire to form and maintain closer and richcr relationships. ln old age, as the
honnonal springs begin to dry. lhc brain difierences bcgin 1o lose their sharp focus: t5
\lomen tend to bc more assertive and aggidsiive as lhe female hormones losc their
po\rer lo neutralizc the teslosterone prcscnl in all women. Mcn, in lurn. become lcss
aggressiee as their leslosterone loses its power to neutralize their orvn nalllmll)
occLrr.ing female hormoDes: as they conlcmplale the garden lhcy lvonder why lhei'
20
\raslcd so mllcb time climbing up the laddeFof stlccess .
Dcspite this incontrovefliblc.esearch. there is slill the fear thal lhe hard won ballle
tor \\omen s ighls rlill be lost iIthc concept ol'innale scxual dii]erences is conceded.

I2l

lhis l'ar leads some 1o deny thc \,alidily of all lhc thousands of clinical and
sociological sludies as bcing all biased. thc lainted work- in tacl. ol 'conditioned
scicnlisls. Others advocalc lhe slippression altogcther of such rescltrch- for fear ot lhe 25
\, 'n\.qucnce lhr r{.men , \lalu.
\\rde\pread anempls to destroy lhe sexual slercot] pe through
pro!re,si\e ellu(dtron. childrin rre rc.luilcd lo r.od ahout prin.e tet.lalrng dragons
lhcy are asked to rvrile stories beginning 'Nadinc put on the boxing gloves and slcppcd
into the ring...'and 10 look al piclure books displaying female fire-fighlcrs- Al work.
posilive discrimination is employed, dcliberatel) givingjobs to pre-determincd quolas
ot women. \rho sometimes may be lcss well qualilied or proficient, in ordcr 10
compcnsate tor the apparenl unlairness of a male_dominated slstcm. and to give
examplcs ofsuccessfirl role-modcls which will encourage olher llolnen !o compete and
-.n;-e ro fl.i s rlh in il

lllThcrc Jre still

[,{] All this wcll intentioned aclivity is based upon thc false prenise that men and
wonren rre the same. Once their essontial differences arc understood and accepted _
bearing in nind the substanlial number of men and \lomen who are 'excePlions lhai
prove thc rulc' and clearly do not confbrn lo thcse generalised descriptions of se\ual
dilltrencc \!c can begin to clear the air of tho ahosphere ol sexual ridicule and
recrimination. Wc'n1en need no longcr hold men responsible for their supposcd 'lailure'
and men need no1 fcar or deride the cffons of women to bcal them at thcir own game.

t0

i5

:10

/r

I-iberated try honeslr- ralher than imprisoDod by selldcception. men and \\'omen Nill
ha\'e the coniidencc 1() slrike their own balance betwcen love and ambilion, lcndcrness
and striving, and lcarn 10 cnioy and give proper'valuc lo their nalural selves.

45

PASSAGE B
Salzman

wriles...

llowevermuchNc strive as individuals and as a society to be gcndcr neutral. il is


vinually ;mpossible lo be gender blind. Male and fcmale are tundamonlal calegories
for human beings in dealing with one anolher. ln cvcr.,'place and cullurc. people
immcdialcly identily olhers as male or female on firsl coDtacl Some sociologisls would
sa) wc havs lo know a person s scx in order 10 know how lo inleracl wilh him or her.
Any parent who has broughl a baby in gcnder neulral clolhing lo a playground knows
tha{ anong the questions hc or she will hear is. "ls lhat a boy or a girl?' Only once that
has bcn eslablished does lhc slranget begin lo inlcracl t\'ith the baby. At leasl one
sludy has sho\lD that parenls and olhers tend to smile morc at female babies ll may

[]l

rvcll bc that $,e cannol cvcn seille on a facial cxpression

lil,

we have ascenained what

t0

turks bcncath the diaper.

[2] Whar does diflcr from place to place

and from one era to anolher are the perccived

Dormal characteristics of "maleness" and "fcmaleness." and how lhose affccl


individuals. their relationships and society at large. Over the Iast quarlcr cenlury, therc
have been seisnic social-psychological shilts takiDg place behveen the sexes. These
shifts are b a largc extent drivcn by the d)namic inlcraction of sciencc. lechnolog).
and economics, and are maenified-by the increasingly omnipresenl nledia ln one Nay
or anolher. mosl major socielies today have to come to grips rvith big changes in the
bcha!iour ofmen and women

l5

[]l In thc Wes1. there havc been lierce debales, still raginS. aboul which gcnder

)o

play? Can they pay?

25

behaliours are ;nnate and which are socially dclennined. Whal s nature ard rvhal s
nudurc ir the sexes iislill open lo discussion and research, bul t!'hat is cenain is tbal
r\e e. Jnom) doe\ nol mulh carc. it rerr:rrd. pcoplc uho , rn prudu(c rnd , nn\umc and
it care\ n^rhrno aboul $hellrer'h() bullon rheir j x kcl' on the n"hr,'r lhc lell a,n lhc\

[4]

The growing economic porver

ol

womcn owes much to the shifl from land

dependenl agra.ian economies through faclory-dcpcndent manufacturing econonics to


inrclligence-ilependcnr service economies. The more women becoinc iully qualilied
economic pafiicipants in their own right. ralher lhan mere depcndanls ol mcn. the rnore

we all have lo face fuDdamcnlal questions about marriage. procrcalion. the role of
famil) and thc changing fbrce of 'rhc tribe. '

.10

rrsing in mo{ ul the de!el'neil


n.r orr.. rnd thal r.o. h.. ir' imlr(t un rn"rridge i"J froc'calion Fe pl( drc no long.r
orer thc hill at fofly. past it at filiy- old al s;xt) and gonc at seventv wc now have a
\'asllv grcaler range of oplions in mosl arcas of lite and a longer lile in which to
e\periencc tben1. so lhere is less nrsh 1(l gel nanied early and is more challcngitg 10
Lalc parenthood. sccond families and cvcn third families are
stay marricd for a
couplcs sepa.ate and form ncw pairings And more olien lhcse da)'s
far fioln unusual as'ifelime.

Ii] ln our new \\,orld, lil'e expeclanc]'is srerdill

F7

lhan in the past, women are the ones with the wandering eyes.
[6] Shifts in g"ender behaviour are rarely greeted with unanimous approval. Okay make 40
thal "nver." The people with more financial,.social and political power (mostly men)
tend to resist the shift of power by those with less (mostly women). The vehemence of
male resistance varies from culture lo culture, from the extrmes ofAfghanistan under
the Taliban to the rearguard actions of male-solidarity movements in the Uniled Stats.
In lhe short term, it is clear that cultures that resist the rise in femate power are los;ng 45
oi.r1 io those cultures thal accept it because the cultures that accept it are progressing
health, economy, security and technology, lo name just a few.
faster on most fronts
Only history will tell what the longer trm consequences may be.

From paragraph

State in your own words. one negative and one positive example that hormonal eflbcts have

t21
commenting thal,"-..as they contemplale the Sarden fhey
wonder why they wasted so much time climbing up lhe ladder of success." (lines 19 - 20)
Explain your answer fully.

2 Whal is the writer's intention in

trl
From paragraph 2
3 Why do wonen want to protecl their riShts through "...lhe suppression altogether of such
research''? (line 25)

t21

From paragraph 3
does the word, "deliberatety" (line 3l), tell us about the attitude ofwomen's rights
advocales? Explain the reason for this.

4 What

12)

In your own words, xplain why in progressive education the 3 examples are often cited?

trl

From Passage B
From Paragraph 4
6 Why is lhe phrase. "the tribe" (line 3l) in quotation marks? '

tll

I to 5, in about I50 words, accounl for the cbanges in the


probleins
arising from these changes. Use your own words as
status ofwomen and state lhe
lAing material from

paragraphs

tar as possible.

IIUI

'
-

Questions from both Passages


Give the meaning oflh following words as they are used in the passages.
You may write your answer in one word or a short phrase.

From Passag A

...............................t11
(b) premise (line 36)...

From Passage B
(c) ascertained (line

I0)...............

......111

trl
(e) unanimous (line 40).....

Passage

Ill

A acknowledges that men and women are naturally difterent but cautions thal these

differences should not be the reason for gender discrimination. Passage B $ates lhat society
has to bear the consequences arising from i1s reluclance to accord women equal status \4ilh

DrawinS information and ideas from the passages, assss the place of womn in Singapore
today. You should rely on your own knowledge and experience tojustify your answer.

Iql

/at

PJC

['rid-Yerr

07

Paper 2 ANSWER SCIIEME


Qucstions frorn Passagc A

Fron Parrgraph I
L Stalc in )our own words. one negalive

and one positive example

of homonal

(2 marks)

elGcls on adolescents.
Notei Ans}r'ers need not be gender specific as long as

negative nnd

I posiiivc

exanrples are shown.

Beyond normal limits- young mcn


will becomc aggrcssive/resorl to the
use of force OR

Negative

a) In the cxtrmes, youog men


lvill be pushed to ards
violencc.(lines 10- I l) OR
b) young rlomen l0wards
irralional and disruptive swings
ol mood and behaviour.(lincs I

Young women will become illogical


and lcmperamenlal/cmotional.

(l-)

12)

Positiie
c) More commonly, men bccome
more confidenti show greater
concentralion and channel lheir
aggreosion into motivntioo and
aInbil;on-(lines l2-l 3) OR

Men bccome rnore sure of


themselves/self-assu[ed, display strongcr

powers of attenlion and direct their


energy from bad/violnl behaviour 1()
malerialist

ic

pursu ils/ach ieve menl s-

(Notei Award /, mrrk for general


stalements like achieving thcir goals/
achieling success 'ilhout spccific
reference 1o material success/pursuil)
OR

d)

while rvomen arc stimulatcd to


dcsirc to fbrm and maintain closer and
richer rclafionships.(lines ll-11)

(a)

1()

On the othcr hand, womcn are inspired


forge slronger bonds with onc anolher.

(ln)

What is lhe wrilcr's inlention in commenling that, " ....as they confemplale thc
garden lhey \ronder why they waslcd so mnch timc climbing up the ladder of
succoss'-(line l9-20)? I-xplain your answcr tulll.
( 3 marks)

(inlcnl;on)'-inferred

To show lhat lhe diffcrences in

male

behaviour a1 lhe trvo different stagcs of


their lives (I/2n) are a tesuh ol hormonal

changes.(l/2In)

(b). .lhe),lvasled so much lime clinbin

wh"n

!!.f I9S ]S!!C.M d !9!!!9

/o3

their time scnsibly(l/2m) and instead


wcnl all ou1./spen{ all their tine chasing

the ladder ol'success'.

aflcr/lrl'ing lo

acbieve material
goals/tD/ing to rcitch for higher posilions
in lheir careerc. (l/2m)
Mcn in
turn.
aggressivc...female
r9)

ln their old

bccome

horm,,nc\.(line

I7

age. they mellow

and

become lcss ambitious.(l m)

From Prragrapb 2

,. \

h) do t{om(n $anl lo prolecr lhcrr IrLhl\ lhrough ...lhc suppre'.ron dllu8elher


(2 marks)
ofsuch research "l (line 25)
(Nole: The correcl anslvers are in (;) and (ii).llowever if thcy only make a general
slalcment as in (iii).only Z mark to be awardd).
Total marks

L(t

i1

{i)-{ii) and lrri} crvcn:

__

(i)...lhey still lear thal thc hard-won lhey have ioughl so hard lo
(ii)fbr women s rights r}ill
22\

Gi

(ln)

be lost . (line

And are afraid that these enlitlemcnts


be laken a\a,ay

ra. .rt rh" .o*.q,,.tt*s r*


s slalus {lines 25 26)

--ro,

women

achieve

gcnder equalily/ women's libemtion.

battlc...(linc 2l )

lrom thcm. '

allcl
society.

Afraid tbat it will


womcn in

will

(ln)

thc position oI

(l/2\

trom I'aragrat'h J

4.

Whal does the \i'ord, "deliberately" (line 31), lell us aboul lhe attitudc ofrvornen's
(2 marks)
Explain the reason for lhis
ighls

"dlocales?
'deliherrrel-\"

Al lvork. posilive...giving jobs to

ovcrly eager/ anxious/ biased/ forceful/


p.e_

of \\'omen, who
order to
somelimes ...in
compensate...unfairness of a maldelemined quotas

dominaled system...(lines 30' i3)

5.

detcrmined/ adamant in addressing the


issuc ofqender ineoualitv (lm
Reason for havins such an attitude
so as 10 ensure \\,omen have an equal
footing with/arc on par wilh men (l/2m)
in the .iob/labour market/ workforce /
employmenr.(l/2m)
Nole: Reference 1o iob market must bc

In your o\\n words. c\plain \\,h)' ;n progress;ve education the

eramples are oftcn

I mark

ciied?

1l is to Pul an

end

neralization/fi xed idea or ;m

lo

the
fixated

/of

...princesses slaying dragons...Nadine


pur on $e boxing gloles.-.picture books
displaying female fire-figh1crs.(lines 28 -

nolion lhal women/ females are


(lrn)
inferior/weaker scx.

thc

Questions fronl Passage B

From Paragraph

6.

{
ll)

why is the phrasc, "the 1ribe" (line

inferred

in quolalion marks

(lmark+lmarkbonu9
11

is bccause womeo are described in


unusual . fashion/negatively

an

portayed. (lm)

As membcrc of a tribe, they belong


to a social group of people with the
same belieflt/customs (l/2m) and
adhere lo rules & regulations lhal arc

(Bonns)
Explain charrcterislics of "the tribe"

rigidly structured for then.(1/2m)

7.

Using material fron paragraphs to 5. in about 150 'words, account lbr the
changes in the slalus oflroDlen and the problems arising froln thesc changes. Use
(10 Inarks)
your own words as far as possiblc.
1

Ltfi
Reasons

fot the chonges in fie

status

of

a)...*,e strive as individuals and as a ln today's social conlexl, as pcrsons and


societ) to be gender-rentral...(paral. as a group. we are making tremcndous
eflo /rllempl to accept bolh sxes
line l)
wilhoul Drciudice/discrimination..
How men and women are vieNed is
dependent upon the place and lime in
question-/ Over time and in different
places or locations. lhe stalus and/gr thc
perception ofwomen has changedc)...lhere have bcen seismic social- Radical changes in society and lhe
psychological shifts...(para 2, lines 14- mindsels ofpcople are apparent.
I5)
d) ...driven by thc dynanric intcracljon of These changes have been influenced by
science, Iechnolog). and cconomics. and science. lechnology and econom;cs and
are magnified by thc increasingly made more imporlanl lhan the), really are
9!!r!]!!!!!'n441Isq?.!!!!]!:L7I bv the Dowerlul mcdia.
b)What dos differ from place to place
and liom one era lo another arc fhe
pcrccived normil rharaclerislics of
"mrleness" and "temaleness"...(para 2,
lines l2-l3l

/at-

e) ...the economy does nol much care;


it rcu'ards...Can thc) pa)'? (para I, lincs
21,25) OR

-lhe gro\\ing

economic porver of

Economic consideralion transcends lhe


supposed dillerence between men and
women. / Nowadays men and women are
treated in lhe same wa) in our consumerorienled socicly.
(same

pojnt

economic consideralion)

\!omen,. servicc cconomies.(pam 4- lines


26 )t3)

Prcble ms ari

si!8:lM!

l!!!ese chdnees:

f)The morc women become frlly A. t"".* b4"." c"ittlir'iy

employed.

qualificrl crononic prrticipants...face the basic issues of marriagc, childbinh


fundamental queslioDs about marriage, and the function of tamil), will become
procreation,the rolc of family and the concrns that need addressing.
cbanging force of thc tribe. (para 4,
lines 28 3l)
g)ln our nelv world. life crpectancy is Due to the bnger lifespan ioday, \\'omen
are marrying laler, thus aflecting
steadily increasing...impacl
marriagc and procreation. (para 5, Iincs conceplion and childbinh.
h)...and is morc challenging to slay
married for a lifetimc. (pa.a 5. lines 16

i7)

i)

Lite

parenthood.

familics...lar from

second

unllsual

couples . new pairings.(para 5, lines 17

ll

is more

difficuh for womcn to

reDrarn

married lhroughout lhcir lite.

They are hcsitaot in starling rheir famil'es


earll /do not wish 10 start their fanilics

early/divorces/ multiple maniages arc

38)

j)

And more often ...women are the Nowadays. \\,omen arc more Prone to
onrs ]rilh the *andering c)es. (para 5. infi delily/rnfaithlilncss.
lines l8-39)
Questions from bolh passages
8. Cive the meaning oflhe following words as thcy are used in the passages.
You may write your answer ilr one lvord or a slort phrase (s rnarks)
From Prssag( A

asserlive(line
16)

selFassurcd,firm,bold,lorcelul.
strong
confiden1, possess;ng

p'.-;iitlin.

assumplion.hypothesis.postulation

r6)

confidently
aggressivc.
decisive
concept,idca-

idcology,rolion,claim.

lirom Passage B

/rt

cstablished. verified, madc


clear. bc surc- made/make
sure. lo bc sure/verv sure

2l)

irgrained.
inborn. inhercn{. natural.
inslincti!e
born wilh
agree, no objeclion,
Lrnanimous (line in complete agrccment, 1o
lolal agreement by all in a consensus- uniled unopposcd.
10)
innate (linc

group,

unequivocal,

doubtless.

indispulable.

in opinion

llithoul

doubt-all in lavour

9.

Passagc A acknowledgcs thal men and woincn are naturally differnt bul cautions
thal these differences should nol be the reasons for gender discrimination Passage
B stales that society has lo bcar thc consequences arising from its reluctance to

accord women equal slatus wilh men,

Drawing infbrmalion and ideas from the passages. assess thc place of llonen in
Singapore loday.

You should rcly on )our orvn knorvledge and experience lo jusliry ). our answor.
(9 marks)
Candidates a.e to suppon lheir evalration wilh reasons and iustifications Weaker
Onswers ma) shorv evidence of over generalizalion instead ofconcrete support and
ease

otunderslanding

mark)

l!r{k

Band R (,t-gJnarks

,e,?!4!

Examincs points lrom bolh

Examines points from only

Passages..

one passage.

Bolh aspects of lhc


queslion are covered.
Atlempl is made to presenl
ideas systcmatically-

Ans!vers the queslion.

Answers the qnestion

Evaluales the kcy


arguments thoroughly-

Lvaluales relevanl
'orgunrcnls to so*. e,*tcnt
bul discussion is

adding their own

supgrficial.

contributions.

Explanaljon is oflen
limiled or not l;nked 1() thc
maiD arguments well.

General discussion !vi1hou1

Band A (7-9

Points are taken from bolh


passages;lhere are
slslematic rcferences lo
thc lcxts,

"

Brings in ;nteresling
elaborating on these. Able
to jusl ify some (economic,
lirical. educational clc.)

11-3

\\.ilhoLrl referring 1o idcas


in the texts. Answers onlY
one pafl otthe question.

Mcrely summarises the


ideas

ofthe wriler wilhout

relelring lo their socicty.

/al

achievemenls otwomen
in their society clearly.

Sho$,s high dcgree

of

Coherence and

orgarizalion are nol as


shary or sl,slemalic as the
lop band.

organizalion.

Inconsistency in the
argulnent. Coherence is in
question.

Possiblc Points from Passaee A


t ftt"r" i" nJ solid and consistnt eridcnce from scientisti...structurally
diffcrert.,-(p:rra l). {For example. wonen are cxempled from NS and caning
llo*ever, {his must not be the basis 1br society to givc unfair lrcatmenl lo rfomen'

t,"tge' hold Incn re\ponsible lur ihrir


"o
supposed'failure'..(para 4). (They are unable lo succccd in their career because
I

\4

omen *".t

of.sexLral discrimination and unequal distribution

..

of

domestic responsibiliries)

Lib(raled b] ttun."tl-*it" p.op". talue lo lhcir nalural selve\' {para J)'


( In order ibr local women to cnjoy equal righls with men. there must be an honesl
atlempt by all members of society b remove all misconceplions aboul women
being the inferior gender and to embrace/celebrat the unique slrcnglhs/qual;ties
hoth men and somen )

Possilrle Points

of

r. ...1ft"." ttut. U""n scismic sfl(ial-p\]chologi.al shilh laking plac( botscen


lhe sexcs. (para 2) ( Since indcpetdencc, Singapore has $'ilnessed major changes

women significanrly. It will bc difticull for SingapoJe lo progress if womcn were


rlenied thJ opponunities to fully parlicipate in lhe economic devclopmenl of the
counlry. In a globalised economy- it is crucial to have a level playing field tbr

marriage.procreation, the role

of &q4X.4ql\"-j!?'Citlg lqrce of ':t!9

/0p

tribc"(para 1). { I he traditional roics ot women as child bearers and homemakers


have changed. Social expectations srch as eariy malriages. staying married
rhroughout lile- stigma ofdivorces and rcnrarriages etc have changed dlamalicaily
no longcr expected to bc srbserlienl as they are economically

Some Doints to consider aborlt ihe Droeress nrade bY local rr omen:

Pol!l!s4.!44qp49d

E.

Grcalcr involvement ol women in polilics. For example, in the GE 2006


many women candidales lronr both lhe ruling and opposition polilical parties,
stood 1br election logether rvilh thcir male counlcryans. Some womcn like
Sylvia l-im holds important posilion like Secrelary Ceneral of the party
(Workcrs Pany) $at shc bclongs
Women are appointed to high political posilions such as Minisler of State,
Parliamentary Sccretary and as aDbassador aparl from be;ng Mcmbers of
Parliamenl. Examples include Ms.Lim II\\'ee Hrra (MOS, Min ofl'inancc and
T.anspon). Pro{l Chan )'leng Chee (Singapore's Ambassador lo the Uniled
Slalcs). Mrs. Yu Foo Yee Shoon ( MOS, N{ in.ol'Comn un ity Developmcnt.
Youlh and Sporls)
woncn MPs like Madam llalimah Yacob. Indranec Raiah and Dr. Alny
Khor are making their voices heard lhrough their political pa(icipalion and
are well respeclcd in the socielY.

&ry!!a!ry",!rraq!
Successl'u' womcn entreprcncurs are a common feature in Singapore loday
success of Singapore arc by no means
lrivial. Some er(anrpl;s include olivia Lum (Hyllux). Jannie Tay (Hour Glass).
l-ina I an-l-eo (The Link Croup) elc.

l heir contribulions 10 tbe economic

Thcrc are women who are spearhcading big ecoDomic corporalions. for
e\drnplc Ho t lring r lcmasck Iloldindsr. c5\o,rIl( Plolcrsor lv) \r rKK
lluspirdlr ( lairc ( h;dng (Ban\rn lree HoldrnS.r (lc l\e1 pl;,1 moior role'
rn rhc c.ono'nr, L'c\clofmenl uf lhc.uunln.
Woncn coostitule a high percentage ofthe rvorklorce and are no longer seen
as passive contribulors to the econom). ln the pas1. local women lvcrc mainly
performing household chores and werc no1 regarded wilh imponance ;n terms

oftheir cconomic coDtributions to the counln'. "

l:.

.rq!ry

"'a r,rr""ri."l

Women are no longer associalcd \\'ilh jobs ihat are trad;tionally "female"
dominated like nursing and leaching.
They can be lbund in all areas of rvork like. enginecring. law. archilcciure,
neurosurgcrv etc thal *rc once male enclaves. They have also made lhcir
mark rs commcrcial airlinc pilots (fbr example in Jetstar). fighler pilots c1c
Edlrcationally, woncn are exccliing in all liclds of sludy and lhcre is a high
number o1 f!rnalc siudents in all the teniarv insl;tulions in Singapore Th

number of wonen :rchieving Poslgraduale qualificalions is also increasing


More women arc better educaicd today as compared to their predecessors

Some impedimenls that arc hinderinE local women from makins Droercss:

il.

"

,.

Culture and Tradilion


Some Singaporeans ot Mala). Chinese and lndian origins arc still clinging
to thcir age-old cultural and lmdilional beliels and practices Girls are seen

as less imponanl than bols (lbr cg. they are considered lo bc fl1 tor
domeslic chores only) and as such are given lcss priorilics and
opportunilies gencrally. Prolcssional women have also bccn known to
resign from their jobs alier marriagc to look aflcr the childrcn and lheir
husbands as expecled by their cultures and tradjtions Unless these beliefs.
valoes and praclices are changcd, women today ma) slill have to sufler the
fale thal lhei predccssors vrenl through

Minrtsq!
Dcspite 1he lacl thal Singaporc is economically affluent and progrcssing.
somc people slill hold on to thc mindset that \lomen are nol men's equal
and can ncver be.(Sadl). some e(fucalcd women. may also subscribe 1() lhis

think;ng). Singaporeans ofall generalions- regardless ofrace. language and


rcligion need lo lotalll ch"angc their mindscls to accepling womeD as equal
. nnlcrntordrie\ u' nrcn llllr pcr.(pri^r car cnhance $, rrrcn Tdnr. pali^n
'
in all domains ol lifc. Curentl). no iroman is appoinled as lirll lime
Minister in lhe Cabincl.
Religion can also be a barrier lo womcn achieving thc same rights

as men

Ftrltr!rion of nropres\ midc hv local $,,m(n:


L Although thcre ,s equalil) for bolh scxes to excl in all domains. in realily
nary women are denied lhc opporlurities 10 showcase their talents and
abilitics. For exaDple in lhe political arena. no women have been appoinled
as Cabinel Minister. 1el alone Prime Minisler or President Employing womcn
as pilols is mrc and f-ar in bcl$'een.
Rt'fardle\\ oJ hcr cdy.arr. nalcareer r.lirc!c-nenl. r\onran. un n nrarriag(
is slill stereotyped lo be the homemaker. Singapore can slill be considered a

parriarchal socicly $here the lraditional perccpl;on aboul the role of lvomen

as homcmakers prc!ails.Ho\\evcr- the undcrcurrenls of change are palpable

loda].
Fi.\ed mindsels/prejudices of emplol(rrs regardiig the contributions lhai
$omen can make due to lheir dual roles as emplolces ard molhers posc a
hindrance to lheir progres(lhc ironl'is thc country needs them to procreatc
for long lcrm survival bul the mindsel ofenploycrs is exlrenrely resislanl to
changc).

,'/b

RJC Mid-Yeat 07

eaper't

Answer one question from this Paper.


Answers should be between 500 and 800 words in length.

1.
2.

Are high salaries for government officials ever justifiable?

3.

"The lnternei is the best thing that has happened to democracy in recent

"sinqapore needs more scientists than artists." Do you agree?


years-" Discuss.

4. "Future wars will be fought over the control of natural resources-" Comment
5. "Beauty pageants are degrading to women." How far do you agree?
6. ls marriage still relevant in today's society?
7. ls it everjustifiable to infringe intellectual property rights?
8. What can we learn from the study of Mathematics?
L ls capital punishment compatible with a truly enlightened society?
.10. Do you agree that genetic moditlcation brings about more problems than
solutions?

11.

"There is much more we can do to protect the rights of the migranl workers in
Singapore." How far is this true?

12.

"The winners in globalisation benefit solely at the expense of the losers

"

Discuss.

End of Paper

///

Passage

Jim Hatt wrdes abaut the rise of soft paternalism in America

'1

When lhe government tells you that you can't smoke martuana or that you
must wear a helmet when you ride your motorcycle even if you happen to like
the feeling of the wind in yoLlr hair, it is being paternalistic lt is largely treating
you the way a parent taeats a child, restricting your liberty for what it deems to
6" your o*n good. Paternalistic laws aren't very popLllar in this counlry' We 5
hew to the principle that, children and the mentally ill apart' an individual is a
better iudqe of what's good for him than the state is and that people should be
free to do what they wish as long as their aclions don't harm others

But what if it could be shown that even highly competenl, well-informed people
fail to make choices in their best inlerest? And what if the governmenl could 10
somehow step in and nudge them in the right direction withoui interfering with
their liberty, or al leasl not very mLlch? Welcome to the new world of 'soft
paternalism." The old "hard" palernalism says, We know what's best for you,
and we'll force yoLl to do it ln contrast, sofl paternalism says' You know what s
best for you, and we'll help you to do

it,

15

ln some states with casino gambling, like Michigan' compulsive gamblers have
the option of putting their names on a blacklist or "self exclusion" list, thal bars
them from casinos. Once on the list, lhey are banned for life lf they violale the
ban, they risk being arrested and having their winnings confiscated.

The voluntary gambling blacklist is an example of what's called a self binding 20


scheme. lt is a way of restrucluring the external wodd so lhat when fulture
temptations arise, you will have no choice but to do what you ve jLldged to be
besi for yoLr. The classic case is that of Ulysses, who ordered his men to iie
him to the mast of his ship sb that he could hear the song of the Sirens ihout
being lured to his destruclion. As a freely chosen hedge againsl weakness of 25
the;ill, self-binding would seem to enlarge individual liberty, not reduce il So
what is there lo object to"in a programme like Michiqan's?

Plenty, say libertarian critics To begin with, they don't like soJt palernalrsm
when it involves the state's coercive power' they are much happier with private
self-binding schemes, like alcoholism clinics and Weight Watchers clubs What 30
bothers th;m is the way soft paternalism relies for its justification on the notion
that each of us conlains multiple selves - and thal one of those selves is
worth more than the olhers.

The short-run self eares only about the present lt is perfeclly happy to indulge
loday and offload the costa onto fulure selves. For example' recent shjdies 35
show thai teenage smokers are aware of lhe aisk of getting lung cancer as
adLrlts, but they simply don t mind making the future seli suffer for the pleasure
of the moment. The prudenl resolutions oi the long'run self are continually
rgnored

Bui why, sorne sceptics ask, should the government side with your pnrdent 40
long-run self against yoLrr hedonistic short run self? lf the goal is to promote

/'Z

freedom, there is an interesiing argument favouring the former. A distinclive


quality of humans is that we do not simply have desires, we also have feelings
aboul our desires. Take the unhappy heaoin addict: he gives himself an
injection because he desires the drug, but he also has a desire to be rid of this 45
desire.
So why can l soft paterndlsm be lerl to thF prtvale seclor. as sone hbenarlans
prefer? The problem is that private self binding schemes are easjly subverted
when someone can make a quick buck off your weakness of will. One
Michigan man who signed up for a casino s private self-blacklisting programme
found the owners all loo accommodaling when he had a change of heart
.Within
half an hour, I was back in, he said.

50

Besides sofl paternalism, there are certainly rrore exalted ways to achieve
mastery over unwelcome impulses. Exislenlialist thinker Jean-PaulSarlre used
to insrst thal each of us is free Io redefine his characler lhrough an act of
radical choice. For the religiously incJined, an access to divine grace might be
what is needed lo stiffen the will
10

Bui what if you are one of those people who rely on more mundane

slratagems, like self bindang? The general problem you face is lhis: For a given
uphill goai and a given strength of will, does there exist a paih, however
circuiloLrs, that will get you to the top of the hill? By adding a new path here
and there, state soft palernalism makes it more likely that the answer will be
'yes'.

so|]rce New Yark Times.

3 Dec 2006

//3

Passage 2

Daniel

Picture a man gobbling a second helping ot chocolale cake, or chain-smoking


a pack of ciqarettes, or injecting heroin into his veins ls aggression or violence
belng done to an innocent person?

ln a sense, yes. A fleeting, short{erm self ihat enjoys chocolate, nicotine, or

Klein

es aboutthe morcl consequences of paternalism

heroin is working his will on an enduring self that pays the cost. ll makes sense
to describe ourselves as a bundle of multiple selves that overlap, intermingle
and sometimes conflict
Atthough you may not think of yourseF in the plural, the idea of multiple selves
is really familiar Some people recognize a dark self - a Mr. Hyde - that lurks
within, and act stralegically lo defeat him. lf such personal tactics are
insufficient in subduing the Mr. Hydes that lurk, perhaps the government can
lend a helping hand. Afler all, subduing bad guys is what the government is all
about This reasoning underlies numerous palernalistic laws worldwide

Laws help us not lo overuse a huge variety of substances, kom heroin 1(l
penicillin Other laws protect us from buying on impulse, from not saving for our
old age, and lrom murdering ourselves. Allthese laws are offered as a seTvice
to the erstwhile partaker. Yes, people sometimes do things they regret You'd
be ridiculous to say that yoLl never make mastakes, that you never do things to
excess. Bul does that mean the government should step in and protect us from
ourselves?

One good reason to rgect paternallsm is that pubhc oflicrals, do not rn fact'
know better whether an activity is detrimental to our enduring self Passing
blanket restrictions on behaviour rides aoughshod on individuality Some
people drink too much, or gamble too much, but many others do not Another
reason to reject paternalism is that it sels a nefarious precedent. Up to what
point does the government get to play nanny? Where does it end? Sometimes
il is the government that seems to suffer addiction to power and it is the one
thal needs to show reslraint Furthermore, the justification of "il's for their own
good is bound to be abused. lt will be Llsed to justify all manner of special
interest plLrnder, such as excessive licensing laws.

6"

But the chief reasons for rejecting palernalism do not deny irrationality in the
private individual, nor even wisdom and benevolence in government slewards
The chief reasons to reject palernalism are its moral and spiritual
consequences..

Paternalism is demeaning to the Indlvidu;l because it demeans his exlstence.


It makes existence a happening rather than a wilful aciion; it makes the story
arbitrary and alien, rather than purposive and personalised The autonomous
individual admits his inconsistencies, his conkadiciions, his bedevilling
impulses, but insists nonetheless: Grant me the dlgtlfy of choosing which
behaviours deflne my being, and charge me with the responsibility for the Mr
Hydes that lurk. lf you suspect that some Mr. Hyde seeks lo undo me, then I

t0

15

20

25

30

40

say go ahead and let him try.


A tirst mora! consequence of liberty, then, is dignity, the romantic sense of
being the captain of one's soul and relishing the drama of one's existence
There is a second moral argument against paternalism. To give our existence 45
beauliful meaning, io make ourselves becoming, we must learn how to
manage our troublesome impulses But how do people learn self-command?
The best teacher is liberty herself. The second moral argument is that liberty
breeds personal responsibility
ln the intimate contest of self-command, hubris often prevails. Beforehand we
say we won't gamble at all. we won t get angry, we won't watch TV, we won t
we won't! We forget that the mood and vision in which plans are laid may vary
greatly from the mood and scenes experienced as the road is travelled So
hubris produces unhappy experiences and a feeling of regret. From experience

we qain awareness of the need for better self-command and a will to defeat
our entrenched impulses.
10

Rather than leaving the individual free to learn from experience and example,
paternalism deprives us of moral opportunilies of choice. By pre-empting
choice. it weakens the moral faculty of choosing one's own course And by
presuming thal the individLlal is incapable of choosing competenlly, a
paternalistic government may aclually make him incapable Wean a person in
a world of decrees and prescriptions, and he may fear personal independence
and responsibilily.

11

Mr Hyde sometimes makes me eat too much chocolale cake or smoke too
mani cigarettes. that is my problem, and everyday I musi practise the art of
lf a

55

65

subdulng and negolialing with him

So$ce The Freeman: ldeas on Libefty, May 1994

//:'

Answer all the quesiions


you
your
own
yoLl
slill
use
passage
your
musl
for
answer,
select the appropriate material from the
words to express it. Liltle credit can be given to answers which only copy words and phrases
from the passage.

Note: Give your answers lN YOUR OWN WORDS AS FAR AS POSSIBLE Even when

Questions on Passage

1.

According 1o the author in paragraph 1, why are paternalistic laws not very popular
in Arneaica? lJse yout own words as far as possible.

What are the differences betlveen the old hard paternalism' and the new "soft
paternalism (lines 12 - 13)? Use your own words as far as possible

3a

3b.

According 1o the author, how do self-binding schemes help the individual manage
"fLrture temptations (lines 21 - 221? lJse your own words as lar as possrb/e
ln the analogy of Ulysses (lines 23

25), what represents future lemptations"?

121

t21

t1l

tll

ln paraqraph B, what does the aLlthor say about the nature of companies offering
private self binding schemes?

5.

11l

lJsing your own tvoads as far as poss/b/e, explain ihe allernatives to sofi paternalism
sugq;ated by the author in paragraph

I2l

Questions on Passage 2

"After all, subduing bad guys is whai tlle governmeni is all about' (lines 12 13)
What do these lines imply about how the government perceives the world?

tll

Why does the author put the word "my" (line 65) in italics?

I2l

Using material from paragraphs 5 and 71o 10, summarise the auihois argLlments
agai;si paternalism Write your summary in no more than 110 words not counti'g
the openinq words belo\N. lJse your awn words as tar as poss/b/e
One of the reasons why paternalism should be rejecled

is

tgl

,/l

Questions on Passages

and 2

Explain lhe meaning of the following words as they are used in the passage. You
may write your answer in one word or a short phrase.

t51

a) nudge (Passage '1, line 'l1)


b) hedge (Passage 1, line 25)
c) hedonistic (Passage 1,line 4'1)
d) distinctive (Passage 1, line 42)
e) fleeling (Passage 2. |ne 4)

l0

Jim Holt presents some arguments in support of soff paternalism in Passage l, while
Daniel Klein is not in favour of it. Which w.iter's views on patemal,stic governments
are the majority of young people in your country inclined to support? Explain the
reasons for your choice. Your answer must reler to ideas raised in BOTH passages,
as wells your own ideas and knowledge.

End

Iel

of Paper

4v

RJC Mid-Year 07 Papet 2 Answer Scheme


Questions on Passage

Accordlng lo the author in paragraph 1, why are palernalistic laws not very popular in
America? Answer in your own waKls as far as possible

t21

a) We hew lo the principle lhal,children The reason is that Americans believe that a
anc! the mentally ill apart, an individual person rs more capable ol makrng super oJ
is a pg49Li!9!99 of v/hal's sood for decisions about what is beneficial
/advantageous for hamself (1/2 m)

"belletdecisions" (0m)
than the governmenl/authorities

b) thaD the state is (hnes 5-7)

1/2 m)

c) and that people should be free to do People should have the autonomy io act as
they want to (1/2 m)
whal they wish
cl) as long as their aclions don't harm

9!l?rf ..(i!9

!)

provided that they do nol


others. (1/2 m)

hud to

Examiners' comments on Q1:


Most students found this a manageable, skaightforward question

2.

What are" the differences between the "old hard paternalism" and the new soft
paternalism" (lines 12

|roryi!9

13)? Use your own lvords as far as

p9:tegq

..-

a)The alcl "hard" patemalism says, We


know y!3!Sj9S!_b! y9g

posslble.

[2]

The old hard paternalism claims to


of whal is most beneficial for the

and wall coerce lhem to do it (1/2m)


make sut e (0m)
c) ln cantrast, soft paternalism says, while the new soft paternalism thinks that
the cilizens themselves are aware what rs
You know what's best for vou.
most beneficialfor them (1/2 m)
b) AEd well force you to da it (lines 13
14

d) and we' help you lo do it.


15)

nes 14- and will assist them to do so (1/2 m).

Examiners comments on Q2:


Points [a) and (c) were often left out of students' answers. The focus for points (a) and (c)
is on Wbg is assumed to be aware of whal is most beneficial to the indivjdual in each kind
of paternalsrn

3a

According to the author, how do self binding schemes help lhe individual manage "future

/,/,

temptaiions" (lines 21-22)? Answer in your own watds as far as passible.


Fram the passaqe
a) lt is a way af restructuring the
external wotld (line 21)
b)so thal when future temptations arise,
you will have no choice

c) but to da what you ve judged ta be


best fot vou (lines 21-23)

I1l

It is a method ot reshaping/changing the


surroundinqs/the environment/
circumstances (1/2 m
so that when lhese future lemptations arise,
you cannot help (1/2 m)
"chosen" (0m
but do what you have determined lo
benefivadvantaqe you the most (1/2 m)

Students have to get any 2 of 3 points to get the Iull 'l mark.
Lxaminets commenls on O3a
For poinl (c), the parl about what is "best for you" was oflen leJi oui of students' answers
Students should try to be as accuraie as possible in their paraphrasing

3b

ln the analoqy of Ulysses (lines 23 25), what represents "fulu.e temptations"?

l1l

From the passaqe


IJlysses, who ordercd his men lo tie The song of the Sirens/the Sirens.
him ta the masl so thal he could hear
the song of the Slrers wtthoul being OR
lured to his destruclion (lines 23-25)
The inclination / desire to hear the song of
se4! 1!!!I9fq!np19!19!!1
the Sirens re
Lxamrners commenls on O3b

There were some very peculiar answers to this one - eg "Future temptations" are
represented by the desire to sleep;the desire to cause massive destruction: the desire to
go to war: the desire to commit suicide, etc. Needless to say, all such answers failed to
score any mark for this queslion.

ln paragraph 8, what does the author say aboul the nziture of companies ofiering private
self binding

schemes?

Ill

The author implies that such companies are


self -servrng/concerneo only aboLll lurlher ing
their own interest as they seek to make a
quick profrt even at ihe expense of their
clients'welfare (1m)
% m for 'prdfildriven" (no hint of unethical

0mfor

4.J!g!!gpu9t9a

FYaminers' comments on Q4:

,/t

lo score a full mark for this question. Those who lost half a mark
mostly gave answers that failed to convFy the idea that the companies had a lack of
Ivlosl sludents managed

iniegrity.

5.

Explain in your own wotcls as far as possib/e, the alternatives to soft paternalism
suggested by the author in paragraph L

I2l

a) Existentialist thinker Jean-Paul The author suggests one alternative is the


own
Sa,'1re used io /rs/sl lhat each of us is individual re-shaping his
exerting
his
free lo redefine his charactet characler/personalily by
through an act

of radical

choice

willpower to resisl lemptations. (1m)

(lines 54-56)
OR

lhe aulonomy an rndNidual possesses in


decisaon-makinq allows him 1o shape his
unique personality.
(1m)
The key idea here is thal the individual has
the power Io make changes of his own

b) For the

religiously inclined, an lndividuals can seek supernatural assistance


access to divine grace might be what 1o gain conkol over unhealthy desires (1m)
is needed to stiffen the will. (lines 56
idea of the answer is lifted.

57

Examiners' comments on Q5:


'- Point (b) is !q! an example of point (a) They are tlvo separate and distinct
alternatives to'soft paternalism
- A number of students did nol understand the word tadical" in point (a) and
paraphrased it as "rational"l
" - For point {b), the idea of "supernatural help from a higher powef' has to be made
clear. Hence, answers like "being religious" and "having a strong faith" don't get
the fLl ll mark

Questions on Passage 2

6.

"After all, subduing bad guys is whal the government is all aboul " (lines
lines 12 - '13 implt about how the government perceives the

world?

12

13)

What do
I11

Tne governrnent sees the worlcl in a very simplistic/unsophisticated manner


(1t2 m)

b) ;nd that iis duty is simply to restrain evil OR the world is full
temptationsldangqrs. (1/2 m)
.
Examiners' comments on Q6:
Most students dld noi get the half mark for point (a)

of evil/

Why does the author put lhe word "my" ln italics in line 64?

tzl

lnferred Answer
a) The author wants to emphasise/ highlighU stress (1m) his view

b) that the government should not interyene or influence people's decision_


making even if they may nol always make prudent choices OR the individual should
have the freedom to make his own decisions (1m)

Accept any logical inferred answer.


Examiners' comments on Q7:
- For point (a), students should give an answer lhat conveys some degree of
intensily {"to emphasise" is far betlea than to show", for instance)
- lt is 'to emphasise' rather than "to emphasis" (something)

Using material from paragraphs 5, 7 10, summarise the authois arguments against
paternalism. Write your summary in no more lhan 110 words not counting the opening
words below Use your own wards as taras possib/e.
One of the reasons

aternalism should be

Iel

public officiais, do not, in fact,


know better whether an activity is
detrimental 10 our endurinq self

The oovernment is not wiser about


whether certain actions are really
harmful/benef icial for the individual.

btanket restrictions on
behaviour rides roughshod on
individuality (lines 22 23)

Regulating actions uniformly


suppresses the sense of self.

\lines?1-22L.
passinq

Key point herc is the idea of blanket


Up to what point does the
government get to plal nanny?
Wriere.loes rl end? llines 25 26
Sometimes it is the government
that seems to sufier addiction 10
power (lines 26-27)

It is potentially dangerous as authorities


may not know where to stop
as they may be obsessed with the
influence they have

lf degree af intensity of preoccupation is


not prcsent,award r!
justification
"it
of q for their own ALrthorities are certain to enrich
the
qood" is bound to be abused. ll themselves under the guise of knowing
will be used lo juslify all manner of what is good for the individual
special-interest plundeJ. such as
excessive licensing laws (lines 28 OR
30)

it de-means his

35)

existence (line

l\,4isuse power to benefit themselves


Accept eilher idea af "abuse" or
'specjal interesl plundea
Paternalism degrades self-worlh

..

ft/

Eilher
s)

Grani me the digrify of choosing


which behaviours define my
being (lines 39-40)
To give our existence beautitul
meaninq, (line 45)
It makes existence a happening
rather than a wilful action; (line

It makes the acl of being a random


event rather than one of choice

36)

OR
h)

it makes the story arbitrary and


alien, rather than purposive and
Dersonalised. (line 36-37)
to make ourselves becoming, we
must learn rowto manage our
troublesome impulses But how
do people learn self-command?

It makes life random and remote rather

than individualised and goal oriented.


Paternalism disallows us from acquiring
ways to conkol/deal with our instincts /
human nature.

(lines 45-47)

The second moral argumenl is


that liberty breeds personal
responsibility. (lines 48 49)
. the need for better seltcommand and a will to defeat our
entrenched impulses (lines 5556)

i)

The best teacher is liberty herself.


(lines 48)

k)

From expeaience we
awareness (lines Y-56)

as it deprives us of ihe freedom needed


to do so.

Lifl'libeftv=0ii
gain

Paternalism prevents us from learning


from the past.

Lift'experience'=0m
Kev tdea is the idea of exqerience
ll deteriorates our elhical senses.

it weakens the moral taculty of


choosing one's own course iline

m)

And by presuming that the


individual is incapable of
choosing competently, a

It makes a person unable to make good


choices by assuming and acting as if he
cannot

(lines 58-60)

idea of presumption and


consequence must be mentionecl to get

5B)

paternalistic government may


aclually make him incapable Both

n)

person in a world of Paternalism renders one afraid of selt


decrees and prescriptions, and he reliance and accouniabilaty by diclating
may lear personal indePendence his very actions.
and responsibility- (lines 60-62)
(Each rdea = 1m)

Wean

Sample Answer

/ L3'

There are several good reasons lo reject palernalism The governmenl is not wiser about whether
cedain aclions are really harmful for the individual. Regulating actions uniformly suppresses lhe sense
of self ll is potentially dangerous as aulhorities may not know where to stdp as they may be obsessed
with the influence they have Aulhorities are cedain to enrich themselves undea lhe guise of knowing
Paternalism degrades self worth and makes the act of being a random
what is good for lhe
evenl rather lhan one'ndividual.
of choice lt disallows us from acquiring ways to control oLlr instincts as il
deprives us of tbe freedom needed to do so. ( 100 words)

Examiners commenls on Q8:


Some students still failed 1o indicate the word count or to give the right word counl for their
summary, despite repeated reminders Half a mark was deducted for such scripts.
- Some sludents continued to lift key words/phrases and were heavily penalised as a result
Most made a fair attempt at paraphrasing the key ideas.
- An annoyingly cominon language error is the failure to use "the" before "government'- eg.
''Government may become obsessed wilh the influence it has. "

/2J

Questions on Passages

I and 2

Explain the meaning ol the following words as they are used in the passage. You may
write youa answer in one word oa a short phrase

l
a) nudge, verb
(And what if the
governmenl could
somehow steP in
and nuc,ge lhem in
the right
direction
Passaqe 1 line 11)
b) hedge, rout
(As a freely
chosen healge
against weakness
of the will...

mark

Y,

mark

ioax/persuade/prod/
urqe/push

0 mark
shove/poke/
guide/ direcl/move/
place/ encourage/
steer/ propel/ prompt

barrier/boundary/
obstacle/
obstruclion

fence/border/
enclosure/
prevention/ tool/
preventive measure/
precaution

pleasure-seeking
/devoted to sensual
enjoymenvlhe pursuit
of carnal enjoymenl

self indulging

pursuing of
happiness/
sensual/carnat/ evil/
decadenV enjoyable/
cares only about the
present

unique/distinguishing
/defining/
oLrtstanding/
differentialing/
striking

characterisiic/
conspicuoirs/
noticeable/
obvious/ special/
distinguishable

distinct/ differeni
idiosyncraiic/
recognisable/
significanV importanV
crucial/ typical

reshiction/defence/
/saf eg uard/protection

/shield/restraint

Passaqe 1. line 25)

c) hedonistic,

adjective
(should the
government side
with your prudent
long-rLrn self
against your
hedorlsfic shortrun self?
Passage 1, line 41)

d) distinclive
adjective

quality of humans
is that we do not
simply have
"
desires
Passage 1. line 42)

transitory/

e) fleeting,

adjeclNe
(A f/eel/rg, short
term self that
enjoys
chocolate...
Passage 2, line 4)

impermanenV

kansienvmomentary/
quicklY

'

flittingfading/
quick/short lived/
short term (found in
line 4)

passing/Bassing

swiftly/
passing/temporal/
temporary/ brier/ very
briefl eDhemeral

Examiners' comments on Q9:


- Sorne students gave a list of allernatives as theia answer. When a list is produced,
only the first alternative rn the list is marked, even if lhere were an allernative that
is correct after the first one
Many students did not pay atlent'on to lhe word class of their vocab answer. Eg.

I5l

For the verb 'nudge , many gave the answer 'a gentle push" rather lhan "to push
gently . Such answers wil be penalized in the language mark

10

Jim Holt presents some arguments in suppod ot soft paternal'sm in Passage A while
Daniel Klein is not in favour of it Which writeis views on paternalistic governmenls are
the majority of young people in your country inclined to support? Explain the reasons for
your choice Your answer musl refer lo ideas raised in BOTH passages, as well as your
igl
own ideas and knowledge
Re

uirements:

on paternalislic
people
are
ljkely to suppori'
in tfreir country
governments the majority of young
making reference lo ideas raised by both writers.
They must demonstrate a good understanding of young people in their counlry'
and the influences young people may be subjecl to which would shape their
thinking on ihe kind of government lhey find to be desirable.
There must be adequate support for candidales' answers based on ideas from
both passages, as well as their own relevant knowledge and experience
Candidales must clearly idenlify which wriler's views

Explanation

and-

Evaluation:

Candidates should show understanding


choices and related issues are:

of the issLles they choose

Likely

Passage 1:
1 Palernalislic govenments resttict an individual's libefty (para 1)/ Ctitics clislike
paternalistic governmenls as it involves lhe state s coercive poweL (para 5) Younq people may disapprove of paternalistic governments because lhey are al
the stage where they want to assert their independence, instead of having their
parents, and by exlension the state, breathing down their necks
? An individual is a better judge of what's good far him; people should be'free to
do what they wish as long as their actions don't harm others. (parc'1) Young
people may think that they know what's best for them, rather lhan the state
Even highly competent, wellinformed people fail Io make chaices in their besl
interesl, so the govenment shoultl slep in and nudge them in lhe right direction
(para.2) Young people may lack the maturity and experience lo make prudenl
decisions, so they may feel that the state should guide them to make the right
choices.
4.

6.

The shoft-run self carcs only about lhe present; lhe prudenl resolutions af the
Iong-run self are continua y ignared. The government should side wtth the long
run self to prcmote freedom. (para.6-7) - Young people are generally fun loving
and may hence lack moderation in their pursuil of pleasure. The state should
step in to help them make choices that would promote their liberty
Fxlstertia/lsf lhinket... used to insist thal each of us is free to rcdefine his
character lhrough an act of radical choice. (para 9) -Yottng people may loathe
palernalistic qovernance as ii prevents them from delermining their identity and
becoming who lhey lruly are
An access af divine grace might be what is needed Io stiffen the wi (para g)
Young people may prefer to lurn to religion lo help them overcome
unwholesome impulses or desires
Pary!nsl4q39!9!l!!E!E tglpt a!9L9 !9!93t-greater chance af reaghing orte's

/tJ-

gaal. (para 70) - Yolrng people may not mind governmenl inlervention if they
feelthat their aspirations/goals are more likely lo be realised as a result
Passage 2:
1 The government can lend a helping hand to help one subdue his dafu self
(para 1) / Laws help us not lo ovearse sl/bstances (para 4) - Young people may
feel thai they need the s{ate s assistance to prevent ihem from going astray
2 Other laws protect us from buying on impulse. fram not saving for our old age,
ancl lrom murdering ourcelves (pata.4) Young people may accept
paternalistic laws ii they believe that such legislation safeguards their
inieresVwelfare
3 Government officials do not know befter whether an activity is detimentalto our
enduring self. (para 5) Young people may think that they know what's best for
them, ralher than the state {similar lo idea from Passage 1, pt 2)
4 Passing htanket rcstriclions on behaviout tides roughshod on individualily
(para.5) / Paternalism de-means the individual's existence (para 7) and robs a
person of his dignily (para.B) - Young people may loathe paternalistic
governance as it prevents them from determining their identity and becoming
who they truly are. (similar to idea from Passage 1, pt.5)
5 The government may abuse jts power, using palenalistic laws and policies lo
fufther its own ends (para.s) - Young people may rejecl paternalistic
governments because they believe Such authority is likely to be abused.
6 Libefty breeds personal rcsponsibility (para 8) / From expeience we gain
awareress of the need for better self command and a will to defeat our
entrenched impulses (parc.g) Young people can learn to take responsibility
for their own actions when they arqgiven freedom of choice. They wanl to have
the room to make m'stakes and learn from their experience, hence they prefer
non inlerference from the governmenl.
I Patenalistic govenments weaken lhe moral faculty of choosing an individuals
own course. By presuming lhat lhe individual is incapable of chooshg
competently, a paternalistic govenment may aclually make him incapahle
(para.11) Young people may loathe the idea that the government assumes ii
knows better than lhe individual. When they are nol given freedom of choice,
they become incompetent ;n making decisions.
8 Wean a person in a world of decrees and ptosctiptions, and he may fear
personal independence and responsibility. (para 10) - Young people may feel
that they need to have the liberty to make decisions, so thai they can learn to
take responsibility for their own actions.

q"!J9!99L.

Good answers must fulfil the requirements outlined above and demonstrate
logical flow of ideas with good organisation "and the use of relevant
conneclors/signposting words,

Examiners' commenls on Q10:


A significant number of studenis failed 10 address the question adequately lhe
queslion asks about what most young people in one's country feel regarding
paternalism. not about one's personalopinion regarding paternalism
- Many answers lacked support / evidence for their arguments
- Some even contradicted thdmselves: eg. Some claimed that young people
supporied paternalism at the stad of their response, and that they opposed
palernalism by the end of the response!
Some students seemed to know the writers of the passages personally and

rl,4

addressed them by their first names! Students should use last names "Holt" and
"Klein" instead of first names.
Common language errors inclLrde the failure to use determiners "a" or "the" before
"rnajority"; and the tendency to use colloquial expressions like 'kids" or
"youngsters" in reference to young people.

l0

/Ll

SAJC Mid-Year 07 Paper

Franklin D. Roosevelt said that the world should be founded upon 4


freedoms. Freedom of speech, Freedom from want, Freedom to
worship and Freedom from fear. Which, in your opinion, is the most
important freedom?

2.

'The beauty of the arts lies in it not needing a reason to exist ' Do
you agree with the statement?

3.
4.

To what extent is discrimination present in Singapore today?

5.
6.

'The mass media is no longer reliable.' Do you agree?

'sports is about money.'To what extent is this an accurate


description of sports todaY?

"Ask not what your country can do for you but what you can do for
your country." (J.F Kennedy)Are the youths in your country able to

take up this challenge?

7.
B

Humility has no value in today's world What is your view?

9.
'10

ls it true that Man needs religion now more than ever?

tn an increasingly borderless world, national loyalties have become


unimportant. Do You agree?

'A good
is equally concerned over the state of the world and
' Comment
'eader
his country

11

Youth is easily deceived because it is quick to hope " (Aristotle) Do


you dgree?

12.

'Medical science should do what is always good' even though it


might not be always right ' How true is this stalement?

.LI

SAJC Mid-Year 07
A C Grayling says thal ihere are many reasons why Man goes lo war

ln 1932, Albert Einstein wrole lo S'gmund Freud to ask, Why war? He had come lo
see ihal science cannol explain lhe fact ihal human beings, uniquely among animals'
kill their own kind in highly organised ways, channelling vasl resources into doing so
He fell thal the only defence against fulure war is world government What, he asked
did Freud think?
Freud's answer was pessimistic Violence and inequality, he replied, are nalural to
mankind. History sees weak people banding togelher to oppose strong individuals,
their collective strength evenlually conslitliing a legalorder. Such an order might one
day bring aboul the eagerly desired reign of "everlasling" peace. This peace can be
achieved only by the paradoxical means of the colleclive's power to wage war on war
mongers However, a utopian slale of peace is only theoretically conceivable because
in praclice inequality, aggression and slrife are endemic io human existence
ln this answer there is neither real diagnosis nor real cure. lt assumes lhe naturalness
of man's violence towards man, and sees collective aggression as the sum of
individual aggression. Neither assumpiion, however, is convincingly explored Other
animals are aggressive, but in specific and self'limiling ways: males compete for
males, females protect iheir young, all are wonl 1o squabble over food However'
fighls invariably end when one combalant flees or submns. Apart from man, animals
do not prey on their own kind. A troop of baboons mighl drive away territorial invaders
bul will not pursue them to enslave or kill them. Similarly, social animals often seek
new territory in search of food, bul they do not seek to enlarge exisling lerrilory by
conquest. I\Ian alone does such things. Whalever explains war, th.erefore, has to be
souqhl elsewhere, in lhe complexity of man's economic, political and psychological
conslitulion. Here suggestlons are legion.

On one view, wars are atlempls to solve political and economic problems during

periods of international inslability. Wars involve massive redireclion of produclive and


social forces, lhereby easing, if only lemporarily, the pressures ihal cause them
Moreover they quicken the pace of technological innovalion, and by selting massive
logistical problems ior governmenls they sometimes creaiively forge new polilical and
adminlstralive possibilities War is also seen as a lool by which countries struggle 10
gain advaniage over each other States do not slumble accidenlally into war' bLrl
choose it as a deliberale means of securing influence 5nd control of resources

'10

15

20

?5

30

These analyses say thal wa are instrumenls in the hands of political and commercial
interesls; they are whal leaders gel us into. Forlunately for such leaders, these
analysts point oLrt, public control through the simplistic. themes of nationalism and
patriolism can be counied on to rouse enough young men for slaughlerlo follow

35

The lwentieth cenlury's maior wars might not exactly flt these lheories, bul their joinl
effecl was indeed to shift power and redirect wealthi they weakened Europe and
eslablished the Uniled States as supreme in lhe economic and mililary spheres The
only other combatants lo beneflt rn lhe longer term we those who were so havily
"defeated lhal lhey could reslruclure irom scralch, unhampered by illusions of pasl
glory and the burdens of victory Like many wars, they were revolutionary in impact:
Lenin s Soviei Union was crealed by the First World War, Mao's China by lhe Second

40

One of the iess optimistic obserualions on whrch analysls oi all slnpes agree rs that
war is no longer so conlainable a process Vast leaps in military technology have
made ii tolal and polenlially final. The power of modern weaponry shows that we are

45

/r./

cleverer, but not wiser, than our ancestors, ior we have merely subsliluled the
intercontinental ballisiic missile for the spear, but we slill behave like cavemen
motivaled by greed and fear and unable 10 resist lights
lislen lo views aboul war expressed before lechnology reached its
horror story proportions. There are sane voicas who argued lhal it is betler 10 achieve
one's aims by negotiation than by bloodied swords, and those who indefaligably
opposed war on the grounds thal it harms commerce and hinders progress These
voices, alas, were lhe few unequivocal adversaries ol war Much misplaced optimism
was expressed regarding the subiecl, exlolling u/ar's benefiis in promoiing virility
purifying lhe race, and enhancing crealivity Olhers argued that war gives nations tbelr
place in the world, while simullaneously purging their peoples of effeminacy and
degeneracy. WaI was also regarded as integral lo human idenlity and ungovernable
onae starled, and unsloppable until it exhausls itself. Nevertheless many lhought it
has a place in the critical sludy of history, as the way capilalism willdeslroy ilself

ll is instructive to

Romanticisers of war describe war as a physical necessily for Man, arguing that it
purifies and redeems ilself by bringing grealer evils than itself to an end ln this latler
respect they are right, the war againsi Nazism was necessary in just this way The
romanlicisers of war also decried the mechanisation of war, which meanl the passing
ol coudly conflict, the end of chivakous jousts beh/veen knights under flutlering
banners. They saw no romance in arlillery and mud They had an aesthelic view oi
wart once, it was a sublime enterprise, in which every man put on a crown, when lhe
band of flute-players gave lhe signal for attack; all the shields of the line gliitered wilh
therr high polish, and mingled their splendour with the dark red of the purple mantles
Men were sanctified by combal, {rom which rose all that is highesl in culture There is
no greal arl possible to a nalion," said one romanticiser of war, 'but lhai which is
bdsad on batlle
10

Such misconception is, however, harmless in comparison 10 eugenic theories.


Darwinism led some to treat war as a mechanism of natural seiection lts exlreme
proponenls applauded wafs exlerminalion of inferior individuals and nations Oihers
argued that war is compassionale because it rescues the weak and feeble from lhAir
mGery This viel / did not go unchallenged as c tics argued ihat war is actually
dysgenic, because the titiest and besl march off lo be slaughlered leaving ihe unfil at
home

11

Lo

50

55

60

65

/o

/5

falher childten

The only certainty of war is thal there are always losers in war' Sometimes lhe
grealest losers are the victors. lt mighl be necessary io defend one s fleedoms or to
;ombat such evils as Nazism; so there are iusiifiable wars, and we have 1o be
prepared to fight lhem. However, in its inception and characler lhere ls a prolound
madness in war. To organise murder on lhe large scale - to plan ll lo conduct
scienlific research into it. to build ils instruments in factories, to lrain men for il to
applaud them when they do it; or, on lhe passive side of the equation io sit sileni
when bombed houses, spread eagled bodies, weeping refugees, waste and
deslruction appear on our ielevision screens suggesis thai we have embraced as
normal something vaslly ugly and disaased. Perhaps Einslein's question should not
have been, "Why war?" but "How can we possibly tolerate ils existence?
Adaeiedtton tUhv War,lhe Heart oi ThLnos Applvrna Phrbsoph!

1o

the 21"r Centurv

ao

90

Why does the author use quotation marks when he refers

to

everlasling"

peace in line 9?

Ill

Explain whal the aulhor means when he says thal peace can be achieved
only by the 'paradoxical means of the colleclive s power to wage war on warmonoers" (line10 11). Use yaut own words as faras poss/b/e.
I2l
l)sing yaur awn words as far as possib/e, slaie how, in paragraph 3, I\lan s
aggression is different lrom that of animals.
l2l

Explain what is meant by "we are cleverer, bul nol wiser, than our
anceslors . in liae 47 Use your own words as far as possible
I2l

(i) The aulhor does noi fully supporl lhe romanlicisers views on war

in

paragraph 9. SIate in your awn words as far as poss/b/e in what way lheir
views are similar and how lhey are
I31

differenl

(i!) What does the term, tomanticers of war" (line 61) tell you aboul the

authols atlitude towards those who supporl

wat?

I1l

From pa.agraph 10, explain why the authorthinks that juslificalio n for war
based on eugenic theories is harmful?
I1l
Why does the author claim that "the grealest iosers are lhe victors" in
paragraph 1'1?

t1l

The aulhor says that lhere is "profound madness' in war (lines 83-84).
LJsng yaur own words as /ar as possib/e, explain what he means by this

121

Give ihe meaning of the following words as they are used in lhe passage
You may write your answers in one word or a shorl phrase
(a) endemic (line 12)
(b) legion (line 24)
(c) unhampered (line 41)
(d) instructive (line 50)
(e) extolling (line 55)

10

15t

Using material from paragraphs 4-8 (lines 25-60) summarise the authofs
reasons for why people wage war on each other and ihe outcomes of war.
Write your summary in no more than 150 words not counting the opening
words which are prinled below Use yoor own words as far as possib/e [7]
PPaptP $dqP

11

\ a, bP

dLse

ln the passage, Graylinq presenis difierenl views on war. Which of these


views do you agree with? Are these views still reflected in the
Juslify your answer by referring both io what you have read in the passage
and your ideas and knowledge
t8l

/l/

You might also like